You are on page 1of 61

7.3.

INTEGRATION BY PARTS 419


Z
27. e−2θ cos θ dθ u = e−2θ , du = −2e−2θ dθ; dv = cos θ dθ, v = sin θ
Z
= e−2θ sin θ − −2e−2θ sin θ dθ

u = e−2θ , du = −2e−2θ dθ; dv = sin θ dθ, v = − cos θ


 Z 
= e−2θ sin θ + 2 −e−2θ cos θ − 2e−2θ cos θ dθ
Z
= e−2θ sin θ − 2e−2θ cos θ − 4 e−2θ cos θ dθ
Z
sin θ − 2 cos θ
Solving for the integral, we have e−2θ cos θ dθ = + C.
5e2θ
Z
1
28. eαx sin βx dx u = eαx , du = αeαx dx; dv = sin βx dx, v = − cos βx
β
Z
1 α
= − eαx cos βx − − eαx cos βx dx
β β
1
u = eαx , du = αeαx dx; dv = cos βx dx, v = sin βx
β
 Z 
1 α 1 αx α αx
= − eαx cos βx + e sin βx − e sin βx dx
β β β β
2 Z
α 1 α
= 2 eαx sin βx − eαx cos βx − 2 eαx sin βx dx
β β β
Z
eαx (α sin βx − β cos βx)
Solving for the integral, we have eαx sin βx dx = + C.
α2 + β 2
Z
29. θ sec θ tan θ dθ u = θ, du = dθ; dv = sec θ tan θ dθ, v = sec θ
Z
= θ sec θ − sec θ dθ = θ sec θ − ln | sec θ + tan θ| + C
Z
30. e2t cos et dt u = et , du = et dt; dv = et cos et dt, v = sin et
Z
= et sin et − et sin et dt = et sin et + cos et + C
Z
31. sin x cos 2x dx u = cos 2x, du = −2 sin 2x dx; dv = sin x dx, v = − cos x
Z
= − cos x cos 2x − 2 cos x sin 2x dx

u = 2 sin 2x, du = 4 cos 2x dx; dv = cos x dx, v = sin x


 Z 
= − cos x cos 2x − 2 sin x sin 2x − 4 sin x cos 2x dx
420 CHAPTER 7. TECHNIQUES OF INTEGRATION
Z
1 2
Solving for the integral, we have sin x cos 2x dx = cos x cos 2x + sin x sin 2x + C.
3 3
Z
32. cosh x cosh 2x dx u = cosh 2x, du = 2 sinh 2x dx; dv = cosh x dx, v = sinh x
Z
= sinh x cosh 2x − 2 sinh x sinh 2x dx

u = 2 sinh 2x, du = 4 cosh 2x dx; dv = sinh x dx, v = cosh x


 Z 
= sinh x cosh 2x − 2 sinh 2x cosh x − 4 cosh x cosh 2x dx
Z
2 1
Solving for the integral, we have cosh x cosh 2x dx = sinh 2x cosh x − sinh x cosh 2x + C.
3 3

Z p p 1
33. x3 x2 + 4 dx u = x2 , du = 2x dx; dv = x x2 + 4 dx, v = (x2 + 4)3/2
3
Z
1 2
= x2 (x2 + 4)3/2 − x(x2 + 4)3/2 dx
3 3
1 2
= x2 (x2 + 4)3/2 − (x2 + 4)5/2 + C
3 15

Z
t5 1
34. dx u = t3 , du = 3t2 dt; dv = t2 (t3 + 1)−2 dt, v = − (t3 + 1)−1
(t3 + 1)2 3
Z
1 3 3 1 1
= − t (t + 1)−1 + t2 (t3 + 1)−1 dt = − t3 (t3 + 1)−1 + ln |t3 + 1| + C
3 3 3

Z
1
35. sin(ln x) dx u = sin(ln x), du = cos(ln x) dx; dv = dx, v = x
x
Z
= x sin(ln x) − cos(ln x) dx

1
u = cos(ln x), du = − sin(ln x) dx; dv = dx, v = x
x
 Z 
= x sin(ln x) − x cos(ln x) + sin(ln x) dx
Z
1 1
Solving for the integral, we have sin(ln x) dx = x sin(ln x) − x cos(ln x) + C.
2 2

Z
cos x
36. cos ln(sin x) dx u = ln(sin x), du = dx; dv = cos x dx, v = sin x
sin x
Z
= sin x ln(sin x) − cos x dx = sin x ln(sin x) − sin x + C
7.3. INTEGRATION BY PARTS 421
Z
37. csc3 x dx u = csc x, du = − csc x cot x dx; dv = csc2 x dx, v = − cot x
Z Z
= − csc x cot x − csc x cot2 x dx = − csc x cot x − csc x(csc2 x − 1) dx
Z Z
= − csc x cot x − csc x dx + csc x dx
3

Z
= − csc x cot x − csc3 x dx + ln | csc x − cot x|
Z
1 1
Solving for the integral, we have csc3 x dx = − csc x cot x + ln | csc x − cot x| + C.
2 2
Z
1 1
38. x sec−1 x dx u = sec−1 x, du = √ dx; dv = x dx, v = x2
x x −1
2 2
Z   Z
1 2 −1 1 x2 1 2 −1 1
= x sec x − √ dx = x sec x − x(x2 − 1)−1/2 dx
2 2 x x2 − 1 2 2
Z
1 2 −1 1
= x sec x − (x2 − 1)−1/2 (2x dx) t = x2 − 1, dt = 2x dx
2 4
Z
1 2 −1 1 1 1
= x sec x − t−1/2 dt = x2 sec−1 x − (2t1/2 ) + C
2 4 2 4
1 2 −1 1p 2 1 2 −1 p
= x sec x − x − 1 + C = (x sec x − x2 − 1) + C
2 2 2
Z
39. x sec2 x dx u = x, du = dx; dv = sec2 x dx, v = tan x
Z
= x tan x − tan x dx = x tan x − ln | sec x| + C

Z Z Z Z
40. x tan2 x dx = x(sec2 x − 1) dx = x sec2 x dx − x dx

u = x, du = dx; dv = sec2 x dx, v = tan x


Z
1 1
= x tan x − tan x dx − x2 = x tan x − ln | sec x| − x2 + C
2 2
Z 2
1 1
41. x ln(x + 1) dx u = ln(x + 1), du = dx; dv = x dx, v = x2
0 x+1 2
2 Z 2  2 
1 1 x
= x2 ln(x + 1) − dx
2 0 0 2 x +1
Z  
1 1 2 1
= (4 ln 3 − 0) − x−1+ dx
2 2 0 x+1
  2
1 1 2 1 3
= 2 ln 3 − x − x + ln(x + 1) = 2 ln 3 − (ln 3 − 0) = ln 3
2 2 0 2 2
422 CHAPTER 7. TECHNIQUES OF INTEGRATION

Z 1
2x
42. ln(x2 + 1) dx u = ln(x2 + 1), du = dx; dv = dx, v = x
0 x2 + 1
Z Z 1 
1 2x2 1
1
= x ln(x2 + 1) − dx = (ln 2 − 0) − 2 1 − dx
0 x +1 x2 + 1
0 2
0
1 h π i π
= ln 2 − 2 x − tan−1 x 0 = ln 2 − 2 1 − − (0 − 0) = + ln 2 − 2
4 2
Z 4
43. xe−x/2 dx u = x, du = dx; dv = e−x/2 dx, v = −2e−x/2
2
i4 Z 4 i4
= −2xe−x/2 − (−2e−x/2 ) dx = −2(4e−2 − 2e−1 ) − 4e−x/2
2 2 2
4 8 8 12 8e − 12
= − 2 − 4(e−2 − e−1 ) = − 2 =
e e e e e2
Z π
44. ex cos x dx u = cos x, du = − sin x dx; dv = ex dx, v = ex
−π
Z π
π
= ex cos x]−π + ex sin x dx
−π

u = sin x, du = cos x dx; dv = ex dx, v = ex


Z π
π
= [−eπ − (−e−π )] + ex sin x]−π − ex cos x dx
−π
Z π
= e − e + (0 − 0) −
−π π
e cos x dx
x
−π
Z π
1 −π
Solving for the integral, we have ex cos x dx = (e − eπ ).
−π 2
Z 1
1
45. tan−1 x dx u = tan−1 x, du = dx; dv = dx, v = x
0 1 + x2
Z 1 π  1 1
1 x
= x tan−1 x 0 − dx = − 0 − ln(1 + x 2
)
0 1+x 4 2
2
0
π 1 π 1
= − (ln 2 − 0) = − ln 2
4 2 4 2
Z √
2/2
1
46. cos−1 x dx u = cos−1 x, du = − √ dx; dv = dx, v = x
0 1 − x2
Z √2/2
√2/2 x
= x cos x 0
−1
− −√ dx
0 1 − x2
√ ! Z √2/2
2π 1
= −0 − (1 − x2 )−1/2 (−2x dx)
8 2 0
Z √ p
t = 1 − x2 , dt = −2x dx; t−1/2 dt = 2t1/2 = 2 t = 2 1 − x2
7.3. INTEGRATION BY PARTS 423

√ √ √ r ! √ √
2π 1 p  2/2 2π 1 √ 2π 2
= − 2 1−x2 = − − 1 = − +1
8 2 0 8 2 8 2
Z 3
1
47. A = (1 + ln x) dx u = 1 + ln x, du = dx; dv = dx, v = x
e−1 x 2
Z 3
3 3
= (x + x ln x)]e−1 − dx = (3 + 3 ln 3) − (e−1 − e−1 ) − x]e−1
e−1
 
1 1 1/e 3
= 3 + 3 ln 3 − 3 − = 3 ln 3 +
e e
Z 0 Z 1
48. A = − tan−1 x dx + tan−1 x dx
−1 0

1 -1 1
u = tan−1 x, du = dx; dv = dx, v = x
1 + x2
  Z Z 1
0
x 0 1 x
= − x tan x −1 − −1
dx + x tan−1
x − dx
−1 1 + x 0 1+x
2 0 2
" 0 # " 1 #
 π 1 π  1
=− 0− − ln(1 + x )
2
+ − 0 − ln(1 + x ) 2
4 2 −1 4 2 0
π 1 π 1 π
= + (0 − ln 2) + − (ln 2 − 0) = − ln 2
4 2 4 2 2
Z 5
2 ln x 2

49. V = π (ln x)2 dx u = (ln x)2 , du = dx; dv = dx, v = x 1

1 x
Z 5 5

5
= πx(ln x)2 1 − π 2 ln x dx
1
2
u = 2 ln x, du =dx; dv = dx, v = x
x
 Z 5 
5
= π[5(ln 5) − 0] − π 2x ln x]1 −
2
2 dx
1
h i
5
= 5π(ln 5)2 − π (10 ln 5 − 0) − 2x]1 = 5π(ln 5)2 − 10π ln 5 + 8π
Z ln 3 Z ln 3 Z ln 3
50. V = 2π x(3 − e ) dx = 6π
x
x dx − 2π xex dx 3

0 0 0

u = x, du = dx; dv = e dx, v = ex
x 2

Z ln 3 !
ln 3 x ln 3
= 3πx2 0
− 2π xe ]0 − x
e dx 1

0
h i
ln 3
= 3π(ln 3)2 − 2π (3 ln 3 − 0) − ex ]0 = 3π(ln 3)2 − 6π ln 3 + 4π ln 3
424 CHAPTER 7. TECHNIQUES OF INTEGRATION

Z π
51. V = 2π x sin x dx
u = x, du = dx; dv = sin x dx, v = − cos x
1

0
 Z π  !

π π
= 2π −x cos x]0 + cos x dx = 2π [−(−π − 0) + sin x]0 ] = 2π 2
0

1 sin x
52. y 0 = (− sin x) = − = − tan x
cos x cos x
Z π/4 p Z π/4 p Z π/4 √ Z π/4
s= 1 + tan x dx =
2
1 + (sec x − 1) dx =
2 sec x dx =
2 | sec x| dx
0 0 0 0
On [0, π/4], sec x > 0, so we have
Z π/4 √ √
π/4
s= sec x dx = ln |sec x + tan x| ]0 = ln | 2 + 1| − ln |1 + 0| = ln(1 + 2).
0

Z 2
1 x x 2
53. fave = tan−1 dx u = tan−1 , du = dx; dv = dx, v = x
2−0 0 2 2 4 + x2
 i2 Z 2 2x     i2 
1 −1 x 1 π
= x tan − dx = 2 − ln(4 + x )
2
2 2 0 0 4+x
2 2 4 0
1 hπ i π 1
= − (ln 8 − ln 4) = − ln 2
2 2 4 2
Z
54. s(t) = e−t sin t dt u = e−t , du = −e−t dt; dv = sin t dt, v = − cos t
Z
= −e cos t − e−t cos t dt
−t
u = e−t , du = −e−t dt; dv = cos t dt, v = sin t
 Z 
= −e−t cos t − e−t sin t + e−t sin t dt
Z
1 1
Solving for the integral, we have s(t) = e−t sin t dt = − e−t cos t − e−t sin t + C. Now
2 2
1 1 1
0 = s(0) = − + C, so C = and s(t) = (1 − e−t cos t − e−t sin t).
2 2 2
Z
55. v(t) = te−t dt u = t, du = dt; dv = e−t dt, v = −e−t
Z
= −te−t + e−t dt = −te−t − e−t + C.

Now 1 = v(0) = −1 + C, so C = 2 and v(t) = −te−t − e−t + 2.


Z Z
s(t) = (−te−t − e−t + 2) dt = − te−t dt + e−t + 2t = −(−te−t − e−t ) + e−t + 2t + C

= te−t + 2e−t + 2t + C.
Now −1 = s(0) = 2 + C, so C = −3 and s(t) = te−t + 2e−t + 2t − 3.
7.3. INTEGRATION BY PARTS 425
Z 1 Z 1
56. W = 62.4π x sin2 πx dx = 62.4π
x
(1 − cos 2πx) dx 1
1/2 1/2 2 y
Z 1 Z y
1
1/2
= 31.2π x dx − 31.2π x cos 2πx dx 1
1/2 1/2

1
u = x, du = dx; dv = cos 2πx dx, v = sin 2πx x

1 Z 1 !
1 1 1
= 15.6πx2 − 31.2π
x sin 2πx − sin 2πx
1/22π 1/2 1/2 2π
  1 !
1 1
= 15.6π 1 − − 31.2π 0 + cos 2πx
4 4π 2 1/2
7.8 15.6
= 11.7π − (1 + 1) = 11.7π − ≈ 31.7910 ft-lb
π π
57. Using symmetry,
 Z 2 
πx πx 4 πx
F = 2 62.4 x cos dx u = x, du = dx; dv = cos dx, v = sin
0 4 4 π 4
2 Z 2 ! 2 !
4x πx 4 πx 8 16 πx
= 124.8 sin − sin dx = 124.8 + 2 cos
π 4 0 π 0 4 π π 4 0
 
8 16
= 124.8 − 2 ≈ 115.4825 lb
π π
Z π/2
π/2
58. A = sin x dx = − cos x]0 = −(0 − 1) = 1 2

0
Z π/2
1

My = x sin x dx u = x, du = dx; dv = sin x dx, v = − cos x !


0
Z π/2 -1
π/2 π/2
= −x cos x]0 + cos x dx = −(0 − 0) + sin x]0 =1
0
Z Z  π/2
1 π/2 2 1 π/2 1 1 1 π π
Mx = sin x dx = (1 − cos 2x) dx = x − sin 2x = =
2 0 4 0 4 2 0 4 2 8
1 π/8 π
x = = 1, y = =
1 1 8
Z 4 √
tan −1
x √ 1
59. √ dx t = x, dt = √ dx
1 x 2 x
Z 2
1
=2 tan−1 t dt u = tan−1 t, du = dt; dv = dt, v = t
1 1 + t2
 Z 2  " 2 #
 t  π 1
−1 2
= 2 t tan t 1 − dt = 2 2 tan 2 − −1
− ln(1 + t )
2
1 1+t 4 2
2
1
π π 5
= 4 tan−1 2 − − (ln 5 − ln 2) = 4 tan−1 2 − − ln
2 2 2
426 CHAPTER 7. TECHNIQUES OF INTEGRATION
Z √ √
60. xe x
dx t= x, x = t2 , dx = 2t dt
Z Z
= t2 et (2t dt) = 2 t3 et dt u = t3 , du = 3t2 dt; dv = et dt, v = et
 Z 
= 2 t e − 3t e dt
3 t 2 t
u = t2 , du = 2t dt; dv = et dt, v = et
 Z 
= 2t e − 6 t e − 2te dt
3 t 2 t t
u = t, du = dt; dv = et dt, v = et
 Z 
= 2t e − 6t e + 12 te − e dt = 2t3 et − 6t2 et + 12tet − 12et + C
3 t 2 t t t

√ √ √ √ √
= 2x3/2 e x
− 6xe x
+ 12 xe x − 12e x + C
Z
√ √
61. sin x + 2 dx t= x + 2, x = t2 − 2, dx = 2t dt
Z
= (sin t)2t dt u = 2t, du = 2 dt; dv = sin t dt, v = − cos t
Z
= −2t cos t + 2 cos t dt = −2t cos t + 2 sin t + C
√ √ √
= −2 x + 2 cos x + 2 + 2 sin x + 2 + C

Z π2 √ √
62. cos t dt x= t, t = x2 , dt = 2x dx
0
Z π Z π
= cos x(2x dx) = 2 x cos x dx
0 0

u = x, du = dx; dv = cos x dx, v = sin x


 Z π 
π π
= 2 x sin x]0 − sin x dx = 2 cos x]0 = 2(−1 − 1) = −4
0

Z
n(ln x)n−1
63. (ln x)n dx u = (ln x)n , du = dx; dv = dx, v = x
x
Z
= x(ln x)n − n (ln x)n−1 dx

Z
64. sinn x dx u = sinn−1 x, du = (n − 1) sinn−2 x cos x dx; dv = sin x dx, v = − cos x
Z
= − sinn−1 x cos x + (n − 1) cos2 x sinn−2 x dx
Z
= − sinn−1 x cos x + (n − 1) (1 − sin2 x) sinn−2 x dx
Z Z
= − sinn−1 x cos x + (n − 1) sinn−2 x dx − (n − 1) sinn x dx
7.3. INTEGRATION BY PARTS 427
Z
Solving for the integral sinn x dx, we have

Z Z
sinn−1 x cos x n − 1
sinn x dx = − + sinn−2 x dx.
n n
Z
65. cosn x dx u = cosn−1 x, du = −(n − 1) cosn−2 x sin x dx; dv = cos x dx, v = sin x
Z
= cosn−1 x sin x + (n − 1) sin2 x cosn−2 x dx
Z
= cos n−1
x sin x + (n − 1) (1 − cos2 x) cosn−2 x dx
Z Z
= cosn−1 x sin x + (n − 1) cosn−2 x dx − (n − 1) cosn x dx
Z
Solving for the integral cosn x dx, we have

Z Z
cosn−1 x sin x n − 1
cosn x dx = + cosn−2 x dx.
n n
Z
66. secn x dx u = secn−2 x, du = (n − 2) secn−2 x tan x dx; dv = sec2 x dx, v = tan x
Z
= secn−2 x tan x − (n − 2) secn−2 x tan2 x dx
Z
= secn−2
x tan x − (n − 2) secn−2 x(sec2 x − 1) dx
Z Z
= secn−2 x tan x − (n − 2) secn x dx + (n − 2) secn−2 x dx
Z
Solving for secn x dx, we have

Z Z
secn−2 x tan x n − 2
secn x dx = + secn−2 x dx, n 6= 1.
n−1 n−1
Z Z
sinn−1 x cos x n − 1
67. Using sinn x dx = − + sinn−2 x dx with n = 3,
n n
Z Z
sin2 x cos x 2 sin2 x cos x 2
sin x dx = −
3
+ sin x dx = − − cos x + C.
3 3 3 3
Z Z
secn−2 x tan x n − 2
68. Using secn x dx = + secn−2 x dx with n = 4,
n−1 n−1
Z Z
sec2 x tan x 2 sec2 x tan x 2
sec x dx =
4
+ sec2 x dx = + tan x + C.
3 3 3 3
428 CHAPTER 7. TECHNIQUES OF INTEGRATION
Z Z
cosn−1 x sin x n − 1
69. Using cosn x dx = + cosn−2 x dx with n = 3,
n n
Z
cos3 10x dx u = 10x, du = 10 dx
Z Z
1 cos2 u sin u 2
= cos3 u du = + cos u du
10 10 · 3 10 · 3
cos2 u sin u 1 cos2 10x sin 10x 1
= + sin u + C = + sin 10x + C.
30 15 30 15
Z Z
cosn−1 x sin x n − 1
70. Using cos x dx =
n
+ cosn−2 x dx with n = 4,
n n
Z Z  Z 
cos3 x sin x 3 cos3 x sin x 3 cos x sin x 1
cos x dx =
4
+ cos x dx =
2
+ + cos x dx
0
4 4 4 4 2 2
 Z 
cos3 x sin x 3 cos3 x sin x 3
= + cos x sin x + dx = + (cos x sin x + x) + C.
4 8 4 8

Z Z
sinn−1 x cos x n − 1
71. Using sinn x dx = − + sinn−2 x dx, we have
n n
Z π/2 Z
π/2
sinn−1 x cos x n − 1 π/2 n−2
sin x dx = −
n
+ sin x dx
0 n 0 n 0
 n−1  Z
sin (π/2) cos(π/2) sinn−1 0 cos 0 n − 1 π/2 n−2
=− − + sin x dx
n n n 0
Z Z
n − 1 π/2 n−2 n − 1 π/2 n−2
= −(0 − 0) + sin x dx = sin x dx.
n 0 n 0

Z Z
π/2
n−1 π/2
72. Repeated use of sinn x dx = sinn−2 x dx yields
0 n 0

Z Z
π/2
n−1 n−3 k+3 k+1 π/2
sin x dx =
n
· ··· · sink x dx,
0 n n−2 k+4 k+2 0

where k = 0 when n is even and n ≥ 2, and k = 1 when n is odd and n ≥ 3. Thus, we get,
respectively:
Z Z
π/2
n−1 n−3 3 1 π/2
(a) sinn x dx = · ··· · sin0 x dx
0 n n−2 4 2 0
Z
n−1 n−3 3 1 π/2
= · ··· · dx
n n−2 4 2 0
n−1 n−3 3 1 π  π 1 · 3 · 5 · · · (n − 1)
= · ··· · −0 = · .
n n−2 4 2 2 2 2 · 4 · 6···n
7.3. INTEGRATION BY PARTS 429

Z Z
π/2
n−1 n−3 4 2 π/2 1
(b) sinn x dx = · ··· · sin x dx
0 n n−2 5 3 0
n−1 n−3 4 2 π/2
= · ··· · (− cos x)]0
n n−2 5 3
n−1 n−3 4 2 2 · 4 · 6 · · · (n − 1)
= · ··· · [0 − (−1)] = .
n n−2 5 3 3 · 5 · 7···n
Z π/2
π 1·3·5·7 105π 35π
73. sin8 x dx = · = =
0 2 2·4·6·8 768 256
Z π/2
2·4 8
74. sin5 x dx = · =
0 3·5 15
Z
ex 1
75. e2x tan−1 ex dx u = tan−1 ex , du = dx; dv = e2x dx, v = e2x
1 + e2x 2
Z  3x 
1 1 e
= e2x tan−1 ex − dx t = ex , dt = ex dx
2 2 1 + e2x
Z Z  
1 1 t2 1 2x 1 1
= e2x tan−1 ex − dt = e tan −1 x
e − 1 − dt
2 2 1 + t2 2 2 1 + t2
1 1 1 1 1
= e2x tan−1 ex − t + tan−1 t + C = (e2x + 1) tan−1 ex − ex + C
2 2 2 2 2
Z
2 sin−1 x
76. (sin−1 x)2 dx u = (sin−1 x)2 , du = √ dx; dv = dx, v = x
1 − x2
Z
2x sin−1 x
= x(sin−1 x)2 − √ dx
1 − x2
1 2x p
u = sin−1 x, du = √ dx; dv = √ dx, v = −2 1 − x2
1 − x2 1 − x2
 p Z 
= x(sin−1 x)2 − −2 1 − x2 sin−1 x − −2 dx
p
= x(sin−1 x)2 + 2 1 − x2 sin−1 x − 2x + C
Z
xex 1 1
77. dx u = xex , du = (x + 1)ex dx; dv = dx, v = −
(x + 1)2 (x + 1)2 x+1
Z  
x x x 1 x ex
=− e + e dx = 1 −
x
ex + C = e +C = +C
x+1 x+1 x+1 x+1
Z
x2 e x 1 1
78. dx u = x2 ex , du = x(x + 2)ex dx; dv = dx, v = −
(x + 2)2 (x + 2)2 x+2
Z
1
=− x e + xex dx
2 x
u = x, du = dx; dv = ex dx, v = ex
x+2
Z  
x2 x x2 x−2 x
=− e + xe − e dx = x − 1 −
x x
ex + C = e +C
x+2 x+2 x+2
430 CHAPTER 7. TECHNIQUES OF INTEGRATION
Z
79. We first compute ex sin x dx:

Z
ex sin x dx u = ex , du = ex dx; dv = sin x dx, v = − cos x
Z
= −ex cos x + ex cos x dx

u = ex , du = ex dx; dv = cos x dx, v = sin x


Z
= −ex cos x + ex sin x − ex sin x dx.

Z Z
1 x
Solving for the integral, we have ex sin x dx = e (sin x − cos x). Similarly, ex cos x dx =
2
1 x
e (sin x + cos x). Then
2

Z
1
xex sin x dx u = x, du = dx; dv = ex sin x dx, v = ex (sin x − cos x)
2
Z
1 x 1
= xe (sin x − cos x) − (ex sin x − ex cos x) dx
2 2
 
1 x 1 1 x 1 x
= xe (sin x − cos x) − e (sin x − cos x) − e (sin x + cos x) + C
2 2 2 2
1 1 1
= xex sin x − xex cos x + ex cos x + C.
2 2 2

Z
80. We first compute e−x cos 2x dx:

Z
1
e−x cos 2x dx u = e−x , du = −e−x dx; dv = cos 2x dx, v = sin 2x
2
Z
1 −x 1
= e sin 2x + e−x sin 2x dx
2 2
1
u = e−x , du = −e−x dx; dv = sin 2x dx, v = − cos 2x
2
 Z 
1 −x 1 1 −x 1
= e sin 2x + − e cos 2x − e cos 2x dx .
−x
2 2 2 2

Z
1 −x
Solving for the integral, we have e−x cos 2x dx = e (2 sin 2x − cos 2x). Similarly,
5
7.3. INTEGRATION BY PARTS 431
Z
1
e−x sin 2x dx = − e−x (sin 2x + 2 cos 2x). Then
5
Z
1
xe−x cos 2x dx u = x, du = dx; dv = e−x cos 2x dx, v = ex (2 sin 2x − cos 2x)
5
Z
1 1
= xe−x (2 sin 2x − cos 2x) − (2e−x sin 2x − e−x cos 2x) dx
5 5
 
1 2 1
= xe−x (2 sin 2x − cos 2x) − − e−x (sin 2x + 2 cos 2x)
5 5 5
 
1 1 −x
+ e (2 sin 2x + cos 2x) + C
5 5
2 −x 1 4 3
= xe sin 2x − xe−x cos 2x + e−x sin 2x + e−x cos 2x + C.
5 5 25 25
Z  p 
81. ln x + x2 + 1 dx

  √
p 1 + x/ x2 + 1
u = ln x + x2 +1 , du = √ dx; dv = dx, v = x
x + x2 + 1

p  Z  
1 x
= x ln x + x + 1 −
2 √ 1+ √ x dx
x + x2 + 1 x2 + 1
√ √ !
 p  Z x − x2 + 1 x2 + 1 + x
= x ln x + x2 + 1 − √ x dx
x − (x + 1)
2 2
x2 + 1
 p  Z p  p  x

= x ln x + x2 + 1 − x +1−x
2 x +1+x
2 √ dx
x2 + 1
 p  Z x
= x ln x + x + 1 −
2 √ dx
x +1
2
 p  p
= x ln x + x2 + 1 − x2 + 1 + C

Z −1
−1 −1 esin x
82. esin x
dx u = esin x
, du = √ dx; dv = dx, v = x
1 − x2
Z −1
sin−1 x xesin x
= xe − √ dx
1 − x2
−1
−1
esin x x p
u = esin , du = √ x
dx; dv = √ dx, v = − 1 − x2
1−x 2 1−x 2
 p Z 
sin−1 x sin−1 x sin−1 x
= xe − −e 1 − x − −e
2 dx
p Z
−1 −1 −1
= xesin x + esin x 1 − x2 − esin x dx
Z −1
p
−1 esin x
Solving for the integral, we have esin x
dx = (x + 1 − x2 ) + C.
2
432 CHAPTER 7. TECHNIQUES OF INTEGRATION

83. (a) Graph shown at right.


3
(b) We use the reduction formula
Z Z
sinn−1 x cos x n − 1
sinn x dx = − + sinn−2 x dx π 2π
n n
with n = 4 and n = 2.
Z 2π
A= (3 + sin2 x − 5 sin4 x) dx
0
!
Z 7.4.  2π Z 431

sin3 x cos x 3 2π 2
POWERS OF TRIGONOMETRIC FUNCTIONS

= 3x]0+2 sin x dx − 5 − 2
+ figure
83. (a) TODO sin x dx
0 4 0 4 0 !
sin x cos x n − 1
! n−1
(b) We use the reduction formula sin x dx = − + n
sin n−2
x dx with
  Z 2π n = 4!and n = 2. 2π Z n2π !n
0 0 7 7 sin x cos x 2π
1
= 6π + 5 − + sin2 x dx = 6π − A = −(3 + sin! x − 5 sin x) dx" + # ! dx $
2 4

4 4 4 0 4 2 sin x cos2x
0

0+ 3
2π 3 2π
7.4. POWERS OF TRIGONOMETRIC = 3x] FUNCTIONS sin x dx − 5 0− sin431

+2 2
x dx 2
0
4 4
2π % &
0
" #
0 0
$
7 17π 83. (a) TODO figure ! ! 2π
0 0 7 ! 7 − sin x cos x

1 2π
= 6π +! 5 − + − sin x dx = 6π − +2

= 6π − x =
dx
sin x cos x n − 1 4 n−1
. (b) We use the reduction formula sin 4x dx4= − 4 + n
sin 2 with
x dx 0 2 n−2 0 0

8 0 4 n = 4!and n = 2. = 6π − x
7
#
=
17π
.
n n


8 4
A= (3 + sin2 x − 5 sin4 x) dx 0
0 84. ! (a) TODO figure 3$
84. (a) Graph shown at right.
" #2π !

sin3 xand
cos second
x 3 2π intersection
= 3x]0 + 2 (b)sin For
2 x > 0, the first
+points of of y = x sin x and y = x cos x are

x dx − 5 − sin2 x dx
0 x1 = π/4 and x2 = 5π/4. 4 0 4 0

(b) For x > 0, the first and second points of intersection of y = x sin x % & ! !5π/4 " ! 5π/4 #2π ! 2π $
0 0A = 7 2π(x sin x cos 7 sin x cos x 1
= 6π + 5 − + − −=
sin2 xxdx − dx =−
6π x) + cos x) dx
x(sin x − dx
4 4 4π/40 4 π/4 2 2 0
and y = x cos x are x1 = π/4 and x = 5π/4. 2 7
#2π
17π
0
u = x, du = dx; dv = (sin x − cos x) dx, v = − cos x − sin x
!
Z 5π/4 Z 5π/4
= 6π − x
8 0
=
4
.
5π/4
! 5π/4
= −x(cos x + sin x)]π/4 − −(cos x + sin x) dx
A= (x sin x − x cos x) dx = x(sin x − cos x) dx
84. (a) TODO figure
'
(b) For x > 0, the first and second
" √ √ $
5π points 2of intersection
2
π/4
"√ √ $(
2 x-3
π of y2 = x sin and y = x cos x are
5π/4
π/4 π/4 x1 = π/4 and x2 = 5π/4.= − 4 − 2 − 2 + 4 2 + 2 + (sin x − cos x)]π/4
! 5π/4 !'"
√ 5π/4√ √ $ "√ √ $( √
u = x, du = dx;
dv = (sin x − cos x) dx, v = − cos x − sin x
A=
π/4
(x sin x − x cos x)
=
3π dx2 =
2
+ π/4−
2
2 x−
x(sin
+
2
2 cos x) dx2

2

2
2
=
3π 2
2
u = x, du = dx; dv = (sin x − cos x) dx, v = − cos x − sin x
Z 5π/4 7.4 Powers ! 5π/4
of Trigonometric Functions
5π/4
5π/4 = −x(cos x + sin x)]π/4 − −(cos x + sin x) dx
= −x(cos x + sin x)]π/4 − −(cos x' +"sin
1. √ x) dx " √ √ $(
√ $
π/4

π/4 5π 2 2 π 2 2 5π/4
= − − − + + + (sin x − cos x)]π/4
" 4 2. 2
√ √ ! √ √ !# √ 3.'" √
2 4
√ $ "√
2 2
√ $( √
5π 2 2 π 2 2 =
3π 2
+
2
+
2 2 2 3π 2
=5π/4
= − + + + (sin x − cos x)]π/4
− − −
− − 2 4. 2 2 2 2 2
4 2 2 4 2 2 5.
7.4 Powers of Trigonometric Functions
√ " √ √ ! √1. √ !# 6. √
3π 2 2 2 2 2 7. 3π 2
= + − + − 2. − =
2 2 2 2
3. 2 8. 2
4.
5.

7.4 Powers of Trigonometric Functions 6.


7.
Z 8.

1. sin1/2 x cos x dx u = sin x, du = cos x dx


Z
2 3/2 2
= u1/2 du = u + C = (sin x)3/2 + C
3 3
Z
2. cos4 5x sin 5x dx u = cos 5x, du = −5 sin 5x dx
Z  
1 1 1
= u − du = − u5 + C = − cos5 5x + C
4
5 25 25
7.4. POWERS OF TRIGONOMETRIC FUNCTIONS 433
Z Z Z Z Z
3. cos x dx =
3
cos x cos x dx =
2
(1 − sin x) cos x dx =
2
cos x dx − sin2 x(cos x dx)
1
= sin x − sin3 x + C
3

Z Z Z
4. sin3 4x dx = sin2 4x sin 4x dx = (1 − cos2 4x) sin 4x dx
Z Z
1 1 1
= sin 4x dx + cos2 4x(−4 sin 4x dx) = cos3 4x − cos 4x + C
4 12 4

Z Z Z Z
5. sin5 t dt = sin4 t sin t dt = (1 − cos2 t)2 sin t dt = (1 − 2 cos2 t + cos4 t) sin t dt
Z Z Z
= sin t dt + 2 cos2 t(− sin t dt) − cos4 t(− sin t dt)
2 1
= − cos t + cos3 t − cos5 t + C
3 5

Z Z Z Z
6. cos t dt =
5
cos t cos t dt = (1 − sin t) cos t dt = (1 − 2 sin2 t + sin4 t) cos t dt
4 2 2

Z Z Z
= cos t dt − 2 sin2 t(cos t dt) + sin4 t(cos t dt)
2 1
= sin t − sin3 t + sin5 t + C
3 5

Z Z Z
7. sin x cos x dx =
3 3
cos x sin x cos x dx = (1 − sin2 x) sin3 x cos x dx
2 3

Z Z
1 1
= sin3 x(cos x dx) − sin5 x(cos x dx) = sin4 x − sin6 x + C
4 6

Z Z Z
8. sin 2x cos 2x dx =
5 2
sin 2x cos 2x sin 2x dx =
4 2
(1 − cos2 2x)2 cos2 2x sin 2x dx
Z
= (1 − 2 cos2 2x + cos4 2x) cos2 2x sin 2x dx

Z Z Z 
2 Z
1 − cos 2t 1
9. sin t dt = (sin t) dt =
4 2 2
dt = (1 − 2 cos 2t + cos2 2t) dt
2 4
Z   Z  
1 1 + cos 4t 1 3 1
= 1 − 2 cos 2t + dt = − 2 cos 2t + cos 4t dt
4 2 4 2 2
3 1 1
= t − sin 2t + sin 4t + C
8 4 32
434 CHAPTER 7. TECHNIQUES OF INTEGRATION

Z Z Z  3
1 + cos 2θ
10. cos6 θ dθ = (cos2 θ)3 dθ = dθ
2
Z
1
= (1 + 3 cos 2θ + 3 cos2 2θ + cos3 2θ) dθ
8
Z    
1 1 + cos 4θ
= 1 + 3 cos 2θ + 3 + (1 − sin 2θ) cos 2θ dθ
2
8 2
Z  
1 5 3
= + 4 cos 2θ + cos 4θ − sin 2θ cos 2θ dθ
2
8 2 2
5 1 3 1
= θ + sin 2θ + sin 4θ − sin3 2θ + C
16 4 64 6
Z Z Z
11. sin x cos x dx = (1 − cos x) cos x dx = (cos4 x − cos6 x) dx
2 4 2 4

Z " 2  3 #
1 + cos 2x 1 + cos 2x
= − dx
2 2
Z
1
= [2(1 + 2 cos 2x + cos2 2x) − (1 + 3 cos 2x + 3 cos2 2x + cos3 2x)] dx
8
Z
1
= (1 + cos 2x + cos2 2x − cos3 2x) dx
8
Z  
1 1 + cos 4x
= 1 + cos 2x − − (1 − sin 2x) cos 2x dx
2
8 2
Z  
1 1 1
= − cos 4x + sin 2x cos 2x dx
2
8 2 2
1 1 1
= x− sin 4x + sin3 2x + C
16 64 48
Z Z Z Z Z
cos3 x cos2 x 1 − sin2 x
12. dx = cos x dx = cos x dx = (sin x)−2
(cos x dx) − cos x dx
sin2 x sin2 x sin2 x
1
=− − sin x + C = − csc x − sin x + C
sin x
Z Z Z  2
1 1 1 − cos 4x
13. sin x cos x dx =
4 4
sin 2x dx =
4
dx
16 16 2
Z Z  
1 1 1 + cos 8x
= (1 − 2 cos 4x + cos 4x) dx =
2
1 − 2 cos 4x + dx
64 64 2
Z  
1 3 1
= − 2 cos 4x + cos 8x dx
64 2 2
3 1 1
= x− sin 4x + sin 8x + C
128 128 1024
Z Z  2 Z Z
1 1 1 1 − cos 12x
14. sin2 3x cos2 3x dx = sin 6x dx = sin2 6x dx = dx
2 4 4 2
Z Z
1 1 1 1
= dx − cos 12x dx = x − sin 12x + C
8 8 8 96
7.4. POWERS OF TRIGONOMETRIC FUNCTIONS 435
Z Z Z
15. tan 2t sec 2t dt =
3 4
tan 2t sec 2t sec 2t dt = tan3 2t(1 + tan2 2t) sec2 2t dt
3 2 2

Z Z
1 1
= tan3 2t(2 sec2 2t dt) + tan5 2t(2 sec2 2t dt)
2 2
1 1
= tan4 2t + tan6 2t + C
8 12
Z √ Z Z Z
16. (2 − tan x) sec x dx = 4 sec x dx − 4(tan x) sec x dx + tan x sec2 x dx
2 2 2 1/2 2

8 1
= 4 tan x − (tan x)3/2 + tan2 x + C
3 2
Z Z Z
17. tan2 x sec3 x dx = (sec2 x − 1) sec3 x dx = sec5 x dx − sec3 x dx

u = sec3 x, du = 3 sec2 x sec x tan x dx; dv = sec2 x dx, v = tan x


Z Z
= tan x sec3 x − 3 tan2 x sec3 x dx − sec3 x dx
Z
1 1
From Example 5 of Section 7.3, sec3 x dx = sec x tan x + ln | sec x + tan x| + C, so
2 2
Z Z
1 1
tan2 x sec3 x dx = tan x sec3 x − 3 tan2 x sec3 x dx − sec x tan x − ln | sec x + tan x|.
2 2
Solving for the integral, we have
Z
1 1 1
tan2 x sec3 x dx = tan x sec3 x − sec x tan x − ln | sec x + tan x| + C.
4 8 8
Z Z
1
18. tan2 3x sec2 3x dx = tan2 3x sec2 3x(3 dx) u = tan 3x, du = sec2 3x(3 dx)
3
Z
1 1 1
= u2 du = u3 + C = tan3 3x + C
3 9 9
Z Z
19. tan x(sec x)
3 −1/2
dx = tan2 x(sec x)−1/2 tan x dx
Z
= (sec2 x − 1)(sec x)−3/2 sec x tan x dx
Z Z
= (sec x) (sec x tan x dx) − (sec x)−3/2 (sec x tan x dx)
1/2

2
=
(sec x)3/2 + 2(sec x)−1/2 + C
3
Z Z Z  
x x x x x x x x x x
20. tan3 sec3 dx = tan2 sec2 sec tan dx = sec2 − 1 sec2 sec tan dx
2 2 2 2 2 2 2 2 2 2
Z   Z  
x x x x x x
= sec4 sec tan dx − sec2 sec tan dx
2 2 2 2 2 2
2 5 x 2 3 x
= sec − sec +C
5 2 3 2
436 CHAPTER 7. TECHNIQUES OF INTEGRATION
Z Z Z
21. tan x sec x dx =
3 5
tan x sec x sec x tan x dx = (sec2 x − 1) sec4 x sec x tan x dx
2 4

Z Z
= sec6 x(sec x tan x dx) − sec4 x(sec x tan x dx)
1 1
= sec7 x − sec5 x + C
7 5
Z Z Z
22. tan x sec x dx =
5
(tan x) (sec x tan x dx) =
2 2
(sec2 x − 1)2 (sec x tan x dx)
Z
= (sec4 x − 2 sec2 x + 1)(sec x tan x dx)
Z Z Z
= sec x(sec x tan x dx) − 2 sec x(sec x tan x dx) + sec x tan x dx
4 2

1 2
= sec5 x − sec3 x + sec x + C
5 3
Z Z Z Z Z
23. sec5 x dx = sec3 x sec2 x dx = sec3 x(1 + tan2 x) dx = sec3 x dx + tan2 x sec3 x dx
Z
1 1
From Example 5 of Section 7.3, sec3 x dx = sec x tan x + ln | sec x + tan x| + C. From
Z 2 2
1 1 1
Exercise 17, tan2 x sec3 x dx = tan x sec3 x − sec x tan x − ln | sec x + tan x| + C1 .
4 8 8
Z  
1 1
Thus, sec5 x = sec x tan x + ln | sec x + tan x|
2 2
 
1 1 1
+ tan x sec3 x − sec x tan x − ln | sec x + tan x| + C2
4 8 8
3 3 1
= sec x tan x + ln | sec x + tan x| + tan x sec3 x + C2 .
8 8 4
Z Z Z
1
24. dx = sec2 x sec2 x dx = (1 + tan2 x) sec2 x dx
cos4 x
Z Z
1
= sec2 x dx + tan2 x sec2 x dx = tan x + tan3 x + C
3
Z Z Z
cos3 x (1 − sin2 x)
25. cos2 x cot x dx = dx = cos x dx
sin x sin x
Z Z
1
= (sin x)−1 (cos x dx) − sin x(cos x dx) = ln | sin x| − sin2 x + C
2
Z Z
1
(Alternatively, sin x cos x dx = cos x(sin x dx) = − cos2 x + C or
2
Z Z
1 1
sin x cos x dx = sin 2x dx = − cos 2x + C).
2 4
Z Z
1 1
26. sin x sec7 x dx = (cos x)−7 (sin x dx) = (cos x)−6 + C = sec6 x + C
6 6
7.4. POWERS OF TRIGONOMETRIC FUNCTIONS 437
Z Z Z
27. cot
10
x csc x dx =
4
cot x csc x csc x dx = cot10 x(1 + cot2 x) csc2 x dx
10 2 2

Z Z
= − cot10 x(− csc2 x) dx − cot12 x(− csc2 x) dx
1 1
= − cot11 x − cot13 x + C
11 13
Z Z Z
28. (1 + csc t) dt = (1 + 2 csc t + csc t csc t) dt = [1 + 2 csc2 t + (1 + cot2 t) csc2 t] dt
2 2 2 2 2

Z
1
= (1 + 3 csc2 t + cot2 t csc2 t) dt = t − 3 cot t − cot3 t + C
3
Z Z Z
sec (1 − t)
4
sec (1 − t)
2
1 + tan (1 − t)
2
29. dt = sec2 (1 − t) dt = sec2 (1 − t) dt
tan (1 − t)
8
tan (1 − t)
8
tan8 (1 − t)
Z Z
= [tan(1 − t)] sec (1 − t) dt + [tan(1 − t)]−6 sec2 (1 − t) dt
−8 2

1 1
= [tan(1 − t)]−7 + [tan(1 − t)]−5 + C
7 5
1 1
= + +C
7 tan7 (1 − t) 5 tan5 (1 − t)
Z √ √
sin3 t cos2 t √ 1
30. √ dt u= t, du = √
t 2 t
Z Z
=2 sin3 u cos2 u du = 2 sin2 u cos2 u sin u du
Z
=2 (1 − cos2 u) cos2 u sin u du
Z Z
= 2 cos2 u(sin u du) − 2 cos4 u(sin u du)
2 2 2 √ 2 √
= − cos3 u + cos5 u + C = − cos3 t + cos5 t + C
3 5 3 5
Z Z
31. (1 + tan x) sec x dx = (1 + 2 tan x + tan2 x) sec x dx
2

Z Z Z
= sec x dx + 2 tan x sec x dx + tan2 x sec x dx
The last integral is evaluated in Example 8 of Section 7.4. Thus,
Z
1 1
(1 + tan x)2 sec x dx = ln | sec x + tan x| + 2 sec x + sec x tan x − ln | sec x + tan x| + C
2 2
1 1
= ln | sec x + tan x| + 2 sec x + sec x tan x + C.
2 2
Z Z Z
32. (tan x + cot x)2 dx = (tan2 x + 2 + cot2 x) dx = (sec2 x − 1 + 2 + csc2 x − 1) dx
Z
= (sec2 x + csc2 x) dx = tan x − cot x + C
438 CHAPTER 7. TECHNIQUES OF INTEGRATION
Z Z Z
33. tan x dx =
4
tan x tan x dx =
2 2
tan2 x(sec2 x − 1) dx
Z Z
1
= tan2 x sec2 x dx − (sec2 x − 1) dx = tan3 x − tan x + x + C
3
Z Z Z
34. tan5 x dx = (tan2 x)2 tan x dx = (sec2 x − 1)2 tan x dx
Z
= (sec4 x − 2 sec2 x + 1) tan x dx
Z Z Z
= sec3 x(sec x tan x dx) − 2 sec x(sec x tan x dx) + tan x dx
1
sec4 x − sec2 x − ln | cos x| + C
=
4
Z Z Z Z Z
35. cot3 t dt = cot2 t cot t dt = (csc2 t − 1) cot t dt = csc t(csc t cot t dt) − cot t dt
1
= − csc2 t − ln | sin t| + C
2
Z
36. csc5 t dt u = csc3 t, du = −3 csc2 t cot t csc t dt; dv = csc2 t dt, v = − cot t
Z Z
= − cot t csc t − 3 cot t csc t dt = − cot t csc t − 3 (csc2 t − 1) csc3 t dt
3 2 3 3

Z Z
= − cot t csc3 t − 3 csc5 t dt + 3 csc3 t dt See Exercises 7.3, Problem 37
Z  
1 1
= − cot t csc t − 3 csc t dt + 3 − cot t csc t + ln | csc t − cot t|
3 5
2 2
Z
3 3
= − cot t csc3 t − cot t csc t ln | csc t − cot t| − 3 csc5 t dt
2 2
Solving for the integral, we have
Z
1 3 3
csc5 t dt = − cot t csc3 t − cot t csc t + ln | csc t − cot t| + C.
4 8 8

Z Z Z
37. (tan x − tan x) dx =
6 2
(tan x tan x − tan x) dx =
4 2 2
(tan4 x − 1) tan2 x dx
Z
= (tan4 x − 1)(sec2 x − 1) dx
Z
= (tan4 x sec2 x − tan4 x − sec2 x + 1) dx
Z Z Z Z
= tan4 x(sec2 x dx) − tan4 x dx − sec2 x dx + dx
Z
1
= tan5 x − tan4 x dx − tan x + x
5
7.4. POWERS OF TRIGONOMETRIC FUNCTIONS 439
Z
1
From Exercise 33, tan4 x = tan3 x − tan x + x + C, so
3
Z  
1 1
(tan6 x − tan2 x) dx = tan5 x − tan3 x − tan x + x − tan x + x + C1
5 3
1 1
= tan5 x − tan3 x + C1 .
5 3

Z Z Z
1
38. cot 2x csc5/2 2x dx = csc3/2 2x csc 2x cot 2x dx = − (csc3/2 2x)(−2 csc 2x cot 2x dx)
2
u = csc 2x, du = −2 csc 2x cot 2x dx
Z  
1 1 2 5/2 1
=− u du = −
3/2
u + C = − csc5/2 2x + C
2 2 5 5

Z Z Z
39. x sin x dx =
3 2
x sin x sin x dx = x(1 − cos2 x2 ) sin x2 dx
2 2 2

Z Z
= x sin x2 dx − x cos2 x2 sin x2 dx
Z Z
1 1
= sin x2 (2x dx) + cos2 x2 (−2x sin x2 dx)
2 2
t = x2 , dt = 2x dx; u = cos x2 , du = −2x sin x2 dx
Z Z   
1 1 1
= sin t dt + u2 du = − cos t + u3 + C
2 2 3
 
1 1 1 1
= − cos x2 + cos3 x2 + C = cos3 x2 − cos x2 + C
2 3 6 2

Z Z
1
40. x tan8 x2 sec2 x2 dx = tan8 x2 (2x sec2 x2 dx) u = tan x2 , du = 2x sec2 x2 dx
2
Z  
1 1 1 9 1
= u du =
8
u +C = tan9 x2 + C
2 2 9 18

Z π/2 √ Z π/2 Z π/2


41. sin3 θ cos θ dθ = sin2 θ(cos θ)1/2 sin θ dθ = (1 − cos2 θ)(cos θ)1/2 sin θ dθ
π/3 π/3 π/3
Z π/2 Z π/2
=− (cos θ)
1/2
(− sin θ dθ) + (cos θ)5/2 (− sin θ dθ)
π/3 π/3
π/2 π/2
2 2
= − (cos θ) 3/2
+ (cos θ) 7/2
3 π/3 7 π/3
"  3/2 # "  7/2 # √ √ √
2 1 2 1 2 2 25 2
=− 0− + 0− = − =
3 2 7 2 6 56 168
440 CHAPTER 7. TECHNIQUES OF INTEGRATION

Z π/2 Z π/2 Z π/2


42. sin5 x cos5 x dx = sin5 x cos4 x cos x dx = sin5 x(1 − sin2 x)2 cos x dx
0 0 0
Z π/2
= sin5 x(1 − 2 sin2 x + sin4 x) cos x dx
0
Z π/2 Z π/2 Z π/2
= sin5 x(cos x dx) − 2 sin7 x(cos x dx) + sin9 x(cos x dx)
0 0 0
π/2 π/2 π/2
1 1 1 1 1 1 1
= sin6 x − sin x 8
+ sin x
10
= − + =
6 0 4 0 10 0 6 4 10 60
Z π Z π Z π
43. sin3 2t dt = sin2 2t sin 2t dt = (1 − cos2 2t) sin 2t dt
0 0 0
Z π Z π π
1 π 1 1
= sin 2t dt + cos2 2t(−2 sin 2t dt) = − cos 2t + cos3 2t
0 2 0 2 0 6 0
1 1
= − (1 − 1) + (1 − 1) = 0
2 6
Z π Z π Z π
44. sin4 x cos2 x dx = sin4 x(1 − sin2 x) dx = (sin4 x − sin6 x) dx
−π −π −π
Z π " 2  3 #
1 − cos 2x 1 − cos 2x
= − dx
−π 2 2
Z
1 π
= [2(1 − 2 cos 2x + cos2 2x) − (1 − 3 cos 2x + 3 cos2 2x − cos3 2x)] dx
8 −π
Z
1 π
= (1 − cos 2x − cos2 2x + cos3 2x) dx
8 −π
Z  
1 π 1 + cos 4x
= 1 − cos 2x − + (1 − sin 2x) cos 2x dx
2
8 −π 2
Z π  
1 1 1
= − cos 4x − sin 2x cos 2x dx
2
8 −π 2 2
 π
1 1 1 1 1 π
= x − sin 4x − sin 2x 3
= (π + π) =
8 2 8 6 −π 16 8
Z π/4 Z π/4 Z π/4
45. tan y sec4 y dy = tan y sec2 y sec2 y dy = tan y(1 + tan2 y) sec2 y dy
0 0 0
Z π/4 Z π/4
= tan y sec2 y dy + tan3 y sec2 y dy
0 0
π/4 π/4
1 1 1 1 3
tan2 y
= + tan4 y = (1 − 0) + (1 − 0) =
2 0 4 0 2 4 4
Z π/3 Z π/3 π/3
2
46. tan x sec3/2 x dx = sec1/2 x(sec x tan x dx) = sec3/2 x
0 0 3 0

2 √ 4 2−2
= (2 2 − 1) =
3 3
7.4. POWERS OF TRIGONOMETRIC FUNCTIONS 441
Z Z Z
1 1
47. sin x cos 2x dx = [sin 3x + sin(−x)] dx = (sin 3x − sin x) dx
2 2
1 1
= − cos 3x + cos x + C
6 2
Z Z
1 1 1
48. cos 3x cos 5x dx = (cos 2x + cos 8x) dx = sin 2x + sin 8x + C
2 4 16
Z Z
1 1 1
49. sin 2x sin 4x dx = (cos 2x − cos 6x) dx = sin 2x − sin 6x + C
2 4 12
Z Z Z
5 − 3 sin 2x 5 3
50. dx = (5 cos 6x − 3 sin 2x cos 6x) dx = sin 6x − [sin 8x + sin(−4x)] dx
sec 6x 6 2
Z
5 3 5 3 3
= sin 6x − (sin 8x − sin 4x) dx = sin 6x + cos 8x − cos 4x + C
6 2 6 16 8
Z π/6 Z  π/6
1 π/6 1 1
51. cos 2x cos x dx = (cos x + cos 3x) dx = sin x + sin 3x
0 2 0 2 3 0
 
1 1 1 5
= + =
2 2 3 12
Z Z  π/2
π/2
3 1 1 π/2 1 1
52. sin x sin x dx = (cos x − cos 2x) dx = sin x − sin 2x
0 2 2 2 0 2 2 0
1 1
= (1 − 0) =
2 2
53. If m 6= n, then using the fact that sin mx sin nx is an even function we have
Z π Z π
sin mx sin nx dx = [cos(m − n)x − cos(m + n)x] dx
−π 0
π π
1 1
= sin(m − n)x − sin(m + n)x = 0.
m−n 0 m+n 0

If m = n, then
Z π Z π Z π Z π
sin mx sin nx dx = sin2 mx dx = 2 sin2 mx dx = (1 + cos 2mx) dx
−π −π 0 0
 π
1
= x+ sin 2mx = π.
2m 0

Z (
π
0, m 6= n
Thus, sin mx sin nx dx = .
−π π, m = n
Z π
54. Since sin mx cos nx is an odd function, sin mx sin nx dx = 0.
−π
442 CHAPTER 7. TECHNIQUES OF INTEGRATION

x
55. y = sec2 2
2
Z π/2 Z π/2 y
x x x
V =π sec dx = π
4
sec2 sec2 dx
−π/2 2 −π/2 2 2 –π/2 π/2

Z π/2  
x x
=π 1 + tan2 sec2 dx
−π/2 2 2
"Z   #
π/2 
x  1 Z π/2 
x  1 x
= 2π sec2 dx + tan2 sec2 dx
−π/2 2 2 −π/2 2 2 2
π/2 !  
x iπ/2 1 x 1 1
= 2π tan + tan3 = 2π 1 − (−1) + − (−1)
2 −π/2 3 2 −π/2 3 3
 
2 16π
= 2π 2 + =
3 3

1
1–y
56. y = sin2 x
Z π/2 Z π/2 –π/2 π/2

V =π (1 − sin2 x)2 dx = π (cos2 x)2 dx


−π/2 −π/2
Z  2Z
1 + cos 2x
π/2
π π/2
=π dx = (1 + 2 cos 2x + cos2 2x) dx
−π/2 2 4 −π/2
Z   Z  
π π/2 1 + cos 4x π π/2 3 1
= 1 + 2 cos 2x + dx = + 2 cos 2x + cos 4x dx
4 −π/2 2 4 −π/2 2 2
 π/2    
π 3 1 π 3π 3π 3π 2
= x + sin 2x + sin 4x = +0+0 − − +0+0 =
4 2 8 −π/2 4 4 4 8

Z π/4
57. A = (cos3 x − sin3 x) dx -!

−3π/4
Z π/4 Z π/4
-1

= (cos2 x cos x − sin2 x sin x) dx = [(1 − sin2 x) cos x − (1 − cos2 x) sin x] dx


−3π/4 −3π/4
Z π/4
= [cos x − (sin2 x) cos x − sin x + (cos2 x) sin x] dx
−3π/4
 π/4
1 1
= sin x − sin x + cos x − cos x
3 3
3 3 −3π/4
" √ √ √ √ ! √ √ √ √ !# √ √
2 2 2 2 2 2 2 2 √ 2 5 2
= − + − − − + − + =2 2− =
2 12 2 12 2 12 2 12 3 3
7.5. TRIGONOMETRIC SUBSTITUTIONS 443

Z Z Z
1 2π
1 2π
θ 1 2π
58. A = r2 dθ = sin2 4θ sin2 dθ = (1 − cos 8θ)(1 − cos θ) dθ
2 0 2 0 2 8 0
Z
1 2π
= (1 − cos θ − cos 8θ + cos 8θ cos θ) dθ
8 0
 2π Z
1 1 1
= θ − sin θ − sin 8θ + (cos 7θ + cos 9θ) dθ
8 8 0 16
 2π
1 1 1 1 π
= (2π) + sin 7θ + sin 9θ =
8 16 7 9 0 4
Z π Z π
2
59. Based on the graphs, the values of cos x dx,
3
cos5 x dx,
Z π 0 0
1
and cos7 x dx all appear to be 0. We note that, for every t
0
π π  π  !
such that 0 ≤ t ≤ , cos − t = − cos + t , thus lending cos3 x
2 2 2
cos5 x
-1
credence to this conjecture.
-2 cos7 x
Z π
60. Based on Problem 59, we conjecture that the value of cosn x dx, where n is a positive odd
Z π 0

integer, is 0. To prove this, we evaluate cosn x dx:


0
Z π Z π
cos x dx =
n
cosn−1 x cos x dx
0 0

n−1
Since n is a positive odd integer, n − 1 is guaranteed to be even, and thus k = is an
2
integer. Further, we substitute cos2 x = 1 − sin2 x:
Z π Z π Z π
cosn x dx = (cos2 x)k cos x dx = (1 − sin2 x)k cos x dx.
0 0 0

By the binomial theorem, (1 − sin x) expands into an expression of the form 1 + c1 sin2 x +
2 k

c2 (sin2 x)2 + c3 (sin2 x)3 + · · · + ck (sin2 x)k . For this particular proof, it is not necessary to
specify the precise values of the binary coefficients ci . Using the u substitution u = sin x,
du = cos x dx, integration is accomplished as follows:
Z π Z π
cos x dx =
n
(1 + c1 sin2 x + c2 sin4 x + · · · + ck sin2k x)(cos x dx)
0 0
 π
c1 c2 ck
= sin x + sin x +
3
sin x + · · · +
5
sin2k+1
x
3 5 2k + 1 0
Z π
Evaluating this, we note that sin π = sin 0 = 0, and therefore cosn x dx = 0.
0

7.5 Trigonometric Substitutions


444 CHAPTER 7. TECHNIQUES OF INTEGRATION
7.5. TRIGONOMETRIC SUBSTITUTIONS
7.5. TRIGONOMETRIC SUBSTITUTIO
7.5 1
Trigonometric Substitutions
Z √ x
1 − x2 1 − x2 7.5
! √
Trigonometric Substi
1. dx x = sin θ, dx = cos θ dθ 1. dx x = sin θ, dx = cos θ dθ
x2 x2 ! √
Z p
" 1 − x2
! 7.5. TRIGONOMETRIC ! SUBSTITUTIO !
Z Z 1 − sin2 θ xcos
2
= 1. = sin
θ θ, dx =2
1 − sin2 θ cos2 θ dx
2 x2 cos θ dθ = dθ = cot
= cos θ dθ = dθ = cot dθ
2 sin sin 2
θ
! √ θ
! ! " √
sin2 θ sin2 θ = (csc1.2
1 − x2
θ − 1) dθx= dx θ − θ x+1=
2 − cot
−sin= sin 2 1−x
θ, θdx =
Z √ = C − cos
x
1 − x TODO −1 sin
2 " 2
! θ
= (csc2 θ − 1) dθ = − cot θ − θ + C = − − sin figure
x+C !
=
1 − sin2 θ
cos
x !
x 3 = (csc 2
θsin−2 θ1) dθ =
2. √ dx x = 2 sec θ, dx = 2 sec θ tan ! θ dθ
x2 − 4 = (csc2 θ!− 1) dθ =
7.5. TRIGONOMETRIC SUBSTITUTIONS ! TODO figure
8 sec3 θ
= √! ! (2 sec θ tan θ) dθ = 8 sec4 θ dθ
4 sec2 θx− 3 4x3
Z 7.5 Trigonometric 2.! x
Substitutions
2. √ √ dx dx x#= x =2 sec2 secθ,θ, dxdx $ ==
x3 1
4 2 4θ dθ = 8! tan θ + 3 tan3 θ
2−
= 8 (1 + tan x22 θ)−xsec
2. √ dx x = 2 sec θ, dx = 2 sec θ tan θ dθ ! √
1−x 2 ! 8 sec 3
3 θ
x2 − 4 1. dx x = sin
" θ, dx = cos 1 θ dθ 3/2 = √ √8 sec 2 θ (2
Z Z x2 = 4 x2 − 4 + (x2 − 4)= + C 4 sec θ − 4(2
22
! " 3 ! ! 4 sec! θ−4
2
8 sec θ
3 7.5. TRIGONOMETRIC 1 − sin θ SUBSTITUTIONS cos θ!
2

= √ (2 sec θ tan θ) dθ = 8 sec4 θ dθ TODO figure =


sin2 θ
cos θ dθ =
=sin8
= 8dθ =
2
(1 + cottan22dθ
(1 +√tan2 θ) sec
θSUBSTITUTIONS
θ) sec
4 sec2 θ − 4 3.
! √ ! 2 7.5. TRIGONOMETRIC
1−x " 1 21
7.5

Z   1. dx x = sin θ, dx
=x2 (csc θ − 1) dθ = − cot θ − θ + " = cos θ =
dθ 4
C=−
− x
2 − 4 + (x27.−
13− sin
2 x
1 ! √ 2 − 4 x+ !(x2 −
= 8 (1 + tan2 θ) sec2 θ dθ = 8 tan θ + tan3 TODO θ 4.+figure
C ! " 1 −2x2
1 − sin θ dx
= 4 ! xcos
dθ =3 cos
2
1. 3. = sin θ dx =
3 ! 5. 3
= 2
sin2xθfigure
cos θ dθ = x θ, cotθ2
" sin θ
2
p TODO ! √
1 2. √
x
dx
!
x = 2 sec θ,4. dx = 2 sec θ tan θ dθ 1 − sin θ
2
1−x
= 4 x2 − 4 + (x2 − 4)3/2 + C 6.x2 − 4 = (csc2 θ − 1) dθ = =
3. 3 5.
− cot θ − θ +2 C = −cos θ dθ
sin x
3 7.
!
!
8 sec θ !
! θ
7.5. TR
=x3 √ (2 sec θ tan θ) dθ = 82 sec4 θ dθ
2. √ 4 sec
dx 4. 2 θx6.−=42 sec θ, dx ==2 sec(csc θ tan θ dθ − 1) dθ = − co
8.
x2 −!4 # $ 7.5
! ! 2 8 sec233θ 1 ! !

Z = 8 (1 + = tan√ 7.θ)√secx θ dθ (2= sec


8 θtantanθθ)+dθ3 = tan8 θ sec+41.θCdθ
3
 p  5.2. = 2 sec = 2 se
1 x 9.
" 4 sec θ − 4
2
! 1 2x2 − 43/2
dx x
#
θ, dx
$
3. √ dx = cosh−1 + C = ln x + x2 − 362 + C, x10.> 6 = 4 x2 − 6. 4 + 8. (x − 4) + 2 C! 1
6 = 8 (1 3 + tan θ) sec θ dθ = tan θθ+ tan θ
88 sec
2 3 3
x2 − 36 = √ 3(2 sec θ
TODO figure "9. 1 4 sec2 θ − 4
11. 7.
= 4 x2 − 4 + (x2 − 4)3/2 ! +C TO
3 !
Alternatively, the substitution x = 6 sec θ could have been used.12.
3. = 8 (1 + tan sec
2 2
! 10. θ) 2.θ d
1 % " &
3. √ 8.
dx = cosh −1 x
+ C = ln x " + x − 36 1 + C, x
2 2
4. x2 − 36 11. 6
13. TRIGONOMETRIC
7.5. SUBSTITUTIONS = 4 x2 − 4 + (x2 − 4)3
5. Alternatively, 9.
the substitution x = 6 sec θ could have 3 been used.
14. ! " 12.
! %
√ 1 √ −1 x
Z p 6. 7.5 4.Trigonometric
3 − x2 dx10.3. x =√ Substitutions
3 sin θ, dx dx== cosh3 cos θ dθ6 + C = ln TO x
√ √ 15. 13. x2 − 36 x
4. 3 − x2 dx x = 3 sin θ, dx = 3 cos θ dθ 7.
! √
1 − x 2
! "
Alternatively, √ the substitution
!
x = 6 sec
3. θ
16.
1. dx 11.=x = 14. 3 −θ,3 sin 2
3 cos
θ cos θ dθ = 3 cos2 θ dθ = 31
Z p Z x2 !sin "
dx = θ dθ

4.
√1
# $
√ 8.
17. ! 4.3"15. 13 2− x2 dx !3 = 323 sin θ, dx
x ! 5.=
= 3 − 3 sin2 θ 3 cos θ dθ = 3 cos2 θ dθ 9. 12.
= 1θ−+sinsin θ 2θ + C = θ cos + θsin θ cos θ + C21
= 2 2 cos θ dθ = ! " 2 22 dθ = cot
6.
1
sin θ
16.
2
# $ √ sin 2 θ 2 √ 7.
Z   13.
! 3 x 3 =
x 3 3
− −x 3 sin θ √33 cos 1θ
= sin −1
√ + √ √ + C = 1sin

−1
1 + cos 2θ 3 1 10. 3 3 = (csc 2 17.2
θ − 1) 3 dθ2= − cot 3 θ#− θ3 + C = −$2 8. 1
x
=3 dθ = θ + sin 2θ + C 11. = θ +5. sin θ cos14. θ+C =
3 1
θ + sin 2θ + C =
x 1
9.
2 2 2 2 TODO 2 figure 18. 2 2 10.
 √ !6. # $1
3 −1 x 3 x 3 − x2 3 12. −12. x √ x3x pdx 15.x 2=19.2 sec θ, dx = 2=sec23 θsin −1 x
tan θ √ dθ3 + 2 √
3 11. x
= sin √ + √ √ + C = sin √7. +x2 − 4 3 − x + C 12.
3
2 3 2 3 3 2 13. 3 2 16.
!
5. 20.
8 sec 3
θ
! 13.

8. = √ (2 sec θ tan θ) dθ = 8 sec 14.θ dθ 4


14.
9.
17.!6. 4 sec2 θ − 4 # 15.
$
15. 1
= 8 (1 + tan2 θ) sec2 θ dθ = 8 tan θ + tan16.3 θ
Z p Z 10. 7. 3 17.
1 1 16. " 1
5. x x2 + 7 dx = (x2 + 7)1/2 (2x dx) = (x2 + 7)3/2 + C 11. = 4 8.x2 − 4 + (x2 − 4)3/2 + C
2 3 17. 3
12.
TODO figure 9.
7.5.3. TRIGONOMETRIC
13. SUBSTITUTIONS
10.
7.5. TRIGONOMETRIC SUBSTITUTIO
Z 4. 14. Trigonometric
7.5 11. 1
Substitutions
x
6. (1 − x2 )3/2 dx x = sin θ, dx = cos θ dθ 5. ! √1 − x2 7.512. Trigonometric Substi
1. dx x = sin θ, dx = cos θ dθ
6. x2 ! √
!13." 1 − x2 ! !
1. 1 − sin2 θ dx xcos
2
= sin
θ θ, dx =2
7. = 14. 2 x2 cos θ dθ = dθ = cot
sin θ sin θ
2

8.
! ! " √
2 1−x
= (csc2 θ − 1) dθ = − cot
= θ − θ +1 C−=sin
− θ cos
x
9.
2
sin θ
TODO figure !
10. ! = (csc2 θ − 1) dθ =
x3
2. √ dx x = 2 sec θ, dx = 2 sec θ tan θ dθ
11. x2 − 4
! TODO figure !
8 sec3 θ
12. = √! (2 sec θ tan θ) dθ = 8 sec4 θ dθ
4 sec2 θx− 34
13. 2.! √ 2 dx x#= 2 sec1θ, dx $ =
= 8 (1 + tan x2 θ) 4 2 θ dθ = 8 tan θ + tan3 θ
− sec
! 3
14. " 8 sec θ
3
1
= 4 x2 − 4 + (x2 − 4)= 3/2
+ C√ (2
15. 3 4 sec2 θ − 4
TODO figure !
7.5. TRIGONOMETRIC SUBSTITUTIONS 445

Z Z Z  2
1 + cos 2θ
=
(1 − sin2 θ)3/2 cos θ dθ = cos4 θ dθ = dθ
2
Z Z  
1 1 1 + cos 4θ
= (1 + 2 cos 2θ + cos2 2θ) dθ = 1 + 2 cos 2θ + dθ
4 4 2
Z  
1 3 1 3 1 1
= + 2 cos 2θ + cos 4θ dθ = θ + sin 2θ + sin 4θ + C
4 2 2 8 4 32
3 1 1 3 1
= θ + sin 2θ + sin 2θ cos 2θ + C = θ + sin 2θ(4 + cos 2θ) + C
8 4 16 8 16
3 1
= θ + sin θ cos θ(4 + 1 − 2 sin2 θ) + C
8 8
3 1
= θ + sin θ cos θ(5 − 2 sin2 θ) + C
8 8
3 1 p
= sin x + x 1 − x2 (5 − 2x2 ) + C
−1
8 8
Z p Z p
1
7. x3 1 − x2 dx = − x2 1 − x2 (−2x dx) u = 1 − x2 , x2 = 1 − u, 2x dx = −du
2
Z Z
1 1
=− (1 − u)u du = −
1/2
(u1/2 − u3/2 ) du
2 2
 
1 2 3/2 2 5/2 1 1
=− u − u + C = − (1 − x2 )3/2 + (1 − x2 )5/2 + C
2 3 5 3 5
Z p Z p
1
8. x3 x2 − 1 dx = x2 x2 − 1(2x dx) u = x2 − 1, x2 = u + 1, 2x dx =7.5. du TRIGONOMETRIC SUBSTITUTIONS
2
Z Z  ! √
1 1 1 2 5/2 2 3/21. 1 − x2
x = sin θ, dx = cos θ dθ
= (u + 1)u du =
1/2
(u + u ) du =
3/2 1/2
u + u + Cx2 dx
2 2 2 5 3 ! " !
1 − sin2 θ co
1 2 1 2 =
sin2 θ
cos θ dθ =
si
= (x − 1) + (x − 1) + C
5/2 3/2
!
Z 5 3 = (csc θ − 1) dθ = − cot θ − θ
2
7.5. TRIGONOMETRIC SUBSTITUTIONS
1
9. dx x = 2 sec θ, dx = 2 sec θ tan θ dθ 7.5. TRIGONOMETRIC SUBSTITUTIONS x ! 3
(x2 − 4)3/2 7.5 Trigonometric Substitutions x 2. √ dx x = 2 sec θ, dx = 2 sec θ tan θ
Z ! x2 − 4 ! " #
2 sec θ tan θ ! √ 81 81 ! 1 + cos 4θ
1 − x2 = (1 + 2 cos 2θ + cos 2θ) dθ =
2
18+sec
2 cos
3
θ 2θ + d
= dθ 1. dx 4 x = sin θ, dx = cos θ dθ =4 √ (2 sec θ2tan θ) d
(4 sec2 θ − 4)3/2 x2
81
! "
" 3 22 2θ +
1
#
81
"4 sec2 θ − 4
! !3 1
#
#
Z Z = ! 1− + sin
2 cos θ
!
cos 4θ dθ2 =
cos θ θ + sin 2θ + sin 4θ +
1 sec θ 1 =4 2 2 cos θ 2dθ = = 8dθ4 =(12+ cot θ) sec28θ dθ = 8
tan22dθ
= dθ = (sin θ) cos θ dθ
−2 sin sin 2
243 81 θ 81 θ
" √1 − x21
7.5. TRIGONOM
4 tan2 θ 4 = !θ + sin 2θ + sin 4θ + C
=8 (csc42 θ − 1) dθ = 32− cot θ − θ +=C4= − x2 − 4 + − (xsin
− 4)x!+ C +
2 −1 3/2√
C
x 3 1−x
1 1 1 243 81 81
2

x = θ+ sin θ cos θ + sin 2θ cos 2θ + C


1. 2

= − (sin θ) + C = − csc θ + C = − √
−1
+C
x
TODO figure 8 2 3. 16
4 4 4 x!2 − 43 = 243 θ + 81 sin θ cos θ + 81 sin θ cos θ(1 − 2 sin2 θ) + C
Z 2. 7.5.√
x
TRIGONOMETRIC
dx 8 x = 2 sec θ,4. dx =
2 SUBSTITUTIONS 8 θ tan θ dθ #
" 2 sec
x2 − 4 243! 1 1 1
10. (9 − x2 )3/2 dx x = 3 sin θ, dx = 3 cos θ dθ = θ + 81 sin3θθ5.
8 sec cos θ + − sin2 θ ! + C !
x 3

7.5 Trigonometric=8 √ 2 θ tan


(23"sec 8 θ)4dθ = 8 sec4 θ dθ
Substitutions 2. √
x x −4 2
Z 243 4 sec 2 θ −
6. 4 5 1
#
! √ = 2 θ! + 81 sin θ cos θ − sin# 2
θ +C1 $
1 − x8
= (9 − 9 sin2 θ)3/2 (3 cos θ dθ) 1. = 8dx (1 + xtan =27. θ) sec
sin 8θ dθ
θ, 2dx ="4=
cos8θ dθtan θ # + tan3 θ + C
3
x2 243 x $ 5 1
Z Z = sin−1! "
" + 9x 1− sin9 2− x23/2 − !x2 +2 C !
=8 4 x=2 −34 +18. (x2 −θ4)cos θ
8+ C 36 cos θ
dθ = dθ = cot2 dθ !
= −81 (1 − sin θ) (− cos θ dθ) = 81 cosTODO 13 $
2 3/2 4
θ dθfigure= 243 sin−1 x + sin 2
θ
x 9 − x2 (45 − 2x2 ) + C
sin θ
2
1
3 89.
! √ 3. √
8 1 − x2 x2 − 36
! $
Z Z  2 3. x2 + 4 dx
= (csc2 θ − 1) dθ = − cot θ − θ + C 7.5. =− TRIGONOMETRIC sin −1
− Alternativel
SUBSTITx+
1 + cos 2θ 11. 10.
x = 2 tan θ, dx = 2 sec θ dθ 2 x ! "
43 +
= 81 (cos2 θ)2 dθ = 81 dθ4. TODO !figure !
7.5 Trigonometric4. −Su
x2

2 ! = 3
$ 11.
4 tan2 θ + 4 2 sec2 θ dθ = 4 sec3 θ dθ 1.
! √
1 − x2
dx x = sin θ, d
x x2
5. 2. √ dx x =12.2 sec θ, dx = 2 sec θ tan θ dθ ! " 2
1 − sin2
4 Section 7.3, Example 5
x2 −See =
! ! sin2 θ
6. 8 sec θ
13.
3 !
= 2 sec θ tan=θ + √2 ln | sec θ + tan(2 sec
θ| + C θ) dθ = 8 sec θ dθ = (csc2 θ − 1)
θ tan 4

7. √ 4 sec2 θ' √−4 ' #


% !& 14. ' ' TODO figure $ 5.
x2 + 4 x ' 2 x2 + 24 x ' 1!
=2 = 8 (1 ++ 2 ln ' θ) sec θ+dθ =
tan ' +8 C tan θ +2. tan x33 θ + 6. C
8. 2 2 15. ' 2 2' 3 x2 − 4 dx
√ x = 2 sec θ,
' ' 7. !
x $ " ' $ 1 ' 8 sec3 θ
9. = 2 2 ' 4x+ + (x
x +=4 4+ 2xln16.
− 2 4 +−
2
x'4)+ C1+ C
3/2 =
8.

4 sec2 θ −
2 3 !
!10. ! 9. 8 (1 + tan2
=
x TODO1figure 1 1
12. dx = (2x 17.
dx) = ln(25 + x2 ) + C 10. " 2
=4 x −4+
1
11.25 +3.x 2 25 + x 2
2 2
3
! 18. TODO figure 11.
1 −1 x 12.
13. 12.√ 4. dx = sin +C 3.
25 − x2 5 19. 13.
4.
!13. 5.
1 1 |x| 20. 5. 14.
14. √ dx = sec−1 +C
14.x x6.2 − 25 5 5 6.
! 7.5. TRIGONOMETRIC SUBSTIT
446
446 CHAPTER 7. TECHNIQUES OF INTEGRATION
Z Z  
81 81 1 + cos 4θ
= (1 + 2 cos 2θ + cos 2θ) dθ =
2
1 + 2 cos 2θ + dθ
4 4 2
Z    
81 3 1 81 3 1
= + 2 cos 2θ + cos 4θ dθ = θ + sin 2θ + sin 4θ + C
4 2 2 4 2 8
243 81 81
= θ+ sin 2θ + sin 4θ + C
8 4 32 7.5. TRIGONOMETRIC SUBSTITUTIONS

243 81 81 81
!
= θ+ sin θ cos θ + sin 2θ cos 2θ + C = (1 + 2 cos 2θ + cos2 2
8 2 16 4
! "
243 81 81 =
81 3
+ 2 cos 2θ + c
1
= θ+ sin θ cos θ + sin θ cos θ(1 − 2 sin2 θ) + C 4 2 2
8 2  8  243 81 81
= θ+ sin 2θ + sin
243 1 1 1 8 4 32
= θ + 81 sin θ cos θ + − sin θ + C 2
243 81 81
8 2 8 4 =
8
θ+
2
sin θ cos θ +
16
  243 81 81
243 5 1 = θ + sin θ cos θ +
= θ + 81 sin θ cos θ − sin2 θ + C 8 2 " 8
8 8 4 =
243
θ + 81 sin θ cos θ
1
+
p   8 2
243 x 5 1 "
= sin−1 + 9x 9 − x2 − x2 + C =
243
θ + 81 sin θ cos θ
5

8 3 8 36 8 8
$
243 x 1 p =
243 x
sin−1 + 9x 9 − x2
= sin−1 + x 9 − x2 (45 − 2x2 ) + C 8 3
8 3 8 243 −1 x 1 $
Z p 7.5. TRIGONOMETRIC SUBSTITUTIONS = sin + x 9−x
8 3 8
! $
11. x2 + 4 dx x = 2 tan θ, dx = 2 sec2 θ dθ 11. 4 +Substitutions x = 2 tan θ, dx = 2 sec2 θ
7.5 Trigonometric 2
x dx
x
Z p Z ! √
! $
1 − x2 = 4 tan2 θ + 4 2 sec2 θ dθ
= 4 tan2 θ + 4 2 sec2 θ dθ = 4 sec3 θ dθ 1.
x2
dx x = sin θ, dx = cos θ dθ
! " 2 See Section
! 7.3, Example
!
1 − sin2 θ cos2 θ
See Section 7.3, Example 5 =
sin2 θ
cos θ dθ =
= 2 sec θ tansin θ+2 2 dθ = cot2
θ ln | sec θ + ta
! √ √' √
x2 θ+−4θ%+x C & '1 − 2x
= 2 sec θ tan θ + 2 ln | sec θ + tan θ| + C = (csc2 θ − 1) dθ = = 2− cot
2 2
=2−ln '' x
+
' x 2
√ √ TODO figure
x2 + 4  x  x2 + 4 x x$ 2 '$
'
= x + 4 + 2 ln ' x2 + 4 +
=2 + 2 ln + +C !
x3 2
2 2 2 2 2. √
x2 − 4
dx !x = 2 sec θ, dx = 2!sec θ tan θ dθ
x 1 1 1
p 12. ! dx = ! = ln(
(2x dx)
xp 2 = !√
258 +
secx32θ 2 25 + x2
(2 sec θ tan θ) dθ = 8 sec4 θ dθ
2
= x + 4 + 2 ln x2 + 4 + x + C1 4 sec12 θ − 4 x
2 13. ! √ dx = sin−1 + #C $
Z Z = 8 (1 25 − x22θ) sec2 θ dθ =58 tan θ + 1 tan3 θ
+ tan
3
x 1 1 1 !
1 1 |x|
12. dx = (2x dx) = ln(25 + x2 ) + C 14. " 2√
= 4 xx −x42 +
1 dx
2 = 3/2 sec−1 +C
25 + x2 2 25 + x2 2 − 325(x − 4)5 + C 5
!
Z TODO figure 15. 1
√ dx x = 4 sin θ, dx = 4 cos
1 x x 16 − x2
13. √ dx = sin−1 + C 3. !
4 cos θ
25 − x2 5 = $
4. 4 sin θ 16 − 16 sin2 θ
Z
1 1 |x| 5.
1
= ln | csc θ − cot θ| + C =
14. √ dx = sec−1 +C 4
x x2 − 25 5 5 6.
Z 7.5. TRIGONOMETRIC SUBSTITUTIONS
446
1 7.
15. √ dx x = 4 sin θ, dx = 4 cos θ dθ
x 16 − x2 7.58. Trigonometric Substitutions4
Z Z TODO figure x
4 cos θ 1 9.
! √
1−x 2 !
= p dθ = csc θ dθ 1. dx x = sin θ,1 dx = cos θ dθ
16. " √ x = 4 sin θ, dx =
4 sin θ 16 − 16 sin2 θ 4 10.
x2
! x2 162− x2
dx
! !
=
1 − sin θ
cos θ dθ =
! cos2 θ
dθ =4 cos
cotθ2
1 1 4 16 − x2 11. sin2 θ = sin2
θ "
= ln | csc θ − cot θ| + C = ln − + C ! √
4 4 x x 12. 2
=
16 sin2 θ 16 1 −−x
(csc θ − 1) dθ = − cot θ − θ + C = −
1 x
13. =− cot θ + C
TODO figure 16 #
√ $
14. ! 1 16 − x2
x3
2. √ dx =θ−tan θ dθ
x = 2 sec θ, dx = 2 sec
15. x2 − 4
!
16 !
x
8 sec3 θ
= TODO (2 sec tan = 8 sec4 θ dθ
2figure
16. √ θ θ) dθ
!! 4 sec θ − 4 # $
17. 1 1
17.
= 8 (1 √ dx2 θ dθ =x8= tan
+ tan2 θ) sec tanθ θ,
+ dx
3
tan=3
θs
"
x 1 + x2
1 !
= 4 x2 − 4 + (x2 − 4)3/2 + C sec2 θ
3 = √
TODO figure tan θ 1 + tan2 θ
3.
= ln | csc θ − cot θ| + C
4.
TODO figure
5.
446 CHAPTER

7.5. TRIGONOMETRIC SUBSTITUTIONS TODO figure 447


!
1
Z 16. √ dx
7.5. TRIGONOMETRIC 2SUBSTITUTIONS x = 4 sin θ, dx = 4 cos θ dθ
1 446 x 16 − x2 CHAPTE
!
16. √ dx x = 4 sin θ, dx = 4 cos θ dθ 4
4 cos θ
x 16 − x2
2 7.5 Trigonometric Substitutions = " =
Z Z TODO figure x 16 sin2 θ 16 − 16 sin2 θ 1
! √
4 cos θ 1 1 − x2 !
1 1
= p = csc θ dθ
2 1. dx
16. " √
x = sin θ, dx = cos θ dθ
= − xcot = θ4+ sinCθ, dx = 4 cos θ dθ
16 sin2 θ 16 − 16 sin2 θ 16 x2
446 ! x2 162− x2
dx 16
! #√ ! $ CHAPTER
! 446 1 − sin θ ! cos 2
θ 2 cot2 dθ CHAPT
√ = cos θ dθ = 1 16dθ− =4xcos θ
1 1 16 − x2 sin θ2
== − sin θ 2
" +C =
= − cot θ + C = − +C TODO! figure 16 16 sin2xθ √16 1 − x216 sin−1θ
2
TODO figure −
16 16 x =! (csc
TODO figure
2
θ
1!
− 1) dθ = − cot θ − θ
1
+ C = −
x
− sin x +
16. √ dx1 =− x = 4cotsin θθ,+ C
dx = 4 cos θ dθ
Z TODO figure ! x2SUBSTITUTIONS 16. √ dx x = 4 sin θ, dx = 4 cos θ d
116 − xx22 16 − x!2 16 !# √
7.5. TRIGONOMETRIC
1 $2
x3 17. x = tan = xθsec
! √ dx
17. √ dx x = tan θ, dx = sec2 θ dθ x 1 + x 2 1 θ, dx
=
4 cos
16 − 42cosθθ dθ
x 1 + x2 7.52. Trigonometric
√ dx x = 2 Substitutions
sec θ, dx = =
2 =
sec θ−tan θ dθ " " + !C =
x! 1616sin secθθ1
x2 − 4
sec216 θθ!sin16
x2 θ− 16 16 − sin162 θsin
2 2
Z Z Z ! √ !
= 1 =
sec θ sec θ 8 sec √
2 3
1 θ dθ
tan1θ)=dθ−=+8 cot
− x 2
= x√= sin figure (2 sec θθ dθ θ2+θCdθ
sec 4
tan θ
= √ dθ = dθ = csc θ1. dθ x2 dx TODO ! 4 sec θ 1− 4
2θ, dx = cos = −tan θcot1θ16
16
+#tan
C θ
$ $ %% √
tan θ 1 + tan2 θ tan θ !!" 1 22 θ !
#
# 2 1

√!
$
1 16 3− x 2 % 1 +2
√ 17.
= 8 (1 √
= 1+ −tansin θ) sec
dx 2
= θlndθ| csc=x 18=θcos
=− tan
tan
−θcot θdθ
16 +−
θ, =3dx
θ| +tan
2C = sec
θ=2 + lnC θ+
% dθ C
2 + x2cos θ dθ = cot
1 + x2 2 16
x x +dθC %
1 x sin 1 θ = − sin θ x
!" 2 1 2 ! 16 √ !
= ln | csc θ − cot θ| + C = ln − +C = 4 x 2− 4TODO
TODO + (x figure − 4)3/2 + C sec
x2
θ 1 − x2 se
x x = (csc figure
θ− ! 1)3dθ = −=cot θ − θ + √ C=− dθ− = sin−1 x +
TODO figure TODO ! figure 1 tan θ 1 + tanx2 θ ta
Z TODO figure ! 17.1 √ dx x = tan θ, dx =2sec2 θ% dθ
1
7.5. TRIGONOMETRIC
18. SUBSTITUTIONS
√1 x dx 1 + x2 x = tan θ, dx = sec θ dθ% √1
3. ! 3 17. 1 + = tan
! =2 θ+secC2 θ=dθ %
ln!% ! s
x 2
√ x 2dx
18. √ dx x = tan θ, dx = sec θ dθ 2 x =!ln | csc θ θ,
x − cotdx sec θ|
7.5 2. Trigonometric
√ xx =12+ sec 2
xθ, dx = 2 sec θ tan
Substitutions = θ dθ dθ = % se
x2 1 + x2 4. x2 − 4
dx
x
=
! sec
tan
2√
θ
√2 θθ! 1 + tan dθ
2 !
θ = sec t
Z Z √ ! sec % √θ
! TODO 8 secfigure = tan 1 + tan = % ta1
sec θ sec θ
2 3 2√ 2
5. 1 − x 2 θ θ dθ
θ
dx = !x√= (2 cos
sec θθ dθtan dθ|1= +8θtan−sec
4 % θ
= √ dθ = dθ 1. sin2θ,θ −
4 sec dx = tan =θ)θln csc 2 θ dθ
cot θθ| + C = lntan%
x2 1 4 $ 2%% √ %
tan2 θ 1 + tan2 θ tan2 θ 6.
!
18. ! "1 − √ 21 dx= −(sin ! x
# −1
=
θ) tan + C
θ, 1 =
dx
! −= csc
sec θ %+θ dθC
1 +=
√= 8 (1 x + tan sin
1 +θ)xcos
θ sec 2
=θ θdθ
lndθ= | csc cos
= 8 θ tan 2
−2 cotθ θ +θ| + tan θ= 3
+C
Z TODO figure cot2 dθln %
2 2
= dθ = 3 C
7. 1 + xsin
TODO 2
figure
2
!θ ! sin θ
sec2√θ
% !x
= (sin θ)−2 cos θ dθ = −(sin θ)−1 + C = − csc θ + C = − =! 4!"√x218. 2− 4+ + C
1 21
√ − 4)=
(x 3/2
dx+C x = √tan θ,1 −dxx2= dθ sec2=θ dθ
8. TODO
= x(csc
1−x figure
θ − 2 1)
x3 2 =
dθ 1 +− cot
x 2 θ − θ +2C = −
tan θ 1 + tan 2 − sin−1 x +
θ
√ TODO figure
7.5. TRIGONOMETRIC 19. ! SUBSTITUTIONS dx x = sin!θ, dx =sec cos x
2 θ dθ !
Z 9. TODO figure x41 = tan
θ
dθ =
1 − x2 3. !
18. 7.5. √
TRIGONOMETRIC dx ! = " x =
−(sin θ) −1
θ,
SUBSTITUTIONS
tan 2+
dx √= sec2 θ dθ
C 1 =
+ −
tan csc
2! θ + C
19. dx x = sin θ, dx = cos θ dθ 7.5
10. Trigonometric
x3 x2 1 + 1x
2
Substitutions ! θ1tan − sin2 θ
θ θ
cos
! θ
2
x4 2. √ dx x = 2 sec
TODO θ, dx==x2 sec
figure 4θ dθ
sec 2cos−1 θ dθ = 4 se
4. =
sin θ√ Substitution θ + = csc +
dθ = sin $θ
2−4 −(sin θ) C − θ C
Z p Z Z 11.
! √ x
1 − x2 7.5 !! √ Trigonometric = ! 2# √ ta
1 − sin2 θ cos2 θ 1.
25.
dx x = 81 sec
sin −
3
θ,
TODOθ
x 2 =figure
dx cos θ dθ
(2 sec1θ tan θ)
tan 2
θ 1 + tan θ 3
θ csc2 xθ dθ 19.= ! √ 3 sin θ, dx =1 cos θ1dθ
dθ = 8 sec θ dθ 4
− x2
= cos θ dθ = dθ = cot12.
2
√ −
"!4 sec24!1θ − √4x dx=2 − cot
x= θ+ C=−
sin4 θ sin4 θ 6. 1.
! 1 −xsin
= 8 (119.
2 1
θ − dx x= 2
3−(sin
! #
" xcos
θ) =
−1
2
θ +θ,
sin
= sin C 1
!
=
dx 3− =
$ x
csc
cos
3 = cos
θθθ+ !dθC =
2 x2θ)cos == 8 x 2=
!3 = + tan sec4 θθ
2 dθ !dx 2 θdθ
tan θ, tan dxcot 2
θ dθ dθ
θ + +C
2
√ 13. sin θ x dθ
1"sin− sin θ3 √ cos
7. ! √
1 1 1 − x2 114. TODO !
" x22figure−1 1 = ! ! 4 "
2cos
21 − θ xdθ2 = !! 4
sin 1 1
− sin
sin θ θ=dθ sin xco c
= − cot θ + C = −
3
+ C = − 38.(1 − x )20. =+ 2 3/2
!4C √(csc
x −2 dx(x =−−4)cot
4θ4−+1) dθ
2 x==θ=+−sec
3/2
Cθ + θ,C dxθ −
= −= sec θ
cosθ tan
cos θ−
# dθ
dθ sin
√ = sis
−1
+
3 3 x 3x15. 1x− x2 3 ! 1! sin sin sin
24θ
θ x
!
TODO
TODO figure 19. dx xtan= θ 13θ,θ + dx = cos1θ dθ #1tan √− 2xθ 2

Z √ 7.5. 7.5. 9.
TRIGONOMETRIC
TRIGONOMETRIC
16. ! SUBSTITUTIONS
SUBSTITUTIONS x4 == − ="
cot
− sec
3=4 θ(csc 2 θ3tan
cot θ
C =θ dθ
−θ+ 1)
− =1
Cdθ=3= − −!cot x1θ3−−θxθ
x −1
2 3. x 3 ! sec 3sin 3 sec x2#θ
2.
10. √ !dx x= 2 sec θ, dx = 2 sec θ1tan − θ"dθ2 θ # cos
20. dx x = sec θ, dx = sec θ tan θ dθ 7.5 Trigonometric
17. x −814
2 √
! Substitutions
x 2θ 2 !− √ 1 2= 81 4
! √
cos θ dθ2= $
1 + cos 34θ4
x 4 4. = (1 + ! 2 TODO
cos x + figure
cos 2θ) 1 =
− 1 sinx34sin
dθ 1
θ θ, + !21 cos 2θ +
x − 1 sin d
θC
4 20. = dθ sec dx =4 θsec θ tan
2
8 sec3 θ xdx =
! 11. tanθθ)+ == θ2tan +dθ
Z Z Z √!20.x4 xC sec #= sec θ dθ

1 ! "= (24 sec
3 #θdx " 8 3θ,secdx dθx√ #θ$ 3
tan θ tan θ2 2
1. 5. − x 81 x =3 sin θ,4 dx sec=θxcos
2 3 x
−14 dθ 1 ! =3!#81 3 θ + sin12θ + 11 − ! !x +2 2
dθ = sin2 12.
dx
= +! 2 cos √ 2θ + θcos 4θ
= sec θ tan θ dθ = θ cosx2 θ dθ 4 "TODO 2 2. 2 = dx − dθ cot
tan xθ4θ=+ tan2C2sec
θ =1θ, − dx =82sin
$ sec4θ θtantan
tan
sec4 θ sec3 θ 6. ! = 8 (121+ figure
tanx22 θ)
− 4=
sec 2
!θ 3 dθ = = 8 tan secθ
! + θ tan
sec tan
θ 3tan
θ3

θ θ =
+
dθ x
C=
13. 243 181− sin θ 81 cos2!sec
θ 4 θsec4 θ 332 7.5. TRIGONOM secsec3
√ ! 3 7.
= =
8
θ + !sin √sin 2θ +
44"2 xθx22−−4 1
cos θ sin
dθ =4θ
321 (x2 − 4)sin
+ C dθ =8 sec θ # √ √
cot dθ # $$3
= (2 sec 2! tan
2
3/2 θ √
1 1 θ √13 θ)
1 1 x −1
2 1 14. ! = + +
1 ==sec C 3√ 2 12= 2 x
x 1.−θ dθ −
1 1 − x2
3/2= 243 θ+4θ,+Csec 4 θ tan
20. 81 3 81 = x
= sin3 θ + C = + C = 3 (x2 −8. 1) + + 2 θ sin
=8Cθ(csc θ 4cos
1)
x θ+
=
dx
cot sin 2θθ cos
+
3 sin
sinC!2θ
3= C 1+
θdx =θ=
− Cx− sec
3sin−1 x x+ Cx2 + #
TODO figure 2 − dθ − 16 ! 3θ − −
x 23

3 3 x 3x 15.
243 81 81 figure= 8 θ (1 + tan θ) sec2 θ dθ
!x
=2 8θ
TODO 9. figure = + sin cos TODO + sin tan
cos − 2 sin 2
+ tan
Z 7.5.3. TRIGONOMETRIC
16. 8
θ
2TODO
θ figure8=
SUBSTITUTIONS
θ θ θ(1 sec θ tan θ dθ =θ) C
" sec4 θ # sec3 θ
1 1 1 "
!
x2 10.
4. x3 243
− =θsin 4 θx2 +−C4 + # (x
1 2
√ − 4)!3/2 $3+
21. = 3 sin = 3 cos = θx+=812 sin
sec θθ,cos + θ tan 2
dx x θ, dx θ dθ 2. 17. √ dx dxθ= 2 sec dθ x3 C
(9 − x2 )3/2 7.5 x2 −Trigonometric
4 8 3" 2 8 1
Substitutions 4 3 1 3 x2 −2.1 √x2 − 4
11.
5. ! = x sin # θ + ! C = +C
Z! √ = =243 8 sec3TODO figure
5 1
Z Z θ sec θ−tan3sin θ) dθθ =+8 C sec4 θ3dθ
θ x
2 θ√ + 81 sin θ cos(2 2
2
9 sin θ sin θ 2 12.
6.
1.
1 − x8
2 dx 4 secx2 =
θ −sin4 θ, dx8 = 4cos θ dθ
= 3 cos θ dθ = dθ = tan θ
x2 243 ! TODO
dθ 3.x " 2 figure
$ " #
5 1 2 1
# $
(9 − 9 sin2 θ)3/2 cos2 θ 13.
7. = =
8
8 sin−1
(1 +
!
3
+
tan 9x
θ) 9
sec
1 − sin θ
2

2 θxdθ2 = 8− tan
8 36
!xθ + +2 C
tan3 θ !+ C
cos3 θ
Z = 4. cos θ dθ = dθ = cot2 dθ !
x 14.8. x"x−1
243 x 1 $
2 − 4 + sin
2
θ sin2 θ 1
= (sec2 θ − 1) dθ = tan θ − θ + C = √ ! − sin−1= = 4 sin
+ C! x(x2 9−−4)x3/22 (45
+−C 2x2 ) + C √ 3. √
8 3 83 1 − x2 x − 36 2

9 −$TODO
x215.
9. figure 3 = 5.(csc2 θ − 1) dθ = − cot θ − θ + C 7.5.
=−
x
sin −1
x+
− Alternativel
TRIGONOMETRIC SUBSTIT
2 11. x + 4 dx
2
x = 2 tan θ, dx = 2 sec θ dθ ! "
16. 7.5 Trigonometric 4. 43 +
−Su
3. 10. TODO !figure
$ 6. ! ! √
x2

! = 3 1 − x2
4 tan2 θ + 4 2 sec2 θ dθ = 4 sec3 θ dθ 1. x = sin θ, d
4. 17.
dx
11. x
7.x = 2 sec θ, dx = 2 sec θ tan θ dθ x2
2. √ dx ! "
1 − sin2
2
4 Section 7.3, Example 5
x2 −See =
5. 12. ! ! sin2 θ
8. 8 sec θ (2 sec θ tan θ) dθ = 8 sec4 θ dθ = (csc2 θ − 1)
=θ + √
3 !
= 2 sec θ tan 2 ln | sec θ + tan θ| + C
6. 13. √ 4 sec2 θ' √
−4 '
x2 + 4 9.% !
x& ' x2 + 4 x ' # TODO figure $ 5.
'2 2 + '' + C 1!
7. 14. = 2 + 2 ln
= 8 (1 + tan θ) sec θ dθ = 8 tan θ +2. tan
' x33 θ + 6. C
2 2 ' 2 2 ' 3 x2 − 4 dx
√ x = 2 sec θ,

15. x $ 10. " ' $ 1 ' 7. !


8 sec3 θ
8. x2 +=4 4+ 2xln
' ' = √
= 2−' 4x+ 2 + (x
4 +−
2
x'4)
+ C1+ C
3/2
8. 4 sec2 θ −
2 3 !
!9. 16. ! 11. 9. 8 (1 + tan2
=
x TODO1figure 1 1
12. dx = (2x dx) = ln(25 + x 2
) + C 10. " 2 1
10.25 +17.
3.x
2 2 25 + x12.
2 2 = 4 x −4+
3
! TODO figure 11.
1 x
13. 11.√ 4. dx = sin −1
13.
+ C 3. 12.
25 − x2 5 13.
4.
12.
! 5.
1 1 14.
|x| 5. 14.
14. √ dx = sec−1 +C
13. 2
243 81 81
θ+ = sin θ cos θ +
8 2 16
243 81 81
446 = θ+ sin θ cos θ +
8 2 " 8
243 1
TODO figure8 = θ + 81 sin θ cos θ +
2
! "
1
16. √ = 243 θdx + 81 sinxθ=cos 4 sin
θ θ, −
5 d
x2 16 − x 82 8
!
243 $ 4 co
448 CHAPTER 7. TECHNIQUES OF INTEGRATION = −1 x
sin = + 9x 29 − "x2
446 8 3 16 sin θ 16 C
243 x 1 1 $
7.5. TRIGONOMETRIC SUBSTITUTIONS = sin−1= − + cot x θ 9+−Cx
Z 8 3 16 8#
x2 TODO ! $ figure √
22. dx x = 2 tan θ, dx = 2 sec 2
θ dθ 7.5 Trigonometric 11.
! 4 +Substitutions
x2 dx x = 2 tan = θ,

1dx = 216 sec−2 θx
(4 + x )
2 3/2
16. √
1 x
!
dx $ x =2 4 sin θ, dx
16 x
=
Z Z ! √
4 tan2 θ tan 2
θ 1.
1 − x2 xx2 = 16sin −x
TODO
2 =
= figure
cos
4 tan θ + 4 2 sec2 θ dθ
dx θ, dx θ !

= 2 sec θ dθ =
2
dθ x2
! "
!
2 1 =See
4 cos θ
(4 + 4 tan2 θ)3/2 sec θ 17.
1 − sin2 θ√ dxSection
!
cosx
2 = 16 sin2 θ
2 = 7.3, "Example
!
θ tan θ, dx 2=
Z Z Z = x 1
cos +θ xdθ 2 2 ln |=
dθ 16 − cot16
sin2 θ = 2 sec θ tansin !θ + sec2θ + ta
sec θ − 1
2
! √ 1
θ
sec√' √θ
= dθ = sec θ dθ − cos θ dθ = ln | sec θ + tan θ| − sin θ +=C−=cot x−2 +=4 cot% &θ +√
− θ +xtan
θ16
C ''1 −x2x
sec θ = (csc2 θ − 1) dθ = 2
2
C = −1 +
# 2√ +θ2 ln ' $ tan
' x 2
√ 1 16 − x 2
4 + x2 x p TODO
figure x $ 2 = ln | csc θ'$ '
− cot θ| +
x x = =− x + 4 + 2 ln x ' x2 + 4+ +
= ln + − √ + C = ln 4 + x!2 √ + xx3 − √ + C1 2 16
2 2 4+x 2 2.
x2 − 4
dx 4 + xfigure
TODO
!x = 22sec θ, dx
xTODO figure
=
1
2!sec θ tan
1
θ dθ
1
12. ! ! dx = (2x dx) ! = ln(
Z 7.5. TRIGONOMETRIC 258 +
secx32θ
! SUBSTITUTIONS 12 25 + x2 2
= ! √18. 1 √ (2 sec θ tan
dx θ) dθ x = tan θ, θdx
= 8 sec 4

1 17. √ 4 sec12 θ − 24 1 + x2
xdx x =x tan θ, dx = sec$2
23. dx x = tan θ, dx = sec 2
θ dθ 7.5 Trigonometric 13. x ! √
1 +Substitutions
x 2 dx = sin −1 #!C
+
− x22θ) sec2xθ!dθ =58= tan θ + 1√ sec23θ
(1 + x2 )2 = 8 (1 25
!
+ tan
sectan 2 2 3 tan θ
θ θ 1 + ta
Z Z Z ! √
" √ 1 1 dx== 1 sec−1 √ |x| dθ
sec2 θ 1 1 − x2
dx 14. θ 5 1++Ctan
− 325(x − 4)5 tan
1. = 4 x x= 2 sin θ, dx =2 cos θ3/2dθ + C 2
θ
= dθ = dθ = cos 2
θ dθ x2 x −x42 + = −(sin θ)−1 + C =
sec θ 1 21θ
!
(1 + tan θ)22 2
TODO figure 15.!
"
1√− sin
!
=θ cos
4 sin
2 !
Z = TODO dx = ln | xcsc
figure
2 − x2cos θ dθ =
16
−2 θcot
θ,dθdx ==
θ| +4C cos=
cot 2
1 1 1 1 3. 1 x sin ! θ √ ! sin θ
= (1 + cos 2θ) dθ = θ + sin 2θ + C = θ + sin θ cos θ !+ C19. 1 − x=2 4 cos θ √
1 −=x
2 2 4 2 4. 2 =TODO (csc2figure
θ − 1) dθx= dx x$= sin θ, dx
4 − cot θ − θ + C = −
4 sin θ 16 − 16 sinx2 θ
  !  ! "
1 1 x 1 1 5. TODO −1 figure 1 x√ 1 1
θ −1θ, − sin 2
θC =
= tan−1 x + √ √ + C = tan x3+18. 2 12+ x+ 2 C
dx = 4 ln x=|=csctan cot
sin
θ| +=
dx sec
cos
2 2 2 2.6. √ x 2dx 1 x+=x2 sec θ, dx = 2 sec
! 4
x
1+x 2 1+x 2
! θ tan θ dθ 2
θ
7.5. TRIGONOMETRIC x2 − 4 SUBSTITUTIONS = 1 sec3 θ
Z 7. ! = −2 cot √ θ! + C = −
x2 =
8 sec3 θ tan 3θ 1 + tan4 2 θ
(2 sec θ tan θ) dθ = 8 sec θ dθ
24. dx x = sec θ, dx = sec θ tan θ dθ 7.5 Trigonometric
8.

x4 sec !θ −
Substitutions
2 √4
(x2 − 1)2 ! x2 −=1 −(sin # θ)x−1 + Cθ,1 = c$
Z Z ! √
Z = 8 (120.
+ tan 2
sec
θ) x 2 dx
4 θ dθ = 8 tan θ +
= sec tandx−3=
θ
9.1 − x2
sec θ
2
sec θ 1.
3 dx x = sin θ, dx = cos θ dθ
TODO figure1 ! 3
= sec θ tan θ dθ = = 10. cscx23 θ dθ
! "
"
=! 4 √x221− 4 + (x2 −!4)3/22=
tan θ
+ C sec!4 θ sec θ tan θ
(sec2 θ − 1)2 tan3 θ 1 − sin1 −θ x2 3 cos θ
=
11. TODO figure 19. sin2 θ 4 cos θdx dθ =
xsin=2 θsindθ = cot2 dθ #
θ, dx = cos θ
1 1
u = csc θ, du = − csc θ cot θ dθ; dv = csc 12. 2
θ dθ, v = − cot ! θ x
!
3√
"= 3 sin θ1+− Cx2= 3
Z 3. Z = (csc θ − 1) dθ = − cot θ − θ + C = − 2
2
− sin
1 − sin θ x
13. TODO=figure cos θ dθ =
= − csc θ cot θ − 2
csc θ cot θ dθ = − csc θ cot θ − csc θ(csc θ − 1) dθ
TODO 4. figure 2 sin4 θ
#√
! 14. 3
Z Z 5. x 1 θ dθ3 1
2. √ dx x = 2 sec θ, dx = 2 sec
= tan
θ− cot θ + C = −
15. x −4
2
3 3
= − csc θ cot θ − csc3 θ dθ + csc θ dθ 6. !
8 sec3 θ
!
16. = √! √ (2 sec θ tan θ) dθ = 8 sec4 θ dθ
Z 7. 4 sec2 x
θ 2−−41
20.!
dx x# = sec θ, $
1 dx = sec θ
= − csc θ cot θ − csc3 θ dθ + ln | csc θ − cot θ| 17.
8. = 8 (1 + tanx 24
θ) sec2 θ dθ = 8 tan θ + tan3 θ + C
3
!
Z 9. " 1 tan θ
= 4 x2 − 4 + (x2 − 4)3/2 = + C 4 sec θ tan θ dθ =
3 sec θ
Solving for the integral csc θ dθ, we have
3 10. figure
TODO #√
1 1 x2
3. 11. = sin3 θ + C =
Z 3 3 x
1 1 12.
csc θ dθ = − csc θ cot θ + ln | csc θ − cot θ| +4.C
3
TODO figure
2 2 13.
  5.
1 x 1 1 6.x 14. 1
+C
=− √ √ + ln √ −√
2 x −1
2 x −1 2
2 2 1
x7. −15. x − 1
2
 
1 x 1 x − 1 8. 16.
=− √ + ln √ + C.
2 x2 − 1 2 x2 − 1 17. 9.

10.

11.

12.

13.

14.

15.

16.

17.
243 81 81
θ+ = sin θ cos θ + sin 2θ cos 2θ
8 2 16
243 81 81
= θ+ sin θ cos θ + sin θ cos θ(1
8 2 " 8
243 1 1 1
= θ + 81 sin θ cos θ + − sin2 θ
8 2 8 4
" #
243 5 1
= θ + 81 sin θ cos θ − sin2 θ +
8 8 4
$ " #
243 5 1
7.5. TRIGONOMETRIC SUBSTITUTIONS 449 =
x
sin−1 + 9x 9 − x2 − x2
8 3 8 36
243 x 1 $
7.5. TRIGONOMETRIC SUBSTITUTIONS = sin−1 + x 9 − x2 (45 − 2x2 ) +
Z 8 3 8
! $
1
25. dx x = 2 tan θ, dx = 2 sec θ dθ 2 7.5 Trigonometric 11. 4 +Substitutions
x 2 dx
x
x = 2 tan θ, dx = 2 sec2 θ dθ
(4 + x2 )5/2 ! √
! $ !
Z Z Z 1 − x2 dx = 4 tan2 θ + 4 2 sec2 θ dθ = 4 sec3 θ
2 sec θ2
1 1 1 1.
x2
x = sin θ, dx = cos θ dθ
= dθ = dθ = cos3 θ dθ ! " 2 See Section
! 7.3, Example! 5
(4 + 4 tan2 θ)5/2 16 sec3 θ 16 =
1 − sin2 θ
cos θ dθ =
cos2 θ
= cot2 dθ
Z Z = 2 sec θ tansin +2 2 dθ
448 sin2 θ θ ln | sec θ7.
θCHAPTER + tan θ| + C
TECHNIQU
1 1 ! √ √' √ '
= cos2 θ cos θ dθ = (1 − sin2 θ) cos θ dθ = (csc θ − 1) dθ =
2 x 2 + 4 %x&
= 2− cot θ − θ + C + =2−ln ''
'1 −x2x2+ 4 −1
− sin x '' +
16 16 !
1 2 2 ' x 2
+ 7.5.' x+TR
2'
C
Z Z 25. figure 2 5/2 dx
TODO x = 2 tan$θ, dx = 2 sec2'θ$dθ 7.5
1 1 1 1 3 (4 + x ) x '
'
'
= cos θ − sin2 θ cos θ dθ = sin θ − ! sin x3 θ + C
! =
2 2θ
2 sec
x2 + 4 + 2 ln ' !x2 + 4 + x' + C1 !!
1 1 11.
16 16 16 2. 48 √ dx !x = 2=sec θ, dx = 2!sec θ tan θdθdθ= dθ =
    x2 − 4 x (4 + 41tan2 θ)5/2 1 16 1sec3 θ 2 16
12. ! = (2x = ln(25 + ) +C
1 x 1 x 3
258 + secx312θ
!
dx
22 25 + x2 1
!
dx)
2 2
x
= √ − + C = !√ =
4 sec1 16
cos
(2 sec θ θcos
tanθ θ) dθdθ = = 8 (1
16
sec− 4
θsin
dθ θ) cos θ dθ
16 4 + x2 48 (4 + x2 )3/2 13. ! √
2 θ −! 4 −1 x! # $
1 dx =2 sin 1 5 + C 2 1 1 1
TO
= 8 (1 25 − x22θ) sec
+=tan cos θθ− dθ = 8 sin tan θ + cos θtan dθ3=θ +sin C θ− !
! 16 16 3 16 2. 48
Z Z 1 " 1 # |x| $ %
x3 x2 14. " 2√ 1 1 dx x= sec−1 1 + C x3
= 4 xx −= x42 +− 325(x √2 − 4)53/2 + −C 5 +C
26. dx = (x dx) u = 1 − x 2
, du = −2x dx ! 16 4 + x 2 48 (4 + x2 )3/2
(1 − x2 )5/2 (1 − x2 )5/2 TODO figure 15. √
1
dx x = 4 sin θ, dx = 4 cos θ dθ
Z   Z !
x3
!
x 16 − x2 x2
1−u 1 1 3. 26. dx = ! (x dx)
4 cos θ u = 1 − x 2
, du = !
1 −2x
= − du = − (u−5/2 − u−3/2 ) du (1 − x2 )5/2 (1 − x= 2 )5/2
# $ ! dθ = cs
u5/2 2 2 4.
!
1−u
"
1 4 sin θ 161 − 16 sin
2
θ −3/2 4 TO

  = −
12
du = −
2
(u −5/2
− u ' ) du
1 ' 4 16
3.
1 2 1 5. u5/2
" = ln | csc θ − cot # θ| + C = ln '' − 4.
=− − u−3/2 + 2u−1/2 + C = (1 − x2 )−3/2 − (1 − x2 )−1/2 =+−C 1 2 −3/2 4 1 4 x
2 3 3 6. 2
− u
3
+ 2u −1/2
+ C = (1 − x2 )−3/2
3
5.

6.
7.5. TRIGONOMETRIC SUBSTITUTIONS 7.5. TR
7. ! !
Z Z 27.
1
dx =
1 7.

1 1 8.

2 + 2x + 10
&
(x + 1) 2+9
dx 7.58.
27. √ dx = p dx 7.5 Trigonometric Substitutions x
x+1
!
x2 + 2x + 10
9.
(x + 1)2 + 9 ! 9. √ x + 1 = 3 tan θ, dx = 3 sec2 θ dθ
1.
1−x 2
! θ dθ !
10.
1. dx x = sin θ, dx = cos 3 sec2 θ
10. x2 11.
x + 1 = 3 tan θ, dx = 3 sec2 θ dθ ! " 3 = √
9 !tancos 2
θ2+ 9
dθ =
!
sec θ dθ = ln | sec θ +
12.
11. 1 − sin2 θ ' θ '
Z Z = cos θ dθ' √ = 2 2 +dθ = cot 'dθ 2
'&13. TO
3 sec θ 2 sin 2
θ
= ln '
' x + sin2x θ 10
+√
x + 1 '
' + C = ln ' 14.
'
x2!+
= √ dθ = sec θ dθ = ln | sec 12.θ + tan θ|! + C ' 3 1− 3 x2' 2.

√ 9 tan 2
θ + 9 13.
= (csc2 θ − 1) dθ = − cot θ − θ + C = −
x
− sin−1 x + C 15.
TODO figure
x2 + 2x + 10 x + 1 p TODO figure 16.
14. ! ! ! !
= ln + + C = ln ! x2 x+ 3 2x + 10
28.
x + x + 1 + C1
dx =
x
dx =
x−2
dx +
17.

3 3 2. 15. √

dx 4x x−=x22 sec θ, dx =42−
&
sec(xθ −tan2)θ2dθ
&
4 − (x − 2)2
x2 − 4
! !
16. 8 sec 3
θ u = 4 − (x − 2) 2
, du = −2(x − 2) dx TO
Z Z Z Z = √ (2 sec θ tan θ) dθ = 8 sec4 θ dθ
4 sec2 θ − 4 1 ! 1 3.
x x x−2 2! − 2 1/2 du#+ 2 sin−11
x − 2 $
28. √ dx = p dx = p dx +17. p = 8 (1 + tan=2 dx 2 θudθ = 8 tan θ + tan
θ) sec 23θ +C
= −u1/2 + 2 sin4.−1
4x − x2 4 − (x − 2)2 4 − (x − 2)2 4 − (x − 2)2 & 3 x−2 5.
" =1− 4 − (x − 2)2 + 2 sin−1 +C 6.
= 4 x 2 − 4 + (x2 − 4)3/2 + C 2
u = 4 − (x − 2)2 , du = −2(x − 2) dx ! 3 ! 7.5.7. TR
TODO figure 1 1
Z 29. dx = dx 7.5 8.
1 7.5. 1TRIGONOMETRIC x − 2
SUBSTITUTIONS 3. x − 2(x2 + 6x + 13)2 [(x + 3)2 + 4]2
449 9. !
=− du + 2 sin −1
= −u + 2 sin
1/2 −1
+C 446 CHAPTER 7. TECHNIQ
1.
2 u1/2 21 ! 4.
1
"
1
2 # x + 3 = 2 tan θ, dx = 2 sec2 θ dθ 10.
TODO figure! ! !
p = x −(12+ cos 2θ) dθ =
16 16
θ + sin 2θ + C
2 !
1=
2 sec θ 2
dθ =
sec θ 2
dθ = 12.
11.
1
= − 4 − (x − 2)2 + 2 sin−1 1 + C 5. 16. √ (4
dx tan x θ=+
2
4)
4 sin 2 dx = 4 cos8θsec
θ, dθ 4 θ 8
θ2+
1 x2 16 − x2
= sin θ cos θ + C6. !
4 cos θ 1
! 13.
16 16 " TRIGONOMETRIC
7.5. # SUBSTITUTIONS = " = θ dθTO
csc2 14.
16 sin 2
16 16 sin 2 16
Z Z =
1
tan −1 x + 3
+
1
7.
x+3 2
+C
θ − θ !

1 1 16 2 16

2 + 6x + 13

2 + 6x + 13
1
= − cot θ + C
2.
15.

29. dx = dx x
7.5 Trigonometric Substitutions x 16
x + 3 1 # √16 − x2 $ 16.
(x2 + 6x + 13)2 [(x + 3)2 + 4]2 = 1 tan−1 x + 3 + 2 8.x +!3 √ + C =− +C 17.
16 2 8(x 9. + 6x + 13) 1−x 2 16 x
1. dx x = sin θ, dx = cos θ dθ
x! + 3 = 2 tan θ, dx! = 2 sec θ dθ 10.
TODO figure 2 x 2
! "
7.5. TRIGONOMETRIC
TODO figure
! 2SUBSTITUTIONS ! ! 443
Z 1 Z 1 Z 117.
− sin2 √ θ 1 x cos
2
= tanθθ,dθdx== seccot 2
TO
30. = = 2 x 1cos
dx
xθ2 dθ =
θ 2dθdθ
(112−sec [36 − sec
(x + 5)θ2 ]2 11. 1
2 dx 2 dx
10x −θ x2 )2 7.5 Trigonometric
sin θ +Substitutions
x sin 2
θ
= dθ = dθ = cos θ! dθ
2 ! ! ! ! 3.
√ sec 2
θ √ sec θ
= 1dθ−=x2 tan θ dθ = csc θ d
(4 tan2 θ + 4)2 x + 58=sec 6 sin4 θ,
θ dx12. = 6 cos 8 θ dθ 1.=
1 − x22
(csc

tan θ 1 + tan2 θ 4.
x2 !θ − 1) dθ = − cot θ − θ + C = − % √− sin x +
dx x = sin θ, dx = cos θ dθ −1
! ! % C
! " 1 x % 1 %% 5.
6 cos θ cos θ 1 1 − sin2 θ = ln | csc θ
!
cos 2
θ
− cot
! % 1+x 2
= dθ13.= TODO figure dθ = sec
= 3
θ dθ cos θ dθ = =+ C
dθ θ| cot=dθln %
2 − %+C
% x x % 6.
(36 − 36 sin2 θ)2 216 cos4 θ 216 !
sin2 θ sin2 θ

! TODO 2figure 1 − x2
14. 3 = (csc 1) = cot + = sin −1
x θ − dθ − θ − θ C − − x + C 7.
See Section 7.3, Example 2. 5
√ dx x = 2 sec ! θ, dx = 2 sec θ tan θ dθ x
" x2 − 4 #TODO figure 1 8.
18. √ dx x = tan θ, dx = sec2 θ dθ
1 1 15.
1 !! x2 1 + x2 !
= sec θ tan θ + ln | sec θ + tan θ|2. +√Cx√
3
8dxsec3 θx = 2 sec θ, dx = 2 sec ! θ tan θ dθ 2 ! !9.
216 2 2 = x −4
2 (2 sec θ tan θ) dθ = sec8 θ sec 4
θ dθ sec θ
" 16.# 4 sec2 θ! − 48 sec3 θ = √ ! dθ =
tan2 θ
dθ = 10. (sin
! #tan 2
θ 1 + tan2 θ
θ) dθ = 8 sec4 θ dθ $
1 6 x+5 = √ (2 sec θ tan
1 √
= √ √ 4 sec2 θ2− 4 1 + x11.2
11 − 10x − x17. 11 − 10x − x= 2 8 (1 + tan! θ) sec 2 θ dθ 2==−(sin 8 tan # −1θ +
θ) + C1 =tan $θ +C
2 3
432 2
= 8 (1 $+ tan θ) sec θ dθ = 8 tan θ + 3tan3 θ + C
− csc θ+C =−
x 12.
+C
$ 3
1 $ 6 " x + 5TODO " 1 $figure1
+ ln $ √ +=√4 x2 − 4=!+4 √x(x 2$
$−+ + 4)
2 C 3/2
4− (x2 − 4)+3/2C+ C 13.
432 $ 11 − 10x − x2 11 − 10x − x312 − x2 3 14.
$ TODO$figure 19. dx x = sin θ, dx = cos θ dθ
x+5 1 TODO $ figure
$ √ x + 11
$ x 4

= + ln 3. $+C ! " ! ! 15.


72(11 − 10x − x2 ) 432 $ 11 − 10x − x2 $ 1 − sin θ 2
cos θ2
3. = cos θ dθ = dθ = cot 16. θ
2
4. sin4 θ sin4 θ
TODO figure #√ $3 17.
5. 1 1 1 − x2 1
4. = − cot3 θ + C = − + C = − 3(
31. 6. 3 3 x 3x
7.5. TRIGONOMETRIC SUBSTITUTIONS 446
449
! " #
1 1 1 TODO figure
= (1 + cos 2θ) dθ = θ+ sin 2θ + C
450 CHAPTER 7. TECHNIQUES
16 OF16INTEGRATION
2
16.
!

1
dx
1 1 x2 16 − x2
θ+ =
sin θ cos θ + C
!
Z 16 16  =
1 1 1 1 −1 x + 3 1 " x+3
#
2
= (1 + cos 2θ) dθ = θ +SUBSTITUTIONS
= sin tan2θ + C+ √ √ +C
16 7.5. TRIGONOMETRIC16 2 16 2 16 x2 + 6x + 13 x2 + 6x + 13 449= −
1
1 ! −1 x + 3 x +"3
1 1 = 1 tan +
8(x= +1 6x θ++13) 1 + C # = −
= θ+ sin θ cos θ + C = 16 (1 + cos 2 2θ) dθ 2
sin 2θ + C 1
16 16 TODO figure

16

16 2 TODO figure
1 x+3! 1 1 x + 3 !1 + 1 sin 1 θ cos θ2+ C
7.5. TRIGONOMETRIC ! SUBSTITUTIONS
1
= tan−1 30. + (11 − 10x √− x2 )2 dx== 16 θ[36 16√
(x + 5) ]
dx + C 17. √
x 1 +Substitutio
x2
dx x
16 2 16 " 7.5
# Trigonometric
− 2 2
x + 6x +1 13 −1 x +x 3 + 16x + 13x + 3
2 2
! √ 22
x!
= tan + dx =√6 cos θ dθ √ 1−x +C =
1 x+3 x+3 16 x + 5 = 62 sin θ, 16 x2 + 6x + 13 1. x2 + x2 6x
dx+ 13x = sin θ, dx = cos θ dθta
= tan−1 + !+ C
1 + θ3
6xcos
!
x + 3 cos θ 1
! ! " 1 !
16 2 8(x2 + 6x + 13) 1 − sin θ = ln | c
2
== tan−1 + 2 dθ2 = + C dθ = sec
= 3 θ dθ2 cos θ dθ =
16 (36 − 362sin2 θ)8(x + 6x + 216 13) cos4 θ 216 !
sin θ
Z Z ! 7.5.! TRIGONOMETRIC SUBSTITUTIONS TODO
= (csc2figure
θ − 1) dθ = − cot θ
1 1 30. 1 See Section 1 7.3, Example 5 !
30. dx = dx dx = " dx
6 # TODO figure
18. 6

1
[36 − (x + 5)2 ](11 2 − 10x − x2 )2 7.51 [36Trigonometric
1− (x + 5) ] 1
dx x
(11 − 10x − x2 )2
2 2
Substitutions +√C5x23 dx xx = 21sec + x2x + 5
sec θ tan θ + ln | sec θ + tan θ|2.x +
! 2
= ! θ
θ, dx = 2 sec
216x!+2√ 5 = 6 sin θ, dx 2= 6 cos θ dθ x −4 θ =
x + 5 = 6 sin θ, dx = 6 cos θ dθ 1. !1 " 1 − x26 dx # !
x = sin θ, +dx5θ = cos θ dθ
xcos
√!
= 11√ − 810xsec−3
θx2 t
(2 sec θ tan
= √ 6xcos
2 θ √ 4 sec2 θ − 4
Z Z = 432(36 −1136−sin dθ = dθ !
=
10x
2 − 2 " 11 − 10x4− x2
216 cos θ −(s
6 cos θ cos θ ! θ) $!
2 x
1 − sin2 θ
! = 8 (1 + tan
cos2 θ $$
! θ) sec θ dθ =
2 2

= dθ = 1 dθ 3+ 1 ln = $$ 62 cos θ dθ = x + 52TODO
+
"dθ2 $=figure1cot2 dθ
+
(36 − 36 sin θ) 2 2 216 cos
= 4
216 θ
sec θ dθ √
See
432 $! 11 − 10x − x2Section
sin θ 7.3, √
Example 5sin =!θ
11 − 10x − x12 −
4 √ x $ x√
− 4 +C (x2 − 4)3/2
2 3
Z " $ # TODO$figure19. 1dx− x2 x =
1 x1 + 5 1 θ$$ − 1) x dθ+=11 − cot θ$$− θ + C = −
4
1 = 1(csc 2 x −s
= sec θ dθ See Section 7.3, =
= sec tan + ln ln
sec + tan 3. + C + C x ! "1
216 Example
−2 10x − x2 ) 5 2432 $ 11 − 10x − x2 $
3 θ θ | √θ θ|
72(11
216 TODO
" figure # 4.
=
 TODO figure 1!  6 x+5
1 1 1 = √ x3 √ 5. 1
432 √ 11 − 10x dx − x$ x = 211sec
2 −θ,10x x22 sec θ tan θ dθ = − co
= sec31.
θ tan θ + ln | sec θ + 2. tan θ| x2 + −4 1C dx−=6. $ 3
216 2 2 +
$
ln !$ √ 8 sec3 θ
6
+
7. √
x + 5! √ !$$
+1C
 32.  432 = $ √ 11 − 10x − x(2 2
sec 11
tan− 10x
θ)20.
dθ =
x2 −
− 8x2x$4 secdx 4
θ dθ x =
1 6 x + 5 x+5 $
4 sec$ θ −x4 + 11
2 8. θ $
= √ 33. √ = +
1
! ln $ √ 9.
$
$ +#C $! tan
432 11 − 10x − x2 11 − 10x 72(11− −x210x − x2 )= 8 432(1 + $tan11 2 − 10x 2 − x2 $
θ) sec10. θ dθ = 8 tan θ + tan3 θ +secC
1 =
34. ! 3
1 x − 3
6 ! x−3 x +"5 2 1 11.

1
= sin3
+
31.
35. ln √ dx = + √ =4 x −4+ +
dx (x C
2
− 4) + C
3/2 x 2 + 6x + 13 3
432(5 − 4x −11 x )
− 10x − x[92− (x + 2) 11] − 10x − x2 3 12. x+3
2 3/2 2 3/2
TODO figure
36. xTODO
+ 2 = figure
3 sin dx = 3 cos θ dθ
θ, 13. θ
x+5 1 !x + 11 ! 2
= 37. + ln √ = 3. 3 sin θ − 5 + C θ dθ) = 9 sin 14.
θ cos θ − 15 cos θ
72(11 − 10x − x2 ) 432 11 −(910x
− 9 sin − xθ)3/2
2 2 (3 cos 15.
27 cos3 θ

38. 4.! ! ! 16. !
Z Z 7.5.1 TRIGONOMETRIC
sin θ 5 1SUBSTITUTIONS 1 5
x−3 x−3 = 5. dθ − dθ = tan
17. θ sec θ dθ − sec2 θ dθ
31. dx = 39. dx 7.5
3 cos2 θ
Trigonometric
9 cos2 θ
3
3
Substitutions
9
(5 − 4x − x2 )3/2 [9 − (x + 2)2 ]3/2 1
= 6.sec θ √
5
− tan θ + C x+2
40. 3 "! 9 2 #
1−x " #
x + 2 = 3 sin θ, dx = 3 cos θ dθ 17.1. 3 dx 5x = sin θ, x+ dx2 = cos θ dθ
41. = √ x2 − √ +C
Z 3 5 − 4x − x 2 9 " 5 − 24x − x 2
3 sin θ − 5 8.
−5x − 1
!
1 − sin θ
!
cos2 θ
!
= (3 cos θ dθ) = √ + C=
sin 2 cos θ dθ =
sin 2 dθ = cot2 dθ
(9 − 9 sin θ)3/2
2
99. 5 − 4x − x2 !
θ θ

Z TODO figure
1 − x2
9 sin θ cos θ − 15 cos θ = (csc 1) = cot + = −s
2
10. θ − dθ − θ − θ C −
x
= dθ
27 cos θ 3
11. TODO figure
Z Z Z! Z
1 sin θ 5 1 12. √ x3 dx
12. 5sec θ, dx =2 2 sec θ tan θ dθ
= dθ − dθ = tan x2 θ− sec
4 θ dθ − x = 2 sec θ dθ
3 cos2 θ 9 cos2 θ 3
13. ! 9 !
8 sec3 θ
1 5 = √
4 sec 2θ−4
(2 sec θ tan θ) dθ = 8 sec4 θ dθ
= sec θ − tan θ + C 14. ! # $
3 9    = 8 (1 + tan 2
θ) sec 2
θ dθ = 8 tan θ +
1
tan 3
θ +C
15. 3
1 3 5 x+2
= √ − √ 16. + C = 4 x2 − 4 + (x2 − 4)3/2 + C
" 1
3 5 − 4x − x2 9 5 − 4x − x2 3
−5x − 1 17. TODO figure
= √ +C 3.
9 5 − 4x − x2
4.

5.

6.

7.

8.

9.

10.

11.

12.

13.

14.

15.
7.5. TRIGONOMETRIC SUBSTITUTIONS 446
449
! " #
1 1 1 TODO figure
= (1 + cos 2θ) dθ = θ + sin 2θ + C
450
!
CHAPTER 16 7. TECHNIQUES 16 2 OF INTEGRATION 1
16. √ dx x = 4 sin θ, dx =
1 1 x2 16 − x2
= θ+ sin θ cos θ + C !
4 cos θ
16 16 SUBSTITUTIONS =
! ! " TRIGONOMETRIC
7.5. # "
1 1 =
1
tan−1
x+3
+
1

x+3

2
+C
16 sin2 θ 16 −
32. dx = dx 16 2 16
7.5x +Trigonometric
2 6x + 13 x2 + 6x +Substitutions
13
1
= − cot θ + C
(x2 + 2x)3/2 [(x + 1)2 − 1]3/2 1 x+3 x+3
16
x + 3 1 # √16 − x2 $
7.5. TRIGONOMETRIC SUBSTITUTIONS =
16
x + 1 = sec θ, dx = sec θ tan θ dθ
tan−1
2
+ ! √
8(x2 + 6x + 13)
1.
+C
1 − x2
dx
451 x = sin θ, dx = cos θ dθ
=−
16 x
TODO figure x2 TODO figure
! ! !! ! ! " ! 22SUBSTITUTIONS ! !
sec θ tan 1θ sec θ1
7.5. TRIGONOMETRIC
Z Z 7.5. = 30.
TRIGONOMETRIC 2 =
dx =
2 )SUBSTITUTIONS 5)2=
dx (sin θ)−2 cos=θ dθ 17. 2 x√1cos
1 − sin θ 1
dx
xθ2 dθ =
x cos
2
= tanθθ,dθdx== se
1 1 (11 − 10x − xdθ
(sec2 θ − 1)3/2
[36 − (x +dθ
tan2 θ
]2 7.5 Trigonometric sin θ +Substitutions
x! sin θ 2
2
sec θ √
32. dx = dx x + 5 = 6 sin θ, dx = 6 cos θ dθ
x + 1!
1.=
! !√
1 − x22
(csc
x2 !θ − 1) dθ = − cot θ − θ + C = −
dx
=
x = sin θ, dx = cos θ dθ

tan θ 1 + tan2 θ
1

(x2 + 2x)3/2 [(x + 1)2 − 1]3/2 7.5 x +Substitutions


Trigonometric!
1
= −(sin θ)−1 + C = − csc=θ + C 6=cos−θ √ dθ = TODO +cosC θ
dθ =
1
sec
! "
= 3 θ dθ2
1
1 − sin2 θ = ln | csc θ
!
cos 2
−2 θcot
!
=+ C
dθ θ|
figure cos θ dθ = co
(36 − 36 sin θ)x + 2x216 cos θ
2 2 2 4 216 !
sin θ sin θ

! √ ! 1−

x + 1 = sec θ, dx = sec θ tan θ dθ


3 TODO
= (csc2figure
θ − 1) dθ = − cot θ − θ + C = −
TODO figure 1 − x2 See Section 7.3, Example 2. √5 x dx x = 2 sec ! θ, dx = 2 sec θ tan θ dθ x
1
1. dx x = sin θ,
1
"dx = cos θ dθ
1 1
x2 − 4 #TODO figure 18. √ dx x = tan θ, dx = s
Z Z !
Z x2 = sec θ tan θ + ln | sec θ + tan θ|2. +√Cx√
7.5.! ! TRIGONOMETRIC
3
8dx
x2 1 + x2
sec3 θx = 2 sec θ, dx = 2 sec
SUBSTITUTIO !
2x + 4 1
! θ tan θ dθ 2

sec θ tan θ sec θ = (2 sec tan = 8


" 216 2 2 2−4 θ θ) dθ sec θ sec
33. dx = ln(x2 + −2 4x + 13)! + C1 − sin #! !4 sec2 θ! − 48 sec3 θ = #tan2 θ!√1 !+ tan4 2
x
"
2
cos 2
= = = (sin θ) cos θ dθ
x2 + 4x + 13 θ x + 5θ tan θ)1dθ = 8 sec θ
1 6 ! =
dθ dθ 2 √4 sec2 θ2− 4 (2 sec θ = (11+ c
= = √ cos θ dθ = √
2 −dθ x=2 = 8 (1 + cottan2! θ)dθsec θ dθ 2==−(sin 8 16 θ) + C1 =ta−
tan
# −1θ +

(sec2 θ − 1)3/2 tan θ ! 2 450 432


sin θ
2 11 − 10x − x2
$
11 − 10x
sin θ = 8 (1 $+ tan2 θ)CHAPTERsec θ dθ = 8 tan θ + 3tan37
3
1 1 −1 x − 3 ! +
1 $
ln $$ √
6 "√ x +
+=√4 x2 − 4=!+4 √2x(x
5 TODO"1 $ 2 $figure
2 −+
1 3/2
=
1
+
1
si
34. dx = tan + C SUBSTITUTIONS432 110x 4
−−xx312 −$ x2 3 449−+ C 4)
(x 2
4)+3/2
C + θ
x+1
− C
7.5. TRIGONOMETRIC 11 − 10x − x!2 11 −446 16 16
4 + (x − 3)2 2 2 = (csc2 θ −!1)xdθ = − cot θ − $θ + C = TODO − $figure19. − sin
dx
−1x = sin
x + θ,Cdx = cos

= −(sin θ)−1 + C = − csc θ + C = − √ +C = !


+5 1 +
1" TODO $ figure
$
ln $ √
x + 11 #
$
$ +C x 1 x 4
"=
1
tan −1 x
! !2 " # 32. 172(11 − 10x − x2 ) 432 1 dx
3. θ + 11sin
1=− 10x − x2 $
3. !
dx 1 −164sin2 θ cos θ d
x + 2x
TODO figure
x2 TODO TODO 16 figure
figure =
16
(1 +(xcosx + dθ
2 2θ) 2x) = 3/2
16 2
2θ 4.[(x+ C + 1)!2 − 1]3/2=
1
sin θ
1 sin θ,−1dxx=
35. dx = 1− 2 dx = x − 41 tan−1 + C 4. 16. √ dx =x = 4tan #
Z x + 16
2 ! 31. x 3+ 16 = θ+
1 4θ cos θ + C
sin
5.

x + 1 = sec θ, dx =
x2 16 − x2 1
= − !cot316
sec θ+C =−
θ tan
1

2x + 4 ! √ 2. √
x
dx
16 16
x = 2 1sec θ,−1 xdx + 3= 21sec θ tan
" 5.
7.5. TRIGONOMETRIC
x +θ 3 dθ! 7.
#
6.
TODO! figure
SUBSTITUTIONS =
3
"
4 cos3θ
!!sin2 θ =16sec−
33. dx = ln(x2 + 4x + 13) + C
√ 16
!2
4 − 9x2 x2 − 4
32. = tan + √ √ sec 20. θ tan +1
x2 − 1
θC4 dx =dx − = cot θ sec
x = 1sec θ, dx
+C θ
x2 + 4x + 13 36.
x
dx 3x = 2 sin θ, 3 dx! = 2 cos16θ 3dθ 2
33. 8 sec
16 6. x2 + 6x + 13 8.!
7.5 =
Trigonometric x2 + 6x +Substitutions
30.
(11
13 x
− 10x − x
x 2 )2! =

+ 3 tan
16 #
θ √[36 −2(x $
=2C810.(sec sec4θθ−dθ1) 1 sec tan
1 θ x+3 + x+3 9. 2 3/2 = θ16tan θ 2dθθ
Z ! $ 34. =
" √ = 16#tan−1 2(2 sec 8(xθ2tan7. !θ)√dθ +
+ 6x + 13)
1
= sec
−4θ
16 x +
−x
x 5 =
4 sec2 θ − 4 − x #√
1 1 x−3 =
4 − TODO
4 sin2figure
θ 2450!cos θ dθ!
1.
8. # x= 2
dx
−(sin11.
x = sin
−1 TODO
θ,
!+
$C
dx =
CHAPTER cos
= −3csc
θ dθ
1
figure = sin θ + C
! =θ2 + C
3 ! 7.= TE 1
3 6= c
x

34. dx = tan−1 +C 35.


2 !
3 sin θ
1 3
= 8 2 2(1 dx+ tan 2 1 2
sec 9.= 8 tan θ +
12. ! 1 θ)"
7.5. TRIGONOMETRIC
tan 117.
−3 sin 22 1
SUBSTITUTIONS
+ cos θθ,(36
!
3
4 + (x − 3)2 2 2
θ figure dx
30. = θ)
! [36 − (x + 5)2 ]2 θ dθ
dx ! = Trigonometric θ
TODO √ C
cosxθ2 dθ = x = tan dx
dθ = −= se
! 36.2 (11 − 10x!− x ) 13.
7.5 3 sin2 θ x 1 +Substitutions x! sin θ 2
2
cos θ 1 − sin2TODO θ 5 = 16 sinfigure 10. !√ 1 2 secSee θ Sec √
dθ = 2 "32.2 ! x(x+dθ = θ dθ 14.1.=! 1(csc = 1
θ, dx dx dx
Z   =2

Z 21 + 22x)3/23/2 !
37. = 6 cos − x2
θ θ−dθtan 1+ ="tan
x22 !θ − 1)
= sin= dxcot
= cos θ+ −12 θ
[(x + 1) 1]
dx x dθ
3/2 θ, − θ C
sin θ = 4 xsin−θ4 + ! 6(xcos θ− 4) 11. + C cos θ − 1
x2 16
15.
21
x ! 38. ! = 3 2x2 +2 dθ 4 = TODO figure =
1
sec
! "
= 3 θ dθ2
1 − sin θ
cos=θ dθ
ln =
=
!
cos
| csc θ216
2
θ
−2 cot
! se
=2+ C
35. = 1 dx = x − 4 tan +C
dθ θ|
−1 33. (36 − 236 sin θ) 12. ! dx =4+ ln(x

16.
+ 4x !+dx13)
co
=+
2
dx − = 2 TODO csc39.θ figure
dθ − 2 sin θ dθ = 2SeelnSection x| csc+θ7.3,
4x cot13
+ θ| +
216 cos
23xcos
θ 1 =216
θ + Csec θ,=TODO
sin θ
secCθ tan θ "
sin θ

θ − 1) dθ = − cot θ − θ +1C = −
dθ√1 −
x2 + 16 x2 + 16 4
17.
−Example 5!x
(csc2figure
! θ tan=θ dθ √x
2.
13. √ dx x = 2 sec ! θ, dx = 2 sec
432 11
x2 − 4 sec tan 1
sec
Z √ % 40. √
3.7.5.% 2TRIGONOMETRIC
% ! "
√ SUBSTITUTIONS
1 1
1θ + 14. 1
ln | =
#TODOθ
1 θ|2.2=!+!√−1
figureθ
18. √
−3 θxx32dθ 1 +=
dx x = θ
tan
! θ tan2θ dθ 2dθ =
θ,
!dx = s
4 − 9x2 %% 34.
x2
8x
4 − 9x2 = 4 − 9x sec
2 θ tan sec θ + tan Cx√
3
sec
% (sectan x −1) tan2 √θ ! sec2
#2 dx = 4 sec2 θ! − 48+
3/2 = 2 sec θ, dx = 2 sec
7.5.θ −
dx
(2 sec tan = sec8
% + 2 216 " 24 ++(xC− 3) 2 C θ θ) dθ θ
= 2 ln % 41.− 4
2
= SUBSTITUTIO
36. 3x = 2 sin θ, 3 dx = 2 cos θ dθ
TRIGONOMETRIC
dx % 3x 3x 12 x + 52 2 √ sec2 θθ− 4 (2 sec θ tan#tanθ) dθθ = 18 + 1sec tan4 θ
3
% 6 15. !
4. 2 =
=
x+
x 7.5 Trigonometric Substitutions
%
% 2 − √4 − 9x2 % $
%
3x
432

! 11 −210x − x$$16. 2

=11!−(sin" − θ)
− 10x x=
−1 (1 + tan2! θ)4 sec
2 8 + C= = −seccsc
2
θ dθθ==+−(sin
8C
8#(1 $+ tan2 θ) sec2 θ dθ = 8 172(11
θ)=
=tan
# −1 !θ ++ =ta−

−C1−tan
tan θ + 3
3 + 1cx
3

Z p x 1 6 " 16x + 5TODO $ = (1


2   "1 $figure
5.
= 2 ln %
% ! √ %
+ 35.
4 !
− 9x 2+C + ln
dx
$ √
$17.11= 1 2−
+ √ TODO
= 4 x − 4=!+4 √x2(x
2 figure dx
2+ C1 3/2
$− 4x−2=
+ 4)
x
(x − 4)
2
+
−3/2 16
C+4C tan −1
%
2

4 − 4 sin θ 2 1−x % 432 10x 11 10x 3


2 − − x − − x 1 3
% 3x 2x
x 2+4 16 cos $ x2 $$+
TODO 16 x4 dx x== sin1 θ,θ +
19.
figure

dx =1cos
si
= cos θ dθ 6. 1. dx 33. =x = sin 5 dx = dx 1 TODO θ$ dθfigure
ln=$$ √ln(x +
2 1131.
3. 4x$+ + C13) + C
x +θ, x+
+ ! " 16 2 16
x2 x2 −+
! √10x 4x−+ x2 )13 432 11 − 10x − x2 $ 1 − sin θ

3 sin θ
2 3 TODO figure
TODO figure !! "72(11

36. 1 − sin
4− 9x 2 3.
! 4. 32. !
= 14
= sin θtan−1 #
cos θxd

1 3x−1 5.= 2θsin θ, 3 dx = 21cos


7. 2
cos 2 16θ dθ 1
Z Z = 1 xθ cos4.dx x −dθ 3 = cot2=dθ
37.2 34. dx θ dθ= = tan 33.
+C − cot31θ + C = − x

cos2 θ 1 − sin θ
31. 6. 2 3
= tan−1 3
4 sin
+
2
(x θ− 3)2 5. 2 ! $sin θ2 !2 √ 2" 16#
8.
=2 dθ = 2 34.
4 − TODO 4#sin √ 21 dxx2 x = sec θ, dx = sec
7.
dθ 32. ! 20. θxx14−−
! 2 2 ! " = figure cos
sin θ sin θ 38. = (csc xθ − 1) dθ = − cot θ − θ9.16
6. 8.
+2 C! = −
35. θ dθ
tansinθ! x x +
−1
1 3 = −
!
3 sin θdx = xx− 4
33.
Z Z 9. 35. dx = 7. 1 − 210. 30. tan4−1sec θ tan+θ dθC
39. 34. x2 + 16 !x +36.216 (11 − 10x!− x2 )2 dxsec=θ 2 4[36 − # √(x
TODO figure cos θ 1=−sinsin
1
θ + Cθ
1 x
= 2 csc θ dθ − 2 sin θ dθ = 2 ln | csc θ − cot θ| + 2 cos θ + C ! √ 8.
10. 11.
x=+dθ
3

35. 4 − 9x 2 = 2 37. dθ = 2 3 35 =

40. ! 36. 3x ! = 213.sinsin


12.
θ
θ, 3 dx = sin θ !
! 2 cos θ dθ=
3 dx 9. TODO figure 6c
11. 2. 36. √ x x = 2 sec x θ, dx 10. = 2 sec θ tan14.θ dθ
38.
2 √ √ dx
!=$
(36 − 3

4 − 9x2 41. x2 − 4 2 csc 39.θ dθ 2 − " 2 sin θ # dθ = 2SeelnSec |c


4 − 9x 2
12.
37.
! 11. 4 − 4 sin θ ! 2
15.

= 2 ln − +2 +C 38.
= √
8 sec3 θ =
(212.sec θ tan 23θ)%%sin
%16. 40. √ cos
dθθ= 8 4 3− sec9x 4 %
θ%dθ √ 1 " 1
3x 3x 2 42.
13. 39. 4 sec θ − 4
2
=! 2 ln %
17.2
41. − !
θ2dθ %
%$+2 θ2 216 " 2
= 4 − 9x se2

! 13. cos %#3x


2
θ 3x 1 − sin % 12
dθ = 2 1
2 − √4 − 9x2 p 43. 40. =2 2 =
dθ √
= 8 (1 + tan2 θ) sec 14. θ dθ = sin 8%θ tan√ θ + tan3% θ θ + C 432 11

14.
41. ! % 2 − 4!−3 9x2 sin % $
= 2 ln + 4 − 9x + C 2
15. " 1
15.
= 2 =3/22csc
%
ln θ% dθ − 2 sin θ dθ
%
% += 24ln−| csc 9x2θ+−
3x 3
= 4 x2 − 4 + (x216.
% %
− 4) + C % 3x % x+
Z Z p % 2 √ √ = 72(11 − 1
p 16. TRIGONOMETRIC
7.5. TODO figure TODO figure17.
SUBSTITUTIONS % 4 − 9x2 %% 4 − 9x2
= 2 ln % − TODO figure% + 2 +C
37. 6x − x2 dx = 9 − (x − 3)2 dx x − 3 = 3 sin θ,17.dx3.= 3 cos θ dθ 37. % 3x 3x % 2
7.5 Trigonometric 3
Substitutions % √
31. %
Z p Z x – 3 %% 2 − 32. 4 − 9x2 %% $
!4. √ 38. = 2 ln % % + 4 − 9x2 + C
% 3x %
= 9 − 9 sin θ 3 cos θ dθ = 9 cos θ dθ 1.
2 2 1 − x 2
dx x =!sin θ, dx = cos θ dθ ! $
33.
5. x2 39. $ 34.
Z   !37." 6x − x2 dx = ! 9 −2 (x − 3)2!dx x − 3 = 3 sin θ
9 9 1 6. =
1 − sin2 θ
40. 2
cos θ35.
cos θ dθ =! $ 2 dθ = cot2 dθ !
= (1 + cos 2θ) dθ = θ + sin 2θ + C sin θ sin 36.
9 − 9θ sin2 θ √
2 2 2 7. !
41.2
=
37.
3 cos θ dθ = 9 cos2 θ d
1 − x2"
  √= (csc θ − 1) dθ = − cot9θ!− θ + C = − − sin−1 x + # C
9 9 9 x − 3 9 8. x − 3 6x − x 2 = (1 + 38. 2θ) dθ =
cos x 9 θ + 1 sin 2θ +
= θ + sin θ cos θ + C = sin −1
+ TODO figure 42. +C 2 2 2
39.
2 2 2 3 2 !9. 3 3 9 x−40.3
"
9 x−3
#&
(x −
x3 43. = sin−1 + 1−
9 p
−1 x − 3 1 2. 10. √ dx x = 2 sec θ, dx = 2 sec2θ tan θ dθ341. 2 3 9
= sin + (x − 3) 6x − x + C 2 x 2−4
9 x − 3! 3

9 − 9 + 6x −
2 3 2 11.
!
8 sec θ
3
= sin −1
+ (x 4 − 3)
Z Z = √ 2 θ) dθ =38 sec
(2 sec θ tan 2 θ dθ 3
1 1 4 sec2 θ − 4
12. ! 9 # x − 3 1 $ $
38. dx = u = x − 3, du = dx = sin −1
√ p dx +
1 (x − 3) 6x − x2
= 8 (1 + tan2 θ) sec2 θ dθ 2= 8 tan 3θ + tan 2 3θ +C
6x − x 2 9 − (x − 3) 2
13. 38.
3
Z " 1
1 u x − 3
14. = 4 x2 − 4 + (x2 − 4)3/2 + C
= √ du = sin−1 + C = sin−1 +C 3
32 − u2 3 315.
TODO figure

3. 16.

4. 17.

5.

6.

7.

8.

9.

10.

11.
452 CHAPTER 7. TECHNIQUES OF INTEGRATION

Z 1 p
39. 4 − x2 dx x = 2 sin θ, dx = 2 cos θ dθ
−1
Z π/6 p Z π/6
= 4 − 4 sin θ 2 cos θ dθ = 4
2
cos2 θ dθ
−π/6 −π/6
Z π/6 iπ/6
= (2 + 2 cos 2θ) dθ = (2θ + sin 2θ)
−π/6 −π/6
" √ ! √ !# √
π 3 π 3 2π + 3 3
= + − − − =
3 2 3 2 3

Z √ Z
3
x2 π/3
4 sin2 θ
40. √ dx x = 2 sin θ, dx = 2 cos θ dθ = p 2 cos θ dθ
−1 4 − x2 −π/6 4 − 4 sin2 θ
Z π/3 Z π/3 iπ/3
=4 sin θ dθ =
2
(2 − 2 cos 2θ) dθ = (2θ − sin 2θ)
−π/6 −π/6 −π/6
" √ ! √ !#
2π 3 π 3 √
= − − − + =π− 3
3 2 3 2
Z 5
1
41. dx x = 5 tan θ, dx = 5 sec2 θ dθ
0 (x2 + 25)3/2
Z Z Z
π/4
5 sec2 θ 1 π/4
1 1 π/4
= dθ = dθ = cos θ dθ
0 (25 tan θ + 25)
2 3/2 25 0 sec θ 25 0
π/4 √
1 2
= sin θ =
25 0 50
Z 2
1
42. √
√ dx x = sec θ, dx = sec θ tan θ dθ
2 x3 x2 − 1
Z Z π/3 Z π/3
π/3
sec θ tan θ tan θ
= √ dθ = dθ = cos2 θ dθ
π/4 sec 3 θ sec2 θ − 1
π/4 sec 2 θ tan θ
π/4
Z π/3  π/3
1 1 1
= (1 + cos 2θ) dθ = θ + sin2 θ
2 π/4 2 2 π/4
" √ !  # √
1 π 3 π 1 π+3 3−6
= + − + =
2 3 4 4 2 24
Z 6/5
16
43. √ dx x = 2 sin θ, dx = 2 cos θ dθ
1 x4 4 − x2
Z sin−1 (3/5) Z sin−1 (3/5)
32 cos θ 2 cos θ
= p dθ = dθ
π/6 16 sin θ 4 − 4 sin θ
4 2
π/6 sin4
(2 cos θ)
Z sin−1 (3/5) Z sin−1 (3/5)
= csc2 θ csc2 θ dθ = (1 + cot2 θ) csc2 θ dθ
π/6 π/6
7.5. TRIGONOMETRIC SUBSTITUTIONS 453

Z sin−1 (3/5) Z sin−1 (3/5)


= csc θ dθ +
2
cot2 θ csc2 θ dθ
π/6 π/6
isin−1 (3/5) sin−1 (3/5)
1
= − cot θ − cot θ
3
π/6 3 π/6
√ 1 √
5
= −[cot(sin−1 3/5) − 3] − [cot3 (sin−1 3/5) − 3 3] 3
   3 
4 √ 1 64 √ √ 172 4
=− − 3 − −3 3 =2 3−
3 3 27 81
Z 1/2
44. x3 (1 + x2 )−1/2 dx x = tan θ, dx = sec2 θ dθ
0
Z 452 CHAPTER 7. TECHNIQUES OF INTEGRATION
tan−1 (1/2)
= tan θ(1 + tan
3 2
θ)!−1/2
1/2 sec2 θ dθ
0 44. x3 (1 + x2 )−1/2 dx x = tan θ, dx = sec2 θ dθ
0
Z −1 Z −1 ! tan−1 (1/2)
tan (1/2)
tan3 θ sec2 θ tan (1/2)
tan=2 θ0 tan θ sec
tan θ(1 + tan θ)−1/2 sec2 θ dθ
3 2
= dθ = θ dθ
0 sec θ 0 ! tan−1 (1/2)
tan3 θ sec2 θ
! tan−1 (1/2)

Z = dθ = tan2 θ tan θ sec θ dθ


tan−1 (1/2) 0 sec θ 0
!
= (sec2 θ − 1) tan θ sec θ dθ =
tan−1 (1/2)
(sec2 θ − 1) tan θ sec θ dθ
0 0
 tan−1 (1/2) " #$tan−1 (1/2)
1
1 =
3
sec3 θ − sec θ
= sec3 θ − sec θ 0
3 0 =
1
[sec3 (tan−1 1/2) − 1] − [sec(tan−1 1/2) − 1]
3% & %√ &
1 √ √
= [sec3 (tan−1 1/2) − 1] − [sec(tan−1 1/2) − 1]= 31 5 8 5 − 1 − 25 − 1 =1 16 −247 5
3 ! ! TODO figure √
√ √ 2
1 5 5 5 16 − 7 5
= −1 − −1 =45.
3 8 2 46.
24
47. 7.5. TRIGONOMETRIC SUBSTITUTIONS
Z
1 48. 1 7.5. TRIGONOMETRIC SUBSTITUTIONS
45. x2 sin−1 x dx u = sin−1 x, du = √ dx; dv = x2 dx, = x3
7.5 v Trigonometric 1
Substitutions
x
1−x ! √ 3 2 7.5
2 49.
Z 50.
1−x Trigonometric Substitution
1. dx x = sin θ, dx = cos θ dθ
1 3 −1 1 x3 x2 ! √
= x sin x − √ dx 51.x = sin θ, dx = cos θ dθ ! " 1 − x2
1 − sin2 θ
!
xcos
2 !
3 3 1 − x2 52. = 1. dx
2 x2 cos θ dθ =
= sin
θ θ, dx =2 cos θ dθ
dθ = cot dθ
Z Z sin θ sin θ
"
2

1 3 −1 1 sin3 θ 53. 1 3 −1 1 = ! (csc32 θ − 1) dθ = − cot θ!− θ +1 C−=sin−2 θ1 − x2 − sin−1!x +c
= x sin x − p cos θ dθ = x sin x − sin θ dθ = cos θ dθ =
3 3 1 − sin2 θ 54. 3 3
TODO figure
sin2 θ x
s
Z !
55.
1 3 −1 1 !
x3 = (csc2 θ − 1) dθ = − cot θ − θ
= x sin x − (1 − cos2 θ) sin θ56.dθ 2. √ dx x = 2 sec θ, dx = 2 sec θ tan θ dθ
3 3 x −4
2
! TODO figure !
 
57. 8 sec3 θ
1 3 −1 1 1 = √! (2 sec θ tan θ) dθ = 8 sec4 θ dθ
= x sin x − − cos θ + cos θ + C
3 58.
2.!
4 sec 2θ−
x 34
x#= 2 sec1θ, dx $ = 2 sec θ tan
3 3 3 59. = 8 (1 + tan
√ 2
x2 θ)
dx
4 2 θ dθ = 8 tan θ + tan3 θ + C
− sec
p ! 3
1 3 −1 1 1 60. " 8 sec θ
3
= x sin x + 1 − x2 − (1 − x2 )3/2 + C 1
= 4 x2 − 4 + (x2 − 4)= 3/2
+ C√ (2 sec θ tan θ)
3 3 9 61. 3
!
4 sec2 θ − 4
#
TODO figure
62.
=8 (1 + tan2 θ) sec2 θ dθ = 8
3.
" 1
4. =4 x2 − 4 + (x2 − 4)3/2 + C
3
5. TODO figure
6.
3.
7.
4.
8.

9. 5.

10. 6.
11. 7.
12.
8.
13.
9.
14.
10.
454 CHAPTER 7. TECHNIQUES OF INTEGRATION
7.5. TRIGONOMETRIC SUBSTITUTIONS
Z 7.5. TRIGONOMETRIC SUBSTITUTIO
1 1 1
46. x cos−1 x dx u = cos−1 x, du = − √ dx; dv = x 7.5dx, v Trigonometric
= x2 Substitutions
x
1 − x2 ! √ 2 2 7.5 Trigonometric Substi
1−x
Z 1. dx x = sin θ, dx = cos θ dθ
x2 ! √
1 2 1 x2 ! " 1 2−θ x2 ! !
= x cos x +
−1
√ dx x = sin θ, dx = cos θ dθ= 1. 1 − sin dx xcos
2
= sin
θ θ, dx =2
2 2 1−x 2
sin2 θx
2 cos θ dθ =
sin 2
θ
dθ = cot
Z Z! ! " √
2 1−x
1 2 1 sin2 θ 1 2 1 = (csc22 θ − 1) dθ = − cot = θ − θ +1 C−=sin− θ
cos
= x cos x +
−1
p cos θ dθ = x cos x +−1
sin θ dθ sin 2
θ x
2 2 1 − sin2 θ 2 2
TODO figure !
Z !   = (csc2 θ − 1) dθ =
1 2 1 1 2 −1 2. 1√ x3 dx1 x = 2 sec θ, dx = 2 sec θ tan θ dθ
= x cos x +
−1
(1 − cos 2θ) dθ = x cos x + x2 −θ 4− sin 2θ + C
2 4 2 4 2 ! TODO 8 sec3 θ
figure !
= √! (2 sec θ tan θ) dθ = 8 sec4 θ dθ
1 2 1 1 4 sec θx− 4
2 3
= x cos−1 x + sin−1 x − sin θ cos θ + C 2.! √ 2 dx x#= 2 sec1θ, dx $ =
2 4 4 = 8 (1 + tan x2 θ) 4 2 θ dθ = 8 tan θ + tan3 θ
− sec
3
!
" 1 8 sec θ 3

Z √ =24 x2 − 4 + (x2 − 4)=


3/2
+ C√ (2
√ √ 3 4 sec2 θ − 4
3
1 !
47. A = √ dx x = 3 tan θ, dx = 3 sec2 θ dθ TODO figure
=8 (1 + tan2 θ) sec
1 x 3 + x2 3. 1
Z π/4 √ "
3 sec2 θ 4.
1
=4 x2 − 4 + (x2 −
= √ √ dθ 3
π/6 3 tan θ 3 + 3 tan2 θ 5. TODO
1 figure
2
Z π/4 Z π/4
1 sec θ 1 6.
3.
=√ dθ = √ csc θ dθ
3 π/6 tan θ 3 π/6 7.
4.
π/4 8. √
1 1 √ √ 1 2 − 1 5.
= √ ln | csc θ − cot θ| = √ (ln | 2 − 1| − ln |2 − 3|) = 9.√ ln √ ≈ 0.2515
3 π/6 3 3 2− 3
10. 6.

Z Z 11. 7.
1 p 1 p 1
48. A = x5 1 − x2 dx = x4 1 − x2 x dx 12.
8.
0 0 13.
9.
u = 1 − x2 , x2 = 1 − u, 2x dx = −du 14.
Z 0   Z 0 15.
10.
1 1 1
= (1 − u)2 u1/2 − du = − (u1/2 − 2u3/2 + u5/2 ) du16. 11.
1 2 2 1
 0   17. 12.
1 2 3/2 4 5/2 2 7/2 1 16 8
=− u − u + u =− 0− = ≈ 0.0762 13.
2 3 5 7 1 2 105 105
14.

49. We find the area in the first quadrant and use symmetry.
a
15.
Z ap
16.
A=4 a2 − x2 dx x = a sin θ, dx = a cos θ dθ
0 17.
Z π/2 p Z π/2
a
=4 a2 − a2 sin2 θ a cos θ dθ = 4a2 cos2 θ dθ
0 0
Z  π/2 π
π/2
1
= 2a 2
(1 + cos 2θ) dθ = 2a 2
θ + sin 2θ = 2a2 = πa2
0 2 0 2
a

50. We find the area in the first quadrant and use symmetry.

b
7.5. TRIGONOMETRIC SUBSTITUTIONS 455
7.5. TRIGONOMETRIC SUBSTITUTIONS
Z
ap 2
b
! √ 7.5. TRIGON

A=4 b − x2 dx x = b sin θ, dx = b cos θ dθ 1.


1 − x2
dx x = sin θ, dx = cos θ dθ 7.5 Trig
0 b x2
! √
! " ! !
Z π/2 p Z π/2 1 − sin2 θ cos2 θ 1.
1−

4a
x 2
= cos θ dθ = dθ = cot 2

sin θ
2
sin θ
2
= b2 − b2 sin2 θ b cos θ dθ = 4ab cos2 θ dθ ! √
1−x
b 0 0 = (csc2 θ − 1) dθ = − cot θ − θ + C = −
2
− sin−
x
Z π/2  π/2   ! TODO fi
1
7.5. TRIGONOMET
π x3 !
= 2ab (1 + cos 2θ) dθ = 2ab θ + sin 2θ = 2ab + 0 =2.πab√x2 − 4 dx x = 2 sec θ, dx = 2 sec θ tan θ dθ
3
x
2.
7.5 Trigono √

2 2
x − 2
! ! ! √
0 0 8 sec θ
3
41.
1 − x 2

= √ (2 sec θ tan θ) dθ = 8 sec θ dθ x2


dx
4 sec2 θ − 4
! # $
1
= 8 (1 + tan2 θ) sec2 θ dθ = 8 tan θ + tan3 θ + C
3
" TODO fi
1 TODO figure
= 4 x2 − 4 + (x2 − 4)3/2 + C ! 3. 3
√ 3
Z √ √ ! 2. √
x
4.x2 − 4
dx
3
1 3.
1 2 x % " &
dx = cosh−1 + C = ln x + x2 − 362 + C, x 5.> 6
51. V = π dx x = 3 tan θ, dx = 3 sec2 θ dθ √
x − 36
2 6
0 +x )x2 (3
2
Alternatively, the substitution x = 6 sec θ could have been used.
6.

Z π/4 √ √ Z ! "
1 7.

3 sec2 θ π 3 π/4 1 4. 3 − x2 dx
√ √
x = 3 sin θ, dx = 3 cos θ dθ
8.

=π dθ = 2 dθ ! " !
TODO figure
!9.

π/6 3 tan θ(3 + 3 tan θ)


2 2 9 π/6 tan θ
√ 3.
= 1 3 − 3 2sin2 θ 3 cos θ dθ = 3 cos2 θ dθ = 310.
1 + co
4. 2
√ Z √ Z π/4 √ # π/4 3
#
1
$
3 3 5.
11.

π 3 π/4 π 3 π 3 =
2
θ + sin 2θ + C = θ + sin θ cos θ + C 12.
2 2 2
= cot2 θ dθ = (csc2 θ − 1) dθ = (− cot θ − θ)
6.
13.
# $√
9 π/6 9 π/6 9 3 −1 x
= sin √ +
3

x 3 − x2

3 7. −114. x
+ C = sin8.
√ +
x
π/6 2 3 2 3 3 2 15. 3 2
√ √
π 3 h π  √ π i π 3 √ π
9.
16.
5.
10.
=− 1+ − 3+ = 3−1− ≈ 0.2843 6. 17.

9 4 6 9 12 11.

12.
7.
52. Using the disk method, 13.
1

8. 14.
Z 2
16 9.
15.

V =π dx x = 2 tan θ, dx = 2 sec2 θ dθ 16.

0 (4 + x2 )2 10.
1 2
17.

Z π/4 Z π/4 Z π/4 11.


16 sec2 θ
=π (2 sec2
θ dθ) = 2π dθ = 2π cos
12.
2
θ dθ
0 (4 + 4 tan2 θ)2 0 sec4 θ 0
13.
Z π/4  π/4  
1 π 1 + 2π
π 2 14.
=π (1 + cos 2θ) dθ = π θ + sin 2θ =π + = .
0 2 0 4 2 4
53. Using the shell method,
Z 2 p
V = 2π x2 4 + x2 dx x = 2 tan θ, dx = 2 sec2 θ dθ
0
Z π/4 p Z π/4 5

= 2π 4 tan2 θ 4 + 4 tan2 θ 2 sec2 θ dθ = 32π tan2 θ sec3 θ dθ


0 0
Z π/4 Z π/4 Z π/4
= 32π (sec2 θ − 1) sec3 θ dθ = 32π sec5 θ dθ − 32π sec3 θ dθ.
0 0 0
From Section 7.3, Example 5 we obtain
Z π/4  π/4
1 1
sec θ dθ =
3
sec θ tan θ + ln | sec θ + tan θ|
0 2 2 0
1 √ 1 √
= ( 2)(1) + ln( 2 + 1)
2 2 1 2
1√ √
= [ 2 + ln( 2 + 1)].
2
456 CHAPTER 7. TECHNIQUES OF INTEGRATION

Z π/4
To find sec5 θ dθ we use integration by parts.
0
Z π/4 Z π/4
sec5 θ dθ = sec3 θ sec2 θ dθ
0 0

u = sec3 θ, du = 3 sec2 θ sec θ tan θ dθ; dv = sec2 θ dθ, v = tan θ


iπ/4 Z π/4
= sec θ tan θ
3
−3 sec3 θ tan2 θ dθ
0 0
√ Z π/4
=2 2−3 sec3 θ(sec2 θ − 1) dθ
0
√ Z π/4 Z π/4
=2 2−3 sec5 θ dθ + 3 sec3 θ dθ
0 0
√ Z
3√ π/4 √
sec5 θ dθ + [ 2 + ln( 2 + 1)]
=2 2−3
0 2
Z π/4 Z π/4 √
2 3√ √
Solving for sec θ dθ we obtain
5
sec θ dθ =
5
+ [ 2 + ln( 2 + 1)]. Then
0 0 2 8
(√ ) "√ #
2 3√ √ 2 1 √
V = 32π + [ 2 + ln( 2 + 1)] − 32π + ln( 2 + 1)
2 8 2 2
√ √
= 12π 2 − 4π ln( 2 + 1).

Z 1
x2
54. V = 2π √ dx x = 2 sin θ, dx = 2 cos θ dθ 1
0 4 − x2
Z π/6 Z π/6
4 sin2 θ
= 2π p 2 cos θ dθ = 8π sin2 θ dθ
0 4 − 4 sin2 θ 0
Z π/6  π/6 1
1
= 4π (1 − cos 2θ) dθ = 4π θ − sin 2θ
0 2 0
" √ ! # √
π 3 2π − 3π 3
2
= 4π − −0 = ≈ 1.1383
6 4 3

55. y 0 = 1/x
Z √3 p Z √3 √ 2
x +1
L= 1 + (1/x)2 dx = dx x = tan θ, dx = sec2 θ dθ
1 1 x
Z π/3 √ 2 Z π/3 Z π/3
tan θ + 1 sec3 θ sec θ
= sec θ dθ =
2
dθ = sec2 θ dθ
π/4 tan θ π/4 tan θ π/4 tan θ
Z π/3 Z π/3   Z π/3
sin θ
= csc θ(tan2 θ + 1) dθ = + csc θ dθ = (sec θ tan θ + csc θ) dθ
π/4 π/4 cos2 θ π/4
iπ/3  
2 1 √ √
= (sec θ + ln | csc θ − cot θ|)
= 2 + ln √ − √ − ( 2 + ln | 2 − 1|)
π/4 3 3
7.5. TRIGONOMETRIC SUBSTITUTIONS 457
√ √ √
=2− 2 − ln( 6 − 3) ≈ 0.9179

56. y 0 = −x + 2
Z 2p 456 CHAPTER 7. TECHNIQUES OF INTE
L= 1 + (2 − x)2 dx 2 − x = tan θ, −dx = sec2 θ dθ
1 56. y ! = −x + 2
Z 0 p Z 0 ! 2"
= 1 + tan2 θ(− sec2 θ dθ) = sec3 θ dθ L = See Section
1 + (2 −7.3, Example
x)2 dx 2 − x5= tan θ, −dx = sec2 θ dθ
π/4 π/4 1
! 0 " ! 0
 π/4    
1 1 1 √= 11 + tan√ 2 θ(− sec2 θ dθ) = 1 sec3 θ dθ See Section 7.3, Example
= sec θ tan θ + ln | sec θ + tan θ| = 2(1)π/4+ ln | 2 + 1| − 0 − π/4ln 1
2 2 0 2 # 2 2 $% π/4 & % #
√ 1 1 1√ 1 √
2 1 √ = sec θ tan θ + ln | sec θ + tan θ| = 2(1) + ln | 2 + 1| − 0
2 2 2 2
= + ln( 2 + 1) ≈ 1.1478 √ 0
2 2 2 1 √
= + ln( 2 + 1) ≈ 1.1478
√ 2 27.5. TRIGONOMETRIC SUBSTITUTIONS
a2 − x2 dy
57. (a) The slope at (x, y) is − , which is also . 7.5 Trigonometric √ Substitutions
x dx a2 − x2
Z √ 2 2
Z57. (a) The slope at !(x,√y) is 2− , which is also
dy
.
a −x 1−x x dx
(b) Separating variables, dx = − dy. Now 1. dx x = sin θ, dx = cos θ dθ
x x 2
! √ 2 " 2 a !
Z √ 2 a! − x 1 − dx
sin2=
!
cos2 θ
!
2 (b) Separating variables, = x
θ− dy.
cos θ dθ = Now 2 dθ = cot2 dθ
a −x
x = a sin θ, dx = a cos θ dθ sin θ sin θ
2
dx ! √
x TODO figure, with theta in the right x
place 1 − x2
Z √ Z! √ Z = (csc 2
θ − 1) dθ = − cot θ − θ + C = − − sin−1 x
x
a2 − a2 sin θ cos 22
a − 2
θ x TODO 1 − sin 2
θ
x = a sin θ, dxdθ= a cos θ dθ
= a cos θ dθ = a dθdx = a figure
a sin θ sin xθ ! x!3 √ sin θ ! !
Z Z 2. √ dxa2 − ax2 = sin2 sec
θ θ, dx = 2 sec θ tan θcos dθ θ
2
1 − sin
=
x2 − 4 a cos θ dθ = a dθ = a
= a csc θ dθ − a sin θ dθ = a ln | csc θ − cot θ| + a cosaθsin !+θC sin θ! sin θ
! 8 sec
! θ3

! = √ (2 sec θ tan θ) dθ = 8 sec4 θ dθ


a √ √ = a csc θ dθ ! −a
4 sec 2θ−4
sin θ dθ = a ln |#csc θ − cot θ| +$a cos θ +
a2 − x2 a2 − x2 )θ + 1 tan3 θ + C
= a ln − +a + C. ' = 8√ (1 + tan2'θ) sec(
'a
2
θ√dθ = 8 tan
x x a ' a2 − x2 '' a2 − x2 3
= a ln ' − " '1+ 2a + C.
' '
a − √a2 − x2 √ x = 4 x 2x− 4 + (x − 4) 3/2 a
+ C
3
' √ figure ''
Then a ln + a2 − x2 = −y + C1 . Now ''y(10) a − a2=− 0
TODO
x2 'and√a2 = 210, so
x Then a ln ' ' + a − x = −y + C1 . Now y(10) = 0 and
√ 3.
' x '
10 − 100 − 100 √ ' √
' 10 −4. 100 − 100 ' √
'

10 ln
10 + 100 − 100 = 0 + C1 and 10 lnC1' = 0. Thus ' + 100 − 100 = 0 + C1 and C1 = 0. Thus
' 10 '
5.

10 − √100 − x2 p 6. '
' 10 − √100 − x2 ' "
'
' '
y = −10 ln − 1007.− x2 . y = −10 ln '
'
' − 100 − x2 .
'
x x
8.
Note: the
Note: If the substitution y = a cos θ is used, we obtain If the substitution
equivalent
9. y = a cos θ is used, we obtain the equivalent soluti
solution
10. ' '
10 − √100 − x2 p ' 10 − √100 − x2 ' "
' '
11. y = 10 ln ' ' − 100 − x2 .
y = 10 ln − 100 − x . 2
' x '
x 12.

13.
58. Using symmetry with respect to the x-axis, we have
58. Using symmetry with respect to the x-axis, we have 14.

15.

16.

17.
458 CHAPTER 7. TECHNIQUES OF INTEGRATION
Z a+r p
V = 4π x r2 − (x − a)2 dx
a−r r

x − a = r sin θ, dx = r cos θ dθ
Z π/2 a–r a+r
p
= 4π (a + r sin θ) r2 − r2 sin2 θ r cos θ dθ
−π/2
Z Z Z π/2
π/2
1 + cos 2θ π/2
= 4πr 2
(a + r sin θ) cos θ dθ = 4πar 2 2
dθ + 4πr 3
cos2 θ sin θ dθ
−π/2 −π/2 2 −π/2
 π/2  π/2
1 1
= 2πar θ + sin 2θ
2
+ 4πr − cos θ
3 3
2 −π/2 3 −π/2
h π   π i 4
= 2πar2 + 0 − − + 0 − πr3 (0 − 0) = 2aπ 2 r2
2 2 3
Z 2 r Z 2p Z 2p
2−x 7.5. TRIGONOMETRIC SUBSTITUTIONS
59. F = 62.4 x dx = 62.4 2x − x2 dx = 62.4 1 − (x − 1)2 dx
1 x 1 1 ! √
1−x 2
7.5. TRIG

1. dx x = sin θ, dx = cos θ dθ 7.5 T


x2
x − 1 = sin θ, dx = cos θ dθ ! "
1 − sin2 θ
!
cos2 θ
! 1.
! √

= cos θ dθ = dθ = cot2 dθ
Z π/2 p Z π/2 Z π/2 !
sin θ
2
sin θ
2

1 − x2
= 62.4 1 − sin2 θ cos θ dθ = 62.4 cos2 θ dθ = 31.2 (1 !+ cos 2θ) dθ = (csc2 θ − 1) dθ = − cot θ − θ + C = −
x
− si
TOD
7.5. TRIGONOM
0 0 0 x3 !
2. √ dx x = 2 sec θ, dx = 2 sec θ tan θ dθ
 π/2 π x2 − 4
2.
7.5 Trigo √

1 !
8 sec3 θ
! ! √
1 − x2

= 31.2 θ + sin 2θ = 31.2 = 15.6π ≈ 49.0088 lb = √


4 sec2 θ − 4
(2 sec θ tan θ) dθ = 8 sec41.θ dθ x2
2 0 2 ! #
1
= 8 (1 + tan2 θ) sec2 θ dθ = 8 tan θ + tan3 θ + C
$

√ 3
Z " 1
TOD
3
1 1 = 4 x2 − 4 + (x2 − 4)3/2 + C
TODO figu
! 3. 3
60. A= √ dx x = tan θ, dx = sec2 θ dθ !
3
% &
2. √
x
4.x2 − 4

1+x 2 1 x "
0 3. √ dx = cosh−1 + C = ln x + x2 − 362 + C, x 5.> 6
x2 − 36 6
Z π/3 Z π/3 6.

sec2 θ Alternatively, the substitution x = 6 sec θ could have been used.


7.

= √ dθ = sec θ dθ 4.
! "
3 − x2 dx 1
√ √
x = 3 sin2θ, dx = 3 cos θ dθ
8.

0 1 + tan2 θ 0 ! " √ !
TODO figu
!9.
1+
iπ/3 √ =
3.
3 − 3 sin2 θ 3 cos θ dθ = 3 cos2 θ dθ = 310.
4.

= ln | sec θ + tan θ| = ln(2 + 3)


11.
# $
3 1 3 3 5.
0 = θ + sin 2θ + C = θ + sin θ cos θ + C 12.
2 2 2 2 6.
13.
√ √ $√
Z Z √3  3 3 x 3
#
x 3−x 2 3 7. −114. x
3
x 1 1 (1 + x2 )1/2 = sin−1 √ +
2 3 2

3

3
+ C = sin
2 8.
√ +
15. 3
My = √ dx = 2x(1 + x2 )−1/2 dx = · 9.

2 0 2 1/2
16.
5.
0 1 + x2 0
10.
17.
√ 6. 11.
p i 3 12.
7.
= 1 + x2 =2−1=1 13.

0 8. 14.

Z √ √3 9.
15.

1 13
1 π 16.

Mx = dx = tan−1 x = 10. 17.

2 0 1+x 2 2 0 6 11.

1 π/6 π 12.
x= √ ≈ 0.76; y = √ = √ ≈ 0.40 13.
ln(2 + 3) ln(2 + 3) 6 ln(2 + 3)
14.
Z
1
61. (a) √ dx ex = sec θ, ex dx = tan θ sec θ dθ, dx = tan θ dθ
e2x − 1
Z Z Z
tan θ tan θ
= √ dθ = dθ = dθ = θ + C = sec−1 ex + C
sec2 θ − 1 tan θ
7.5. TRIGONOMETRIC SUBSTITUTIONS 459
Z p 7.5. TRIGONOMETRIC SUBSTITUTIONS
458 CHAPTER 7.
(b) e2x − 1 dx ex = sec θ, ex dx = tan θ sec θ dθ, dx = tan θ dθ ! "
ex
7.5 Trigonometric Substitutions
Z p Z (b) e2x − 1 dx ex = sec θ, ex dx = tan θ sec θ
! √ ! " !
= sec θ − 1 tan θ dθ = tan θ dθ1.
2 2 1 − x2
dx x = sin θ, dx = cos θ dθ = sec2 θ − 1 tan θ dθ = tan2
x2
Z p ! " 1 ! ! "
1 − sin2 θ cos2=θ tan θ − θ + 2C = e2x − 1 − sec−
=
= (sec θ − 1) dθ = tan θ − θ + C = e2x − 1 − sec−1 esin
2 x2
+θ Ccos θ dθ = sin2 θ dθ = √ cot dθ
! 62.
1 − x2
= (csc2 θ − 1) dθ = − cot θ − θ + C = − − sin−1 x + C
x
62. The circle of radius a, which is centered on the origin, is definedTODO
by x2figure
+y 2 = 7.6 Partial Fractions
a2 . Let c be the distance between the centers of the two circles. ! The circle b
x =c2 sec θ, dx = 2 sec θ tan θ dθ
x3 1.
of radius b is thus defined by x √
2
+ (y − c)2
= b2
. The
√ area is
2. the
√ integral
x2 − 4
dx

of the difference between y = b2 − x2 + c and y = a2 − x2 from −b to ! 2.


8 sec3 θ
a !
= √ (2 sec θ tan θ) dθ = 8 sec4 θ dθ
b. Using symmetry, we have !
4 sec3.2 θ − 4
# $
4. 1
Z b p Z = 8 (1 + tan2 θ) sec2 θ dθ = 8 tan θ + tan3 θ + C
p b p p 3
" 2 5. 1
A=2 [( b2 − x2 + c) − a2 − x2 ] dx = 2 ( b2 − x2 − a=2 4− xx2 −+ 4 +c)(x dx2
− 4)3/2 + C
0 0 6. 3
TODO figure
7.
By using the substitution x = r sin θ, dx = r cos θ dθ , we find 3.
Z p Z p Z 8.
4.
r2 − x2 dx = r2 − r2 sin θ r cos θ dθ = r2 cos2 θ 5.dθ
2 9.

  6. 10.
Z
1 + cos 2θ r2 1 r2 r2
= r2 dθ = θ + sin 2θ +7.C = θ + sin θ11.cos θ + C
2 2 2 2 2 12.
2  
√ 8.
r2 x r x r 2 − x2 r 2
x 1 p 13.
= sin−1 + = sin−1
9. + x r2 − x2 + C.
2 r 2 r r 2 r 2 14.
Substituting b and a respectively, we get 10.
15.
Z bp Z bp Z b 11.
16.
A=2 b2 − x2 dx − 2 a2 − x2 dx + 2 c dx 12. 17.
0 0 0 13.
 x p ib  x p ib ib 18.
= b sin 2 −1
+ x b2 − x2 − a2 sin−1 + x a2 14. − x2 + 2cx 19.
b 0
a 15.
0
 0

h p  i b p 20.
= b2 sin−1 1 + b b2 − b2 − (0 + 0) − a2 sin−1 16. + b a2 − b2 − (021.+ 0) + 2bc
a
17. 22.
πb 2
b p
= − a2 sin−1 − b a2 − b2 + 2bc.
2 a

From the figure, it can be seen that a2 = b2 + c2 or c = a2 − b2 . We substitute to simplify
further:
πb2 b p πb2 b
A= − a2 sin−1 − b a2 − b2 + 2bc = − a2 sin−1 − bc + 2bc
2 a 2 a
πb2 p
−1 b
= − a sin
2
+ b a2 − b2 .
2 a
The special-case lune of Hippocrates specifies a lune where the triangle formed by the origin

2
and the intersections of the two circles is a right isosceles triangle. For this lune, b = a
2
b π
by the Pythagorean theorem and sin−1 is . Substituting these values above yields the
a 4
460 CHAPTER 7. TECHNIQUES OF INTEGRATION

well-known result that the area of the lune of Hippocrates is the same as the area of the right
1
isosceles triangle that defines it, or a2 .
2

7.6 Partial Fractions


x−1 x−1 A B
1. Write = = + .
x +x
2 x(x + 1) x x+1
9x − 8 A B
2. Write = + .
(x − 3)(2x − 5) x − 3 2x − 5
x3 A B C D
3. Write = + + + .
(x − 1)(x + 2) 3 x − 1 x + 2 (x + 2) 2 (x + 2)3
2x2 − 3 2x2 − 3 A B C
4. Write = = + 2+ .
x + 6x
3 2 x (x + 6)
2 x x x+6
4 A B C Dx + E
5. Write = + 2+ 3+ 2 .
x3 (x2 + 3) x x x x +3
−x2 + 3x + 7 A B Cx + D
6. Write = + + 2 .
(x + 2)2 (x2 + x + 1) x + 2 (x + 2)2 x +x+1
2x3 − x Ax + B Cx + D
7. Write = 2 + 2 .
(x + 9)
2 2 x +9 (x + 9)2
3x2 − x + 4 3x2 − x + 4
8. = .
x4 + 2x3 + x x(x3 + 2x2 + 1)
This expression does not fall under any of the four partial fraction decomposition cases covered
in Section 7.6.
1 A B
9. Write = + . Then 1 = A(x − 2) + Bx.
x(x − 2) x x−2
Setting x = 0 and x = 2 gives A = −1/2 and B = 1/2. Thus
Z Z Z
1 1 1 1 1
dx = − dx + dx
x(x − 2) 2 x 2 x−2

1 1 1 x − 2
= − ln |x| + ln |x − 2| + C = ln + C.
2 2 2 x

1 A B
10. Write = + . Then 1 = A(2x + 3) + Bx.
x(2x + 3) x 2x + 3
Setting x = 0 and x = −3/2 gives A = 1/3 and B = −2/3. Thus
Z Z Z
1 1 1 2 1
dx = dx − dx
x(2x + 3) 3 x 3 2x + 3

1 1 1 x
= ln |x| − ln |2x + 3| + C = ln + C.
3 3 3 2x + 3
7.6. PARTIAL FRACTIONS 461

x+2 x+2 A B
11. Write = = + . Then x + 2 = A(2x − 1) + Bx.
2x2 − x x(2x − 1) x 2x − 1
Setting x = 0 and x = 1/2 gives A = −2 and B = 5. Thus
Z Z Z
x+2 1 1 5
dx = −2 dx + 5 dx = −2 ln |x| + ln |2x − 1| + C.
2x2 − x x 2x − 1 2

3x + 10 A B
12. Write = + . Then 3x + 10 = A(x + 2) + Bx.
x + 2x
2 x x+2
Setting x = 0 and x = −2 gives A = 5 and B = −2. Thus
Z Z Z
3x + 10 1 1
dx = 5 dx + −2 dx = 5 ln |x| − 2 ln |x + 2| + C.
x + 2x
2 x x+2

x+1 A B
13. Write = + . Then x + 1 = A(x − 4) + B(x + 4).
x − 16
2 x+4 x−4
Setting x = −4 and x = 4 gives A = 3/8 and B = 5/8. Thus
Z Z Z
x+1 3 1 5 1 3 5
dx = dx + dx = ln |x + 4| + ln |x − 4| + C.
x − 16
2 8 x+4 8 x−4 8 8

1 A B
14. Write = + . Then 1 = A(2x − 5) + B(2x + 5).
4x2
− 25 2x + 5 2x − 5
Setting x = −5/2 and x = 5/2 gives A = −1/10 and B = 1/10. Thus
Z Z Z
1 1 1 1 1
dx = − dx + dx
4x − 25
2 10 2x + 5 10 2x − 5

1 1 1 2x − 5
+ C.
= − ln |2x + 5| + ln |2x − 5| + C = ln
20 20 20 2x + 5

x A B
15. Write = + .
2x2
+ 5x + 2 2x + 1 x + 2
Then x = A(x + 2) + B(2x + 1).
Setting x = −1/2 and x = −2 gives A = −1/3 and B = 2/3. Thus
Z Z Z
x 1 1 2 1 1 2
dx = − dx + dx = − ln |2x + 1| + ln |x + 2| + C.
2x + 5x + 2
2 3 2x + 1 3 x+2 6 3

x+5 A B C
16. Write = + + .
(x + 4)(x2 − 1) x+4 x−1 x+1
Then x + 5 = A(x2 − 1) + B(x + 4)(x + 1) + C(x + 4)(x − 1).
Setting x = −4, x = 1, and x = −1 gives A = 1/15, B = 3/5, and C = −2/3. Thus
Z Z Z Z
x+5 1 1 3 1 2 1
dx = dx + dx − dx
(x + 4)(x2 − 1) 15 x+4 5 x−1 3 x+1
1 3 2
= ln |x + 4| + ln |x − 1| − ln |x + 1| + C.
15 5 3
462 CHAPTER 7. TECHNIQUES OF INTEGRATION

x2 + 2x + 6 A B C
17. Write = + + .
3
x −x x x−1 x+1
Then x2 + 2x − 6 = A(x2 − 1) + B(x2 + x) + C(x2 − x).
Setting x = 0, x = 1, and x = −1 gives A = 6, B = −3/2, and C = −7/2. Thus
Z 2 Z Z Z
x + 2x − 6 1 3 1 7 1
dx = 6 dx − dx − dx
3
x −x x 2 x−1 2 x+1
3 7
= 6 ln |x| − ln |x − 1| − ln |x + 1| + C.
2 2

5x2 − x + 1 A B C
18. Write = + + .
x − 4x
3 x x−2 x+2
Then 5x2 − x + 1 = A(x2 − 4) + B(x2 + 2x) + C(x2 − 2x).
Setting x = 0, x = 2, and x = −2 gives A = −1/4, B = 19/8, and C = 23/8. Thus
Z Z Z Z
5x2 − x + 1 1 1 19 1 23 1
dx = − dx + dx + dx
x − 4x
3 4 x 8 x−2 8 x+2
1 19 23
= − ln |x| + ln |x − 2| − ln |x + 2| + C.
4 8 8

1 A B C
19. Write = + + .
(x + 1)(x + 2)(x + 3) x+1 x+2 x+3
Then 1 = A(x + 2)(x + 3) + B(x + 1)(x + 3) + C(x + 1)(x + 2).
Setting x = −1, x = −2, and x = −3 gives A = 1/2, B = −1, and C = 1/2. Thus
Z Z Z Z
1 1 1 1 1 1
dx = dx − dx + dx
(x + 1)(x + 2)(x + 3) 2 x+1 x+2 2 x+3
1 1
= ln |x + 1| − ln |x + 2| + ln |x + 3| + C.
2 2

1 A B C
20. Write = + + .
(4x2 − 1)(x + 7) 2x − 1 2x + 1 x + 7
Then 1 = A(2x + 1)(x + 7) + B(2x − 1)(x + 7) + C(4x2 − 1).
Setting x = 1/2, x = −1/2, and x = −7 gives A = 1/15, B = −1/13, and C = 1/195. Thus
Z Z Z Z
1 1 1 1 1 1 1
dx = dx − dx + dx
(4x − 1)(x + 7)
2 15 2x − 1 13 2x + 1 195 x+7
1 1 1
= ln |2x − 1| − ln |2x + 1| + ln |x + 7| + C.
30 26 195

4t2 + 3t − 1 A B C
21. Write = + 2+ .
t3 − t2 t t t−1
Then 4t2 + 3t − 1 = A(t2 − t) + B(t − 1) + Ct2 = (A + C)t2 + (−A + B)t − B.
Solving A+C =4 −A+B =3 − B = −1
7.6. PARTIAL FRACTIONS 463

gives A = −2, B = 1, and C = 6. Thus


Z Z Z Z
4t2 + 3t − 1 1 1 1 1
dt = −2 dt + dt + 6 dt = −2 ln |t| − + 6 ln |t − 1| + C.
3
t −t 2 t t2 t−1 t
2x − 11 A B C
22. Write = + 2+ .
x3 + 2x2 x x x+2
Then 2x − 11 = A(x2 + 2x) + B(x + 2) + Cx2 = (A + C)x2 + (2A + B)x + 2B.
Solving A+C =0 2A + B = 2 2B = −11
gives A = 15/4, B = −11/2, and C = −15/4. Thus
Z Z Z Z
2x − 11 15 1 11 1 15 1
dx = dx − dx − dx
x3 + 2x2 4 x 2 x2 4 x+2
15 11 15
= ln |x| + x−1 − ln |x + 2| + C.
4 2 4
1 A B C
23. Write = + + .
x3 + 2x + x
2 x x + 1 (x + 1)2
Then 1 = A(x + 1)2 + B(x2 + x) + Cx = (A + B)x2 + (2A + B + C)x + A.
Solving A+B =0 2A + B + C = 0 A=1
gives A = 1, B = −1, and C = −1. Thus
Z Z Z Z
1 1 1 1
dx = dx − dx − dx
x3 + 2x2 + x x x+1 (x + 1)2
1
= ln |x| − ln |x + 1| + + C.
x+1
t−1 A B C D
24. Write = + 2+ + .
t4 + 6t + 9t
3 2 t t t + 3 (t + 3)2
Then t − 1 = At(t + 3)2 + B(t + 3)2 + Ct2 (t + 3) + Dt2
= (A + C)t3 + (6A + B + 3C + D)t2 + (9A + 6B)t + 9B.
Solving A+C =0 6A + B + 3C + D = 0
9A + 6B = 1 9B = −1
gives A = 5/27, B = −1/9, C = −5/27, and D = −4/9. Thus
Z Z Z Z Z
t−1 5 1 1 1 5 1 4 1
dt = dt − dt − dt − dt
t + 6t + 9t
4 3 2 27 t 9 t 2 27 t+3 9 (t + 3)2
5 1 5 4
= ln |t| + t−1 − ln |t + 3| + (t + 3)−1 + C.
27 9 27 9
Z Z Z Z
2x − 1 2(x + 1) − 3 2 3
25. dx = dx = dx − dx
(x + 1) 3 (x + 1) 3 (x + 1) 2 (x + 1)3
 
2 3 1 3 2
=− + +C = − +C
x + 1 2 (x + 1)2 2(x + 1)2 x+1
464 CHAPTER 7. TECHNIQUES OF INTEGRATION

1 A B C D E F
26. Write = + 2+ + + + .
x2 (x2 − 4)2 x x x − 2 (x − 2)2 x + 2 (x + 2)2
Then 1 = Ax(x2 − 4)2 + B(x2 − 4)2 + Cx2 (x − 2)(x + 2)2 + Dx2 (x + 2)2
+ Ex2 (x − 2)2 (x + 2) + F x2 (x − 2)2
= (A + C + E)x5 + (B + 2C + D − 2E + F )x4 + (−8A − 4C + 4D − 4E − 4F )x3
+ (−8B − 8C + 4D + 8E + 4F )x2 + 16Ax + 16B.
Solving A+C +E =0 B + 2C + D − 2E + F = 0
−8A − 4C + 4D − 4E − 4F = 0 −8B − 8C + 4D + 8E + 4F = 0
16A = 0 16B = 1
gives A = 0, B = 1/16, C = −3/128, D = 1/64, E = 3/128, and F = 1/64. Thus
Z Z Z Z
1 1 1 3 1 1 1
dx = dx − dx − dx
x2 (x2 − 4)2 16 x2 128 x−2 64 (x − 2)2
Z Z
3 1 1 1
+ dx + dx
128 x+2 64 (x + 2)2
1 3 1
= − x−1 − ln |x − 2| − (x − 2)−1
16 128 64
3 1
+ ln |x + 2| − (x + 2)−1 + C.
128 64
1 A B C D
27. Write = + + + .
(x2 + 6x + 5)2 x + 1 (x + 1)2 x + 5 (x + 5)2
Then 1 = A(x + 1)(x + 5)2 + B(x + 5)2 + C(x + 5)(x + 1)2 + D(x + 1)2
= (A + C)x3 + (11A + B + 7C + D)x2 + (35A + 10B + 11C + 2D)x
+ 25A + 25B + 5C + D.
Solving A+C =0 11A + B + 7C + D = 0
35A + 10B + 11C + 2D = 0 25A + 25B + 5C + D = 1
gives A = −1/32, B = 1/16, C = 1/32, and D = 1/16. Thus
Z Z Z Z
1 1 1 1 1 1 1
dx = − dx + dx + dx
(x + 6x + 5)
2 2 32 x+1 16 (x + 1) 2 32 x+5
Z
1 1
+ dx
16 (x + 5)2
   
1 1 1 1 1 1
= − ln |x + 1| − + ln |x + 5| − + C.
32 16 x + 1 32 16 x + 5

1 A B C D
28. Write = + + + .
(x2 − x − 6)(x − 2x − 8)
2 x − 4 x − 3 x + 2 (x + 2)2
Then 1 = A(x − 3)(x + 2)2 + B(x − 4)(x + 2)2 + C(x − 4)(x − 3)(x + 2) + D(x − 4)(x − 3)
= (A + B + C)x3 + (A − 5C + D)x2 + (−8A − 12B − 2C − 7D)x
+ (−12A − 16B + 24C + 12D).
7.6. PARTIAL FRACTIONS 465

Solving A+B+C =0 A − 5C + D = 0
−8A − 12B − 2C − 7D = 0 −12A − 16B + 24C + 12D = 1
gives A = 1/36, B = −1/25, C = 11/900, and D = 1/30. (Note that A and B can be easily
obtained by substituting x = 4 and x = 3, respectively, in the initial equation.) Thus
Z Z Z Z
1 1 1 1 1 11 1
dx = dx − dx + dx
(x2 − x − 6)(x2 − 2x − 8) 36 x−4 25 x−3 900 x+2
Z
1 1
+ dx
30 (x + 2)2
1 1 11
= ln |x − 4| − ln |x − 3| + ln |x + 2|
36 25 900
1
− (x + 2)−1 + C.
30

x4 + 2x2 − x + 9 A B C D E
29. Write = + 2+ 3+ 4+ .
x5 + 2x4 x x x x x+2
Then x4 + 2x2 − x + 9 = Ax3 (x + 2) + Bx2 (x + 2) + Cx(x + 2) + D(x + 2) + Ex4
= (A + E)x4 + (2A + B)x3 + (2B + C)x2 + (2C + D)x + 2D.

Solving A+E =1 2A + B = 0 2B + C = 2
2C + D = −1 2D = 9
gives A = −19/16, B = 19/8, C = −11/4, D = 9/2, and E = 35/16. Thus
Z Z Z Z
x4 + 2x2 − x + 9 19 1 19 1 11 1
dx = − dx + dx − dx
x + 2x
5 4 16 x 8 x 2 4 x3
Z Z
9 1 35 1
+ dx + dx
2 x4 16 x+2
     
19 19 1 11 1 3 1 35
= − ln |x| − + − + ln |x + 2| + C.
16 8 x 8 x2 2 x3 16

5x − 1 A B C D E
30. Write = + + + + .
x(x − 3) (x + 2)
2 2 x x − 3 (x − 3)2 x + 2 (x + 2)2
Then 5x − 1 = A(x − 3)2 (x + 2)2 + Bx(x − 3)(x + 2)2 + Cx(x + 2)2
+ Dx(x − 3)2 (x + 2) + Ex(x − 3)2
= (A + B + D)x4 + (−2A + B + C + 4D + E)x3
+ (11A − 8B + 4C − 3D − 6E)x2 + (12A − 12B + 4C + 18D + 9E)x
+ 36A.

Solving A+B+D =0 −2A + B + C − 4D + E = 0


11A − 8B + 4C − 3D − 6E = 0 12A − 12B + 4C + 18D + 9E = 5
36A = −1
466 CHAPTER 7. TECHNIQUES OF INTEGRATION

gives A = −1/36, B = −79/1125, C = 14/75, D = 49/500, and E = 11/50. Thus


Z Z Z Z
5x − 1 1 1 79 1 14 1
dx = − dx − dx + dx
x(x − 3)2 (x + 2)2 36 x 1125 x−3 75 (x − 3)2
Z Z
49 1 11 1
+ dx + dx
500 x+2 50 (x + 2)2
1 79 14 49
= − ln |x| − ln |x − 3| − (x − 3)−1 + ln |x + 2|
36 1125 75 500
11
− (x + 2)−1 + C.
50
x−1 A Bx + C
31. Write = + 2 .
x(x + 1)
2 x x +1
Then x − 1 = A(x2 + 1) + (Bx + C)x = (A + B)x2 + Cx + A.
Solving A+B =0 C=1 A = −1
gives A = −1, B = 1, and C = 1. Thus
Z Z Z Z
x−1 1 x 1
dx = − dx + dx + dx
x(x2 + 1) x x2 + 1 x2 + 1
1
= − ln |x| + ln(x2 + 1) + tan−1 x + C.
2
1 A Bx + C
32. Write = + 2 .
(x − 1)(x + 3)
2 x−1 x +3
Then 1 = A(x2 + 3) + (Bx + C)(x − 1) = (A + B)x2 + (−B + C)x + (3A − C).
Solving A+B =0 −B+C =0 3A − C = 1
gives A = 1/4, B = −1/4, and C = −1/4. Thus
Z Z Z Z
1 1 1 1 x 1 1
dx = dx − dx − dx
(x − 1)(x2 + 3) 4 x−1 4 x2 + 3 4 x2 + 3
1 1 1 x
= ln |x − 1| − ln(x2 + 3) − √ tan−1 √ + C.
4 8 4 3 3
x A B Cx + D
33. Write = + + 2 .
(x + 1)2 (x2 + 1) x + 1 (x + 1)2 x +1
Then x = A(x + 1)(x2 + 1) + B(x2 + 1) + (Cx + D)(x + 1)2
= (A + C)x3 + (A + B + 2C + D)x2 + (A + C + 2D)x + (A + B + D).
Solving A+C =0 A + B + 2C + D = 0
A + C + 2D = 1 A+B+D =0
gives A = 0, B = −1/2, C = 0, and D = 1/2. Thus
Z Z Z  
x 1 1 1 1 1 1 1
dx = − dx + dx = + tan−1 x + C.
(x + 1)2 (x2 + 1) 2 (x + 1)2 2 x2 + 1 2 x+1 2
7.6. PARTIAL FRACTIONS 467

x2 A B C Dx + E
34. Write = + + + 2 .
(x − 1) (x + 4)
3 2 x − 1 (x − 1)2 (x − 1)3 x +4
Then x2 = A(x − 1)2 (x2 + 4) + B(x − 1)(x2 + 4) + C(x2 + 4) + (Dx + E)(x − 1)3
= (A + D)x4 + (−2A + B − 3D + E)x3 + (5A − B + C + 3D − 3E)x2
+ (−8A + 4B − D + 3E)x + (4A − 4B + 4C − E).

Solving A+D =0 −2A + B − 3D + E = 0


5A − B + C + 3D − 3E = 1 −8A + 4B − D + 3E = 0
4A − 4B + 4C − E = 0

gives A = 4/125, B = 8/25, C = 1/5, D = −4/125, and E = −44/125. Thus


Z Z Z Z
x2 4 1 8 1 1 1
dx = dx + dx + dx
(x − 1)3 (x2 + 4) 125 x−1 25 (x − 1)2 5 (x − 1)3
Z Z
4 x 44 1
− dx − dx
125 x +4
2 125 x +4
2

4 8 1
= ln |x − 1| − (x − 1)−1 − (x − 1)−2
125 25 10
2 22 x
− ln(x2 + 4) − tan−1 + C.
125 125 2

1 Ax + B Cx + D
35. Write = 2 + 2 .
x4 + 5x + 4
2 x +1 x +4
Then 1 = (Ax + B)(x2 + 4) + (Cx + D)(x2 + 1)
= (A + C)x3 + (B + D)x2 + (4A + C)x + (4B + D).

Solving A+C =0 B+D =0 4A + C = 0 4B + D = 1

gives A = 0, B = 1/3, C = 0, and D = −1/3. Thus


Z Z Z
1 1 1 1 1 1 1 x
dx = dx − dx = tan−1 x − tan−1 + C.
x4 + 5x2 + 4 3 x2 + 1 3 x2 + 4 3 6 2

1 Ax + B Cx + D
36. Write = 2 + 2 .
x4 + 13x2 + 36 x +9 x +4
Then 1 = (Ax + B)(x2 + 4) + (Cx + D)(x2 + 9)
= (A + C)x3 + (B + D)x2 + (4A + 9C)x + (4B + 9D).

Solving A+C =0 B+D =0 4A + 9C = 0 4B + 9D = 1

gives A = 0, B = −1/5, C = 0, and D = 1/5. Thus


Z Z Z
1 1 1 1 1 1 x 1 x
dx = − dx + dx = − tan−1 + tan−1 + C.
x4 + 13x2 + 36 5 x2 + 9 5 x2 + 4 15 3 10 2
468 CHAPTER 7. TECHNIQUES OF INTEGRATION

1 A Bx + C
37. Write = + .
x3 − 1 x − 1 x2 + x + 1
Then 1 = A(x2 + x + 1) + (Bx + C)(x − 1) = (A + B)x2 + (A − B + C)x + (A − C).
Solving A+B =0 A−B+C =0 A−C =1
gives A = 1/3, B = −1/3, and C = −2/3. Thus
Z Z Z
1 1 1 1 2x + 4
dx = dx − dx
x −1
3 3 x−1 6 x +x+1
2
Z Z
1 1 2x + 1 1 1
= ln |x − 1| − dx − dx
3 6 x2 + x + 1 2 x2 + x + 1
Z
1 1 1 1
= ln |x − 1| − ln |x2 + x + 1| − dx
3 6 2 (x + 1/2)2 + 3/4
1 1 1 2x + 1
= ln |x − 1| − ln |x2 + x + 1| − √ tan−1 √ + C.
3 6 3 3
81 A B Cx + D
38. Write = + + .
x4 + 27x x x + 3 x2 − 3x + 9
Then 81 = A(x3 + 27) + B(x3 − 3x2 + 9x) + (Cx + D)(x2 + 3x)
= (A + B + C)x3 + (−3B + 3C + D)x2 + (9B + 3D)x + 27A.
Solving A+B+C =0 −3B + 3C + D = 0
9B + 3D = 0 27A = 81
gives A = 3, B = −1, C = −2, and D = 3. Thus
Z Z Z Z
81 1 1 2x − 3
dx = 3 dx − dx − dx
x4 + 27x x x+3 x2 − 3x + 9

x3

= 3 ln |x| − ln |x + 3| − ln |x − 3x + 9| + C = ln 3
2 + C.
x + 27

3x2 − x + 1 A Bx + C
39. Write = + .
(x + 1)(x2 + 2x + 2) x + 1 x2 + 2x + 2
Then 3x2 − x + 1 = A(x2 + 2x + 2) + (Bx + C)(x + 1)
= (A + B)x2 + (2A + B + C)x + (2A + C).
Solving A+B =3 2A + B + C = −1 2A + C = 1
gives A = 5, B = −2, and C = −9. Thus
Z Z Z
3x2 − x + 1 1 2x + 9
dx = 5 dx − dx
(x + 1)(x2 + 2x + 2) x+1 x2 + 2x + 2
Z Z
2x + 2 7
= 5 ln |x + 1| − dx − dx
x + 2x + 2
2 x + 2x + 2
2
Z
7
= 5 ln |x + 1| − ln |x2 + 2x + 2| − dx
(x + 1)2 + 1
= 5 ln |x + 1| − ln |x2 + 2x + 2| − 7 tan−1 (x + 1) + C.
7.6. PARTIAL FRACTIONS 469

4x + 12 A Bx + C
40. Write = + .
(x − 2)(x2 + 4x + 8) x − 2 x2 + 4x + 8
Then 4x + 12 = A(x2 + 4x + 8) + (Bx + C)(x − 2)
= (A + B)x2 + (4A − 2B + C)x + (8A − 2C).
Solving A+B =0 4A − 2B + C = 4 8A − 2C = 12
gives A = 1, B = −1, and C = −2. Thus
Z Z Z
4x + 12 1 x+2
dx = dx − dx
(x − 2)(x2 + 4x + 8) x−2 x2 + 4x + 8
Z
1 2x + 4
= ln |x − 2| − dx
2 x + 4x + 8
2

1
= ln |x − 2| − ln |x2 + 4x + 8| + C.
2
x2 − x + 4 Ax + B Cx + D
41. Write = 2 + 2 .
(x2 + 4)2 x +4 (x + 4)2
Then x2 − x + 4 = (Ax + B)(x2 + 4) + Cx + D = Ax3 + Bx2 + (4A + C)x + (4B + D).
Solving A=0 B=1 4A + C = −1 4B + D = 4
gives A = 0, B = 1, C = −1, and D = 0. Thus
Z 2 Z Z  
x −x+4 1 x 1 −1 x 1 1
dx = dx − dx = tan + +C
(x2 + 4)2 x2 + 4 (x2 + 4)2 2 2 2 x2 + 4
1 A B C Dx + E Fx + G
42. Write = + 2+ 3+ 2 + 2 .
x3 (x2 + 1)2 x x x x +1 (x + 1)2
Then 1 = Ax2 (x2 + 1)2 + Bx(x2 + 1)2 + C(x2 + 1)2 + (Dx + E)x3 (x2 + 1) + (F x + G)x3
= (A + D)x6 + (B + E)x5 + (2A + C + D + F )x4 + (2B + E + G)x3
+ (A + 2C)x2 + Bx + C.
Solving A+D =0 B+E =0 2A + C + D + F = 0
2B + E + G = 0 A + 2C = 0 B=0 C=1
gives A = −2, B = 0, C = 1, D = 2, E = 0, F = 1, and G = 0. Thus
Z Z Z Z Z
1 1 1 2x 1 2x
dx = −2 dx + dx + dx + dx
x (x + 1)
3 2 2 x x 3 x +1
2 2 (x + 1)2
2

1 1
= −2 ln |x| − x−2 + ln(x2 + 1) − (x2 + 1)−1 + C.
2 2
Z
43. For this and possibly later problems, we will encounter cos2 θ dθ. Using Example 12 of
Section 5.2 in the text, we have
Z
1 1 1 1
cos2 θ dθ = θ + sin 2θ + C = θ + sin θ cos θ + C.
2 4 2 2
243 81 81
θ+
= sin θ cos θ +
8 2 16
243 81 81
= θ+ sin θ cos θ +
8 2 " 8
243 1
= θ + 81 sin θ cos θ +
8 2
"
243 5
= θ + 81 sin θ cos θ −
8 8
243 $
470 CHAPTER 7. TECHNIQUES OF INTEGRATION =
x
sin−1 + 9x 9 − x2
8 3
243 −1 x 1 $
= sin + x 9−x
x − 2x + x − 3
3 2
Ax + B Cx + D 7.5. TRIGONOMETRIC SUBSTITUTIONS
8 3 8
Write = 2 + 2 . ! $
x4 + 8x2 + 16 x +4 (x + 4)2 7.5 Trigonometric 11. 4 +Substitutions
x2 dx x = 2 tan θ, dx = 2 sec2 θ
x
!
Then x3 − 2x2 + x − 3 = (Ax + B)(x2 + 4) + Cx + D ! √
1 − x2 =
$
4 tan2 θ + 4 2 sec2 θ dθ
1. 2
dx x = sin θ, dx = cos θ dθ
x
= Ax + Bx + (4A + C)x + (4B + D).
3 2
! " 2 See Section
! 7.3, Example
!
1 − sin2 θ cos2 θ
= cos θ dθ = 2 2 dθ = cot2
sin2 θ = 2 sec θ tansin θ+ θ ln | sec θ + ta
Solving A=1 B = −2 4A + C = 1 4B + D = −3 ! √ √' √
'1 − 2x
= (csc2 θ − 1) dθ = = 2− cotx2 θ+−4θ%+x C &
=2−ln '' x
+
gives A = 1, B = −2, C = −3, and D = 5. Thus 2 2 ' x 2
TODO figure $ '$
Z 3 Z Z Z =
x '
x2 + 4 + 2 ln ' x2 + 4 +
x − 2x2 + x − 3 x 1 !
x x3 2
dx = dx − 2 dx − 3 2. √ dx
dx !x = 2 sec θ, dx = 2!sec θ tan θ dθ
1 1 1
x4 + 8x2 + 16 x2 + 4 x2 + 4 (x2 + x4) 2− 24
12. !
x
dx = (2x dx)! = ln(
Z 258 +secx32θ 2 25 + x2 2
1 = (2 sec tan = 8 sec 4

! 24 sec2 θ − 4 θ θ) dθ θ dθ
+5 dx x = 2 tan θ, dx = 2
13. sec 1
! √ θ dθ dx = sin−1 + x #C $
(x2 + 4)2 = 8 (1 25 + tan− x22θ) sec2 θ dθ =58 tan θ + 1 tan3 θ
  Z ! 3
1 x 3 1 2 sec
14. " 2√ θ 1 1 dx 1 −1 |x|
= ln(x2 + 4) − tan−1 + + 5 = 4 xx2 −x42 + dθ2 −=4)53/2sec+ C 5 + C
−2325(x
2 2 2 x + 4 TODO figure
2 (4 tan θ + 4)
2!
1
Z 15. √ dx x = 4 sin θ, dx = 4 cos
1 x 3 5
7.5. TRIGONOMETRIC x 16 − x2
SUBSTITUTIONS
= ln(x + 4) − tan
2 −1
+ +3. cos θ dθ
2 !
4 cos θ
2 2 2(x2 + 4) 4. 8 ! 470 √
= $ CHAPT
1 − x2 4 sin θ 16 − 16 sin2 θ
1 x 3 5 1. 5 dx x = sin θ, dx = cos θ dθ
= ln(x2 + 4) − tan−1 + +5. θ + x2gives sin Aθ =cos
! 0,"θB+ = 1, CC == 410,lnand | cscD θ − cot θ| Thus
! = −3.
+C =
2 2 2(x2 + 4) 16 16 1 − sin2 θ cos2 θ
!
6. ! = 2 ! cos θ dθ = !2 θ dθ = c
1 x 3 5 x x 2 sin θ 1 sin 1√
= ln(x2 + 4) − tan−1 + +7. tan−1 (x2=+!3)(csc 2 2
dx = dx − 3
x2=+−3cot θ − θ + C(x= 2 + 3)12
2 2 2(x + 4) 16
2 2 θ − 1) dθ
! √

  8. ! 1 −1 x 37.5.
se
5 x 2.
x 3 = √ tan √ − 3
x = 2 sec θ, 3dx = 2 sec θ3tan θ dθ (3 tan7.5 2
+ +C 9.

x2 − 4
dx θ
8 x2 + 4 !
8 sec= 3
1 x
θ√ tan−1 √ − √ θ − √41
1 ! 1
10. = √ 3 (2 sec θ tan 3 θ) dθ2 =3 8 sec 2 3
1 11 x 5x + 12 4 sec2 θ − 4
= ln(x2 + 4) − tan−1 + 11. + C.
!
=
1
tan −1 x # 1
tan1−1 √
2 6 2 8(x + 4)2 √ √
= 8 (1 + tan θ) sec θ dθ = 8 tan θ + tan
2
3
2 −
3 2 3 3

12. " 11 −1 x x
7.5. TRIGONOMETRIC = 4 x2 − 4=+ √(x2tan
SUBSTITUTIONS − 4)3/2√+ C − +C
x 2
Ax + B Cx + D 13. 23 3 3 2x2 + 6 2
44. Write 2 = + . ! $ % " &
(x + 3)2 x2 + 3 (x2 + 3)2 7.514. Trigonometric
3. √
1 3 +Substitutions
dx = cosh x2 x + C = ln x + x2 − 362 + C,
−1
x
x2 − 36 6
! √
Then x2 = (Ax + B)(x2 + 3) + Cx + D 15.
1.
1Alternatively,
− x2
dx
the substitution x = 6 sec θ could have been us
x= sin θ, dx = cos θ dθ
16. x!2 " x4 + 3x2√+ 4 √ 10x + 2
= Ax3 + Bx2 + (3A + C)x + (3B + D). 45.
3 −Write! " (xx+=1)223 sin= θ, xdx−=2x ! 3+cos 6− !2 =
2
4. x2 dx θ dθ
2 (x + 1)
1 − sin θ cos θ
17. = ! ! "2 cos θ √ dθ = ! !dθ = cot3
Solving A=0 B=1 3A + C = 0 3B + D = Then 0 ! = xsin+3θ3x + 24θ 3 cos θ dθ
− 32 sin sin=2 θ3 cos2 θ dθ =
4
dx = (x 2
− 2x + √ dx −
6) 4
1−x
= (csc32 θ(x
# + 1)2 $
− 1)1 dθ = − cot θ − 3θ + C3 = − 5

gives A = 0, B = 1, C = 0, and D = −3. Thus =


2
θ + sin 2θ + C 1= θ + sin θ cos x
2 = x 2 − x22 + 6x − 106
3
θ+
TODO figure # 3 √3 − x2
$
Z Z Z √ ! x3 √ = 23 2sin−1 √x + 32 √x 3 7
x2 1 1 √ + C = si8
2
dx = dx − 3 dx x = 32.tan θ, √ dx = dx 3 xsec
= 52 secθ dθ 3 3
θ, 3dx = 2 sec θ tan θ9x 3

(x2 + 3)2 x2 + 3 (x2 + 3)2 2
5.x −46. 4 Write x − 10x =x− 4
9
! x4 − 10x ! 9 = x 10 −
Z √ Z 8 sec θ3 2+9 x − 10x 2+

1 3 sec2 θ 1 x = 1 √4 sec2 θ − (2 sec θ tan θ) dθ = 8 sec θ 11 4


x 6. d
41)(x2 − 9) + B(x − 1)(x2 − 9)
= √ tan−1 √ − 3 dθ = √ tan−1 √ −Then √!9x = cos A(x2+θ dθ # 12
3 3 (3 tan θ + 3)
2 2
3 7. 3 = 8 3 (1 + tan2 θ) sec2 θ dθ = 8 tan θ + 1 tan3 13θ
Setting x = 1, x = −1, x = 3, and x =3−3 giv
1 x 1 1 8.
D =" 9/16. Thus1
14
= √ tan−1 √ − √ θ − √ sin θ cos θ + C = 4 x2 − 4 + (x2 − 4)3/2 + C 15
3 3 2 3 2 3 9. 3 % 16
 TODO 10. figure
!
x − 10x
5 3 !
9
!
1
1 x 1 x 1 x dx = x dx − − 17

= √ tan−1 √ − √ tan−1 √ − 3. + C x4 − 10x2 + 9 16 x−


3 3 2 3 3 2 x2 + 3 11.
!
4. 9 1
1 x x 12. +
16 x+3
dx
= √ tan−1 √ − 2 +C 5. 13.
2 3 3 2x + 6 =
1
+
9
ln |x − 1| +
9
6. 14. 16
x2 ' 2 ' 16
1 9 'x − 1'
7. = x2 + ln ' '+
2 16 ' x2 − 9 '
8.

9. 1 A B
47. Write = + .
x2 − 6x + 5 x−1 x−5
10.

11.

12.

13.

14.

15.
7.6. PARTIAL FRACTIONS 471

x4 + 3x2 + 4 10x + 2 10(x + 1) − 8


45. Write = x2 − 2x + 6 − = x2 − 2x + 6 − .
(x + 1) 2 (x + 1) 2 (x + 1)2
Z 4 Z Z Z
x + 3x2 + 4 1 1
Then dx = (x 2
− 2x + 6) dx − 10 dx + 8 dx
(x + 1) 2 x+1 (x + 1)2
1 8
= x3 − x2 + 6x − 10 ln |x + 1| − + C.
3 x+1
 
x5 − 10x3 9x A B C D
46. Write 4 = x − = x − + + + .
x − 10x2 + 9 x4 − 10x2 + 9 x−1 x+1 x−3 x+3
Then 9x = A(x + 1)(x2 − 9) + B(x − 1)(x2 − 9) + C(x + 3)(x2 − 1) + D(x − 3)(x2 − 1).
Setting x = 1, x = −1, x = 3, and x = −3 gives A = −9/16, B = −9/16, C = 9/16, and
D = 9/16. Thus
Z Z Z Z Z
x5 − 10x3 9 1 9 1 9 1
dx = x dx − − dx − dx + dx
x4 − 10x2 + 9 16 x−1 16 x+1 16 x−3
Z !
9 1
+ dx
16 x+3
1 9 9 9 9
= + ln |x − 1| + ln |x + 1| − ln |x − 3| − ln |x + 3| + C
x2 16 2 16 16 16
1 9 x − 1
= x2 + ln 2 + C.
2 16 x − 9

1 A B
47. Write = + .
x2 − 6x + 5 x−1 x−5
Then 1 = A(x − 5) + B(x − 1). Setting x = 1 and x = 5 gives A = −1/4 and B = 1/4. Thus
Z 4 Z Z 4 4
1 1 4 1 1 4 1 1 1
dx = − dx + dx = − ln |x − 1| + ln |x − 5|
2 x − 6x + 5 4 2 x−1 4 2 x−5 4 4
2
2 2
4  
1 x − 5
1 1 1
= ln = ln − ln 3 = − ln 3.
4 x − 1 2 4 3 2

1 A B
48. Write = + .
x2−4 x−2 x+2
Then 1 = A(x + 2) + B(x − 2). Setting x = 2 and x = −2 gives A = 1/4 and B = −1/4.
Z 1 Z Z 1 1
1 1 1 1 1 1 1 1 1
Thus dx = dx − dx = ln |x − 2| − ln |x + 2|
0 x −4 4 0 x−2 4 0 x+2 4 4
2
0 0
1  
1 x − 2 1 1 1
= ln = ln − ln 1 = − ln 3.
4 x + 2 0 4 3 4
Z 2 Z 2 Z 2 Z 2
2x − 1 2(x + 3) − 7 2 1
49. dx = dx = dx − 7 dx
0 (x + 3) (x + 3) 0 x+3 0 (x + 3)
2 2 2
0
i2 2  
7 7 7 5 14
= 2 ln |x + 3| + = 2(ln 5 − ln 3) + − = 2 ln −
0 x+3 0 5 3 3 15
472 CHAPTER 7. TECHNIQUES OF INTEGRATION

2x + 6 A B C
50. Write = + + .
x(x + 1)2 x x + 1 (x + 1)2
Then 2x + 6 = A(x + 1)2 + B(x2 + x) + Cx = (A + B)x2 + (2A + B + C)x + A.

Solving A+B =0 2A + B + C = 2 A=6


gives A = 6, B = −6, and C = −4. Thus
Z 5 Z 5 Z 5 Z 5
2x + 6 1 1 1
= 6 dx − 6 dx − 4 dx
1 x(x + 1) 1 x+1 1 (x + 1)
2 2
1 x
i5 i5 5  5
4 x 5
= 6 ln |x| − 6 ln |x + 1| + = 6 ln + 4
1 1 x+1 1 x + 1 1 x+1 1
   
5 1 4 4 5 4
=6 − ln + − = 6 ln − .
6 2 6 2 3 3

1 A Bx + C
51. Write = + 2 .
+ + 2x + 2
x3 x2 x+1 x +2
Then 1 = A(x2 + 2) + (Bx + C)(x + 1) = (A + B)x2 + (B + C)x + (2A + C).

Solving A+B =0 B+C =0 2A + C = 1


gives A = 1/3, B = −1/3, and C = 1/3. Thus
Z 1 Z Z Z
1 1 1 1 1 1 2x 1 1 1
dx = dx − dx + dx
0 x + x + 2x + 2 3 0 x+1 6 0 x2 + 2 3 0 x2 + 2
3 2
1 1 1
1 452 1 1 x
CHAPTER 7. TECHNIQUES OF IN
= ln |x + 1| + ln(x2 + 2) + √ tan−1 √
3 ! 1/2
0 6 0 3 2 2 0
1 44. 1 x3 (1 + x2 )−1/21dx x = 1tan θ, dx = sec2 θ dθ
= ln 2 − (ln 3 − ln 2) + √ tan √ −1
3 60 3 2 ! tan−1 (1/2)2
1 8 1 1 = tan3 θ(1 + tan2 θ)−1/2 sec2 θ dθ
= ln + √ tan−1 √ .
6 3 3 2 2
0
! tan−1 (1/2) ! tan−1 (1/2)
3 2
tan θ sec θ
Z Z Z Z=1 sec θ
dθ = tan2 θ tan θ
1
x 2 1
x +4−4
2 1
1 1
0
! tan−1
0
52. dx = dx = dx − 4 (1/2) dx
0 x4 + 8x2 + 16 0 (x2 + 4)2 0 x2 + 4 =
0 (x 2 + 4)2
(sec2 θ − 1) tan θ sec θ dθ
0
x = 2 tan θ, dx = 2 sec2 θ dθ "
1
#$tan−1 (1/2)
1 Z tan−1 1/2 = sec 3
θ − sec θ
3
1 −1 x 2 sec2 θ 0
= tan −4 1dθ 3
2 2 0 0 (4 tan θ + 4)
2 = 2 [sec (tan 1/2) − 1] − [sec(tan−1 1/2) − 1]
3%
−1

Z tan−1 1/2 √ & %√ & √


1 1 1 1 5 5 5 16 − 7 5
= tan−1
− cos θ dθ
2
= −1 − − 1 =1
2 2 2 0 3 8 2 24
 TODO figure tan−1 1/2 2
1 1 1 1 1
= tan−1 − θ + sin θ cos θ
2 2 2 45.2 2
  0 
1 1 1 46. 1 1 1 2 1 1 1
= tan−1 − tan−1 − √ √ = tan−1 −
2 2 4 47. 2 4 5 5 4 2 10
48.
49.
50.
51.
52.
53.
54.
55.
7.6. PARTIAL FRACTIONS 473

Z Z 1
1
2x3 + 5x 1 1 4x3 + 10x 1
53. dx = dx = ln |x4
+ 5x 2
+ 6|
−1 x4 + 5x2 + 6 2 −1 x4 + 5x2 + 6 2 −1
1
= (ln 12 − ln 12) = 0
2
1 A B C Dx + E
54. Write = + 2+ 3+ 2 .
x5 + 4x4 + 5x3 x x x x + 4x + 5
Then 1 = Ax2 (x2 + 4x + 5) + Bx(x2 + 4x + 5) + C(x2 + 4x + 5) + (Dx + E)x3
= (A + D)x4 + (4A + B + E)x3 + (5A + 4B + C)x2 + (5B + 4C)x + 5C.

Solving A+D =0 4A + B + E = 0 5A + 4B + C = 0
5B + 4C = 0 5C = 1

gives A = 11/125, B = −4/25, C = 1/5, D = −11/125, and E = −24/125. Thus


Z Z Z 2 Z
2
1 11 1
2
4 1 1 2 1
dx = dx − dx + dx
1 x + 4x + 5x
5 4 3 125 1 x 25 1 x2 5 1 x3
Z 2 Z 2
11 2x + 4 2 1
− dx − dx
250 1 x + 4x + 5
2 125 1 (x + 2)2 + 1
2  2  2 2
11 4 1 1 1 11
= ln |x| + − − ln |x2
+ 4x + 5|
125 1 25 x 1 10 x2 1 250 1
2
2
− tan−1 (x + 1)
125 1
   
11 4 1 1 1 11
= ln 2 + −1 − −1 − (ln 17 + ln 10)
125 25 2 10 4 250
2
− (tan−1 4 − tan−1 3)
125
11 40 1 2
= ln − − (tan−1 4 − tan−1 3).
250 17 200 125

Z √ Z √
1 − x2 1 − x2
55. dx = x dx u2 = 1 − x2 , 2u du = −2x dx
x3 x4
Z Z
u u2
= (−u du) = − du
(1 − u2 )2 (1 − u2 )2
u2 A B C D
Write = + + + .
(1 − u )
2 2 1 − u (1 − u) 2 1 + u (1 + u)2
Then u2 = A(1 − u)(1 + u)2 + B(1 + u2 ) + C(1 + u)(1 − u)2 + D(1 − u)2
= (A + B + C + D) + (A + 2B − C − 2D)u + (−A + B − C + D)u2 + (−A + C)u3 .

Solving A+B+C +D =0 A + 2B − C − 2D = 0
−A + B − C + D = 1 −A + C = 0
474 CHAPTER 7. TECHNIQUES OF INTEGRATION

gives A = −1/4, B = 1/4, C = −1/4, and D = 1/4. Thus

Z √ Z
1 − x2 u2
dx = − du
x3 (1 − u2 )2
Z Z Z Z
1 1 1 1 1 1 1 1
= du − du + du − du
4 1−u 4 (1 − u)2 4 1+u 4 (1 + u)2
   
1 1 1 1 1 1
= − ln |1 − u| − + ln |1 + u| + +C
4 4 1−u 4 4 1+u
  √ √ !
1 1 + u 1 u 1 1 + 1 − x2 1 1 − x2
= ln − + C = ln √ − + C.
4 1 − u 2 1 − u2 4 1 − 1 − x2 2 x2

Z r
x−1 x−1 1 + u2 4u
56. dx u2 = , x= , dx = 2 du
x+1 x+1 1−u 2 (u − 1)2
Z Z
4u 4u2
= |u| 2 du = du
(u − 1)2 (u2 − 1)2

4u2 A B C D
Write = + + + . Then
(u2 − 1)2 u − 1 (u − 1)2 u + 1 (u + 1)2

4u2 = A(u − 1)(u + 1)2 + B(u + 1)2 + C(u − 1)2 (u + 1) + D(u − 1)2
= (A + C)u3 + (A + B − C + D)u2 + (−A + 2B − C − 2D)u + (−A + B + C + D).

Solving A+C =0 A+B−C +D =4


−A + 2B − C − 2D = 0 −A + B + C + D = 0

gives A = 1, B = 1, C = −1, and D = 1. Thus

Z r Z Z Z Z
x−1 1 1 1 1
dx = du + du − du + du
x+1 u−1 (u − 1)2 u+1 (u + 1)2
1 1
= ln |u − 1| − − ln |u + 1| − +C
u−1 u+1
r r
x−1 x−1

x+1 − 1 2 p p
x+1 2 − 1 − x +
= ln r −
x−1 + C = ln x x2 − 1 + C.
x−1
+ 1 x + 1 − 1
x+1
7.6. PARTIAL FRACTIONS 475
Z √ Z
3
x+1 u
57. dx u3 = x + 1, 3u2 du = dx = (3u2 du)
x u −1
3
Z 3 Z Z Z
u −1+1 1 1
=3 du = 3 du + 3 du = 3u + du
u −1
3 u −1
3 u −1
3

1 √ 2u + 1
= 3u + ln |u − 1| − ln |u2 + u + 1| − 3 tan−1 √ +C
2 3
√ √ 1 p √

= 3 3 x + 1 + ln | 3 x + 1 − 1| − ln 3 (x + 1)2 + 3 x + 1 + 1
√ 2
√ −1 2 x + 1 + 1
3

− 3 tan √ + C.
3
Z
1
58. √ √ x = u6 , dx = 6u5 du
x(1 + 3 x)2
Z Z
6u5 6u2
= du = du u = tan θ, du = sec2 θ dθ
u3 (1 + u2 )2 (1 + u2 )2
Z Z Z
tan2 θ tan2 θ
=6 sec 2
θ dθ = 6 dθ = 6 sin2 θ dθ
(1 + tan2 θ)2 sec2 θ
Z
3
= 3 (1 − cos 2θ) dθ = 3θ − sin 2θ + C = 3θ − 3 sin θ cos θ + C
2
u 3x1/6
= 3 tan−1 u − 3 + C = 3 tan −1 1/6
x − +C
1 + u2 1 + x1/3
1 A B
59. Write 2 = + . 1
x + 2x − 3 x+3 x−1
Then 1 = A(x − 1) + B(x + 3). Setting x = −3 2 4

and x = 1 gives A = −1/4 and B = 1/4. Thus


Z Z 4 4
1 4 1 1 4 1 1 1
Area = − dx + dx = − ln |x + 3| + ln |x − 1|
4 2 x+3 4 2 x−1 4 2 4 2
4  
1 x − 1 1 3 1 1 15
= ln = ln − ln = ln ≈ 0.1905.
4 x + 3 2 4 7 5 4 7

x3 Ax + B Cx + D 1
60. Write = 2 + 2 .
(x2 + 1)(x2 + 2) x +1 x +2
Then x3 = (Ax + B)(x2 + 2) + (Cx + D)(x2 + 1) 2 4

= (A + C)x + (B + D)x + (2A + C)x + (2B + D).


3 2

Solving A+C =1 B+D =0 2A + C = 0 2B + D = 0


gives A = −1, B = 0, C = 2, and D = 0. Thus
Z 4 Z 4 4 i4
x 2x 1
Area = − dx + dx = − ln(x2
+ 1) + ln(x 2
+ 2)
0 x +1 0 x +2 2
2 2 0
0
4
x2 + 2 18 9
= ln √ = ln √ − ln 2 = ln √ ≈ 0.7806
x2 + 1 0 17 17
476 CHAPTER 7. TECHNIQUES OF INTEGRATION

x A B
61. Write = + . Then x = A(x + 3) + B(x + 2). 1
(x + 2)(x + 3) x+2 x+3
Setting x = −2 and x = −3 gives A = −2 and B = 3. Thus
Z 0  Z 1  -1 1
−2 3 −2 3
Area = − + dx + + dx
−1 x+2 x+3 0 x+2 x+3 -1
i0 i1
= −(−2 ln |x + 2| + 3 ln |x + 3|) + (−2 ln |x + 2| + 3 ln |x + 3|)
−1 0
= −[(−2 ln 2 + 3 ln 3) − (−2 ln 1 + 3 ln 2)] + [(−2 ln 3 + 3 ln 4) − (−2 ln 2 + 3 ln 3)]
8192
= 7 ln 2 − 8 ln 3 + 3 ln 4 = ln ≈ 0.2220.
6561
 
3x3 24 A Bx + C
62. Write 3 =3+ 3 = 3 + 24 + 2 . 2
x −8 x −8 x − 2 x + 2x + 4
1
Then 1 = A(x2 + 2x + 4) + (Bx + C)(x − 2)
= (A + B)x2 + (2A − 2B + C)x + (4A − 2C). -2 -1 1

-1
Solving A+B =0 2A − 2B + C = 0 4A − 2C = 1
gives A = 1/12, B = −1/12, and C = −1/3. Thus
Z Z   
3x3 1/12 x/12 1/3
dx = 3 + 24 − − dx
x3 − 8 x − 2 x2 + 2x + 4 x2 + 2x + 4
Z  
2 2x + 8
= 3+ − 2 dx
x − 2 x + 2x + 4
Z Z Z Z
1 1 1
= 3 dx + 2 dx − (2x + 2) dx − 6 dx
x−2 x + 2x + 4
2 x + 2x + 4
2
Z
1
= 3x + 2 ln |x − 2| − ln |x2 + 2x + 4| − 6 dx
(x + 1)2 + 3
2
x − 4x + 4 √ +1
= 3x + ln 2 − 2 3 tan−1 x√ +C
x + 2x + 4 3
Z 2
3x3 x − 4x + 4 √ +1
Let g(x) = dx = 3x + ln − 2 3 tan−1 x√ . Then
x −8
3 x + 2x + 4
2 3
Z 0 Z 1 i0 i1
3x3 3x3
Area = dx − dx = g(x) − g(x) = [g(0) − g(−2)] − [g(1) − g(0)]
−2 x − 8 0 x −8
3 3 −2 0
   
√ 1 √ −1
= 0 + ln 1 − 2 3 tan−1 √ − −6 + ln 4 − 2 3 tan−1 √
3 3
   
1 √ 2 √ 1
− 3 + ln − 2 3 tan−1 √ − 0 + ln 1 − 2 3 tan−1 √
7 3 3
√ √ ! √ !
3π 3π √ 2 3π
= − + 6 − ln 4 − − 3 − ln 7 − 2 3 tan−1 √ +
3 3 3 3
7 √ √ 2
= 3 + ln − 3π + 2 3 tan−1 √ ≈ 1.0872.
4 3
7.6. PARTIAL FRACTIONS 477

Z 3
4
63. V = π 2 (x + 1)2
dx 1

1 x
4 A B C D
Write 2 = + 2+ + . 3

x (x + 1)2 x x x + 1 (x + 1)2
Then 4 = Ax(x + 1)2 + B(x + 1)2 + Cx2 (x + 1) + Dx2
= (A + C)x3 + (2A + B + C + D)x2 + (A + 2B)x + B.

Solving A+C =0 2A + B + C + D = 0 A + 2B = 0 B=4


gives A = −8, B = 4, C = 8, and D = 4. Thus
 Z 3 Z 3 Z 3 Z 3 
1 1 1 1
V = π −8 dx + 4 dx + 8 dx + 4 dx
1 x+1 1 (x + 1)
2 2
1 x 1 x
 3   3
4 4 x + 1
8x + 4
= π −8 ln |x| − + 8 ln |x + 1| − = π 8 ln −
x x+1 1 x x(x + 1) 1
  
4 7 2 11π
= π 8 ln − − (8 ln 2 − 6) = 8π ln + ≈ 1.3287.
3 3 3 3

Z 2
1
64. V = π
1
dx
0 (x + 1)(x + 4)
1 A B
Write = + .
(x + 1)(x + 4) x+1 x+4 2

Then 1 = A(x + 4) + B(x + 1). Setting x = −1 and x = −4


gives A = 1/3 and B = −1/3. Thus
 Z 2 Z   2
1 1 1 2 1 1 1
V =π dx − dx = π ln |x + 1| − ln |x + 4|
3 0 x+1 3 0 x+4 3 3 0
2  
π x + 1
π 1 1 π
= ln = ln − ln = ln 2 ≈ 0.7259.
3 x + 4 0 3 2 4 3

Z Z 1
4x 1
x+1−1
65. V = 2π dx = 8π dx
0 (x + 1) 0 (x + 1)
2 2 4
Z 1 Z 1 
1 1
= 8π dx − dx
0 x+1 0 (x + 1)
2
 1   
1 1
= 8π ln |x + 1| + = 8π ln 2 + − (ln 1 + 1)
x+1 0 2
= 8π ln 2 − 4π ≈ 4.8543

1
478 CHAPTER 7. TECHNIQUES OF INTEGRATION

Z 1
8x 2
66. V = 2π dx
0 (x2 + 1)(x2 + 4)
8x Ax + B Cx + D
Write = 2 + 2 .
(x2 + 1)(x + 4)
2 x +1 x +4
Then 8x = (Ax + B)(x2 + 4) + (Cx + D)(x2 + 1) 1

= (A + C)x3 + (B + D)x2 + (4A + C)x + (4B + D).

Solving A+C =0 B+D =0 4A + C = 8 4B + D = 0

gives A = 8/3, B = 0, C = −8/3, and D = 0. Thus


 Z 1 Z    1
8 x 8 1 x 4 4

V = 2π dx − dx = 2π ln(x2
+ 1) − ln(x2
+ 4)
3 0 x +1
2 3 0 x +4
2 3 3 0
1  
8π x2 + 1 8π 2 1 8π 8
= ln 2 = ln − ln = ln ≈ 3.9375.
3 x +4 0 3 5 4 3 5

Z Z
cos x 1
67. dx u = sin x, du = cos x dx = du
sin2 x + 3 sin x + 2 u2 + 3u + 2
1 A B
Write = + .
(u + 1)(u + 2) u+1 u+2
Then 1 = A(u + 2) + B(u + 1). Setting u = −1 and u = −2 gives A = 1 and B = −1. Thus
Z Z Z
cos x 1 1
dx = du − du = ln |u + 1| − ln |u + 2| + C
sin2 x + 3 sin x + 2 u+1 u+2

u + 1
+ C = ln sin x + 1 + C.

= ln
u+2 sin x + 2
Z Z Z
sin x −1 1
68. dx u = cos x, du = − sin x dx = du = du
cos2 x − cos3 x u2 − u3 u2 (u − 1)
1 A B C
Write = + 2+ .
u2 (u − 1) u u u−1
Then 1 = A(u2 − u) + B(u − 1) + Cu2 = (A + C)u2 + (−A + B)u − B.

Solving A+C =0 −A+B =0 −B =1

gives A = −1, B = −1, and C = 1. Thus


Z Z Z Z
sin x 1 1 1 1
dx = − du − du + du = − ln |u| + + ln |u − 1| + C
cos2 x − cos3 x u u2 u−1 u

u − 1 1 cos x − 1

= ln + + C = ln + 1 +C
u u cos x cos x
= ln |1 − sec x| + sec x + C.
7.6. PARTIAL FRACTIONS 479
Z Z
et 1
69. dt u = et , du = et dt = du
(et + 1)2 (et − 2) (u + 1)2 (u − 2)
1 A B C
Write = + + .
(u + 1) (u − 2)
2 u + 1 (u + 1) 2 u−2
Then 1 = A(u + 1)(u − 2) + B(u − 2) + C(u + 1)2
= (A + C)u2 + (−A + B + 2C)u + (−2A − 2B + C).

Solving A+C =0 − A + B + 2C = 0 − 2A − 2B + C = 1

gives A = −1/9, B = −1/3, and C = 1/9. Thus


Z Z Z Z
et 1 1 1 1 1 1
dt = − du − du + du
(et + 1)2 (et − 2) 9 u+1 3 (u + 1)2 9 u−2
 
1 1 1 1
= − ln |u + 1| + + ln |u − 2| + C
9 3 u+1 9
t
1 e − 2 1
= ln t + + C.
9 e + 1 3(et + 1)

Z Z Z
e2t et u−1
70. dt = et dt u = e + 1, du = e dt
t t
du =
(et + 1)3 (et + 1)3 u3
Z
1 1 1
= (u−2 − u−3 ) du = −u−1 + u−2 + C = − t +C
2 2(et + 1)2 e +1

71. y 0 = ex
Z ln 2 p
u u
L= 1 + e2x dx u2 = 1 + e2x , 2u du = 2e2x dx, dx = du = 2 du
0 e2x u −1
Z √ Z √  
5
u2 5
1
= du = 1+ du

2 u −1
2 √
2 u2 − 1
1 A B
Write = + .
u2 − 1 u−1 u+1
Then 1 = A(u + 1) + B(u − 1). Setting u = 1 and u = −1 gives A = 1/2 and B = −1/2.
Z √5 Z √5 Z √5
1 1 1 1
Thus L = √ du + du − du
2 u √
2 u − 1 2 √
2 u + 1
√ √5
  5 
1 1 1 u − 1
= u + ln |u − 1| − ln |u + 1| √ = u + ln
2 2 2 2 u + 1 √2
" √ ! √ !#
√ 1 5−1 √ 1 2−1
= 5 + ln √ − 2 + ln √ ≈ 1.2220.
2 5+1 2 2+1
Z Z Z
x3 x3 1 1
72. (a) dx = dx = (4x3 dx). Partial fraction decompo-
(x2 − 1)(x2 + 1) x4 − 1 4 x4 − 1
sition is unnecessary because the substitution u = x4 − 1, du = 4x3 dx will suffice.

You might also like